Download as pdf or txt
Download as pdf or txt
You are on page 1of 156

Placenta and Placental Hormones: PDV

The Ovarian & Endometrial Cycle There is no direct effect on the endometrium
Menstruation since it does not have the receptors for estrogen
- Process of endometrial tissue shedding with & progesterone the stimulation would have to
hemorrhage that is dependent on sex come from the ovaries
steroid hormone-directed changes in blood - Average cycle: 28 days
flow in the spiral arteries - Range: 21 to 25 days (<21 and >35
- Monthly shedding of the lining of the considered abnormal)
endometrium - Duration: 2-6 days (1 day bleeding and 10
- Desquamation of the endometrium days bleeding considered abnormal
purpose of which is to form a new growth of uterine bleeding
the endometrial layer in preparation for - Amount: 20-60 ml (<20ml and >60ml
pregnancy abnormal)
Spiral arteries Quantify the amount by number of
- Found in the functionalis layer pads used
- Terminal branches of uterine artery Qualify the type of pad used
- Main arteries which are involved in the
changes that happen during pregnancy Follicle Stimulating Hormone (FSH)
- are insterspersed in the myometrium - Responsible for feeding the growing follicle
After delivery, myometrium has to for more estrogenized effect happening to
compress or contract to tighten the the follicle during the first part of the cycle
interspersed spiral artery to prevent After the follicles grows too much, there will
postpartum bleeding be a negative feedback to the hypothalamus
Uterine artery to stop production of GnRH
- Comes from the internal iliac artery/ LH will now increase to facilitate ovulation
hypogastric artery Ovulation primary purpose is to expel the
- Supply the lateral portions of the uterus ovum inside the mature follicle

Hypogastric artery Problems affecting the hypothalamus and


- Branch of common iliac artery pituitary gland would have its effect on the
Common iliac artery end organ causing irregular bleeding (ex.
- Branch of abdominal artery Amenorrhea)
Ovarian artery
- Supply the fundal portion of the uterus Strenuous activities GnRH FSH
e rogen leads to unovulatory cycles
The uterus must be properly nourished and
supplied with a lot of vasculature because it Overweight patients e rogen d e o
needs a lot of these blood vessels to supply increased peripheral conversion of
with the necessary blood at the time of androstenedione in the adipose cells to
pregnancy. estrone contributing to the thickening of the
This is also associated with changes with lining of the uterus unovulatory menses
regards to malignancy wherein its spread
has something to do with the vasculature Stress causes abnormal/ Irregular
supplying the uterus menstruations

Upper third of the posterior wall of the uterus/


decidua Ovarian Cycle
- blastocyst implant after it has undergone - 2 phases:
changes from fertilization Follicular (preovulatory) formation/
Decidua growth of follicle
- Endometrium of the pregnancy Luteal (postovulatory) ovum is
being readies to be expelled from
Normal menstrual cycle: 28 days the mature follicle in preparation for
It is important to identify which part of this cycle the meeting with the sperm
would belong to the first or second part because From the time we have been conceived in
this would give us the idea when to suggest to utero, at 20 weeks 6-7 million oocytes
the patient when they are likely ovulating or formed
when is their best chance of getting pregnant These oocytes undergo degeneration,
The Ovarian & Endometrial Cycle atresia/ follicular atresia
- Complex interaction: At birth 1-2million oocytes left
Hypothalamus-pituitary axis Puberty 300,000available for ovulation
Ovaries Only 400-500 will be released for ovulation
Genital Tract
The hypothalamus would have to secrete GnRH Follicular phase
to stimulate the pituitary gland to produce FSH - Recruitment of the follicles from the resting
and LH pool after which a cohort if formed
FSH and LH would have its effect over the - Growth to the antral stage cohort
ovaries for it to secrete estrogen and - From the cohort, the meanest of the follicles
progesterone would have to survive the effects of FSH
Estrogen and progesterone would then have its selected graafian follicle/mature follicle
consequent effect over the endometrium Graafian follicle contains the egg
to be extruded

1
Placenta and Placental Hormones: PDV
- These changes undergo two cell-two due to decrease LH and decreased
gonadotropin theory LH sensitivity of luteal cells
Formation of progesterone happens
when this is inside the thecal cells Estrogen
In the presence of LH in the thecal - 17 - estradiol Most biologically potent
cells, C19 steroids are converted - Secreted by the granulosa cells coming from
into cholesterol the dominant follicle and the luteinized
Cholesterol converted into granulosa cells of the corpus luteum
androstenedione which crosses the - Has 2 receptors:
basement membrane E rogen recep or (ER )
With the action of FSH attaching to responsible for most of the changes
a receptor in the granulosa cell part in the endometrium
in the presence of cAMP, E rogen recep or (ER ) glands
androstenedione is converted to and stroma
estrone - Roles:
Estrone converted to the potent Synthesis of proteins from receptor-
17 - estradiol specific gene transcription
17 - estradiol goes out of the At the cell surface: stimulate nitric
granulosa cell and have its effect on acid production in endothelial cells
the endometrium Replication of the endometrium
TAKE NOTE: Theca cells LH
(True Love) Progesterone
Granulosa cells - Predominant hormone in the luteal phase
FSH (Girl Friend) - Mediated through nuclear hormonal
- Follicle must be ready for ovulation to receptors
happen - 2 receptors:
Proge erone recep or
Ovulation
Proge erone recep or
- Onset of gonadotropin surge 34-46 hours
responsible for most of the glandular
before ovulation
secretion during the luteal phase of
- Peak of LH 10-12 hours before ovulation
the cycle
Ovulatory kit recognizes the high
- Subnuclear vacuolation indication of high
level of LH which indicates ovulation
levels of progesterone and that ovulation
- In the presence of progesterone and
has started
prostaglandins, a stigma happens over the
wall of the graafian follicle weak portion
Endometrial Cycle
ruptures release of oocyte-cumulus
- Has 2 phase:
complex
Proliferative phase (preovulatory)
Graafian follicle contains fluid
noted on ultrasound as fluid in the Secretory phase (postovulatory)
cul de sac indicates ovulation - Synonymous with the menstrual cycle
First day of the endometrial
- Symptomatic indications:
cycle is the first day of the
breast tenderness due to
menses
hormone stimulation
- Can be dated based on the:
pain in hypogastric area due to
Epithelial (glandular)
fluid coming out from the follicle
that irritates the peritoneal cavity Stromal (mesenchymal)
Presence of blood vessels
Luteal Phase More tortuous vessels on
- primarily responsible for the formation of the the latter part of the cycle
corpus luteum and more compact stroma
corpus luteum produces needed for the implantation
progesterone of the blastocyst
- proge erone, e rogen Proliferative Phase
- If no pregnanct happens, corpus luteum - Corresponds to the follicular phase of the
regresses and menstruation happens ovarian cycle
- Responsible for luteinization and - Production of estradiol
neovascularization edematous - Largely stimulated by estrogen
endometrial lining to have the necessary - Day 5 restoration and revascularization of
stroma for the implantation of the blastocyst the endometrium to help rebuild/
Vascular endothelial growth factor proliferation of the endometrium for the next
responsible for neovascularization cycle
- Lifespan is dependent on the presence of Evident by the presence of
LH and HCG thin, tubular and narrow
During pregnancy corpus luteum glands
is maintained continuous - Reepithelialization and angiogenesis stops
production of progesterone bleeding
pregnancy goes on until term Estrogen is responsible
Placenta takes over production of because it increases
progesterone after 8-12 weeks vascular endothelial growth
No pregnancy corpus luteum factor needed for
regresses 9-11 days after ovulation angiogenesis
2
Placenta and Placental Hormones: PDV
- Hyperplasia and increase in stromal ground
substance Mentruation
Estrogen induces growth - Corpus luteum production of progesterone
factor gene expression in drops
stromal cells - Psuedoinflammatory appearance
- Makes the cycle variable (5-7 days; 21-30 - Infiltration of leukocytes important key
days) factors in the initiation of the extracellular
Luteal phase of the ovarian matrix breakdown of the functionalis layer of
cycle and secretory phase the decidua
in endometrial cycle os - Severe coiling of the arteries causing
constant in duration 12- hypoxia of the endometrium and
14 days consequently endometrial ischemia tissue
degeneration
Best time for patient to get pregnant: Same mechanism happens in
- Ask the patient her cycles for the last 4-5 patients with abruption placenta
months Abruption placenta placenta
- Compute the intervals from which this becomes detached from the decidua
patient has her normal cycles because a hematoma happens
- Subtract 14 between the decidua and plancenta
- E . If pa ien normal c cle i 29 da 29- The same manner that a ruptured,
14= 15 day 15 is the time the patient is coiled artery may produce
fertile ovulates hematoma, thus causes the
Inform couple to have shedding of the functionalis layer
sexual contact for 3 with the basalis layer still intact
consecutive days for (basalis layer is left for endometrial
successful conception to growth to happen
happen - Infiltration of neutrophils one to two days
before menstruation: (IL-8)
Secretory Phase - MCP-1= monocytes
- Part of the cycle that tells us that ovulation
has happened Pain during menses:
- Dating based on the histology of glandular - Because of the effect of the prostaglandins
epithelium par ic larl PGF2 causes
- Day 17: glycogen accumulation, vasiconstriction
pseudostratification, subnuclear vacuoles - Vasoactive peptides-- endothelin-
are noted over the apical portion of the enkephalinase system vasoconstrictors
endometrium over the peripheral portion Vasoconstriction endometrial
first sign of ovulation ischemia manifested as
GnRH & LH dysmenorrhea/ pain
Proge erone After endometrial ischemia, tissue
Ovum has been extruded desquamation will no ensue
from graafian follicle
- Phase where there are a lot of tortuous
glands and a lot of mitosis is happening Changes in the ovarian and endometrial
- Day 18: vacuoles moved to the apical cycle are the bases whether a patient will be
portion highly successful in achieving pregnancy or
- Day 19: glycoprotein and not
mucopolysaccharide contents released; Effects of progesterone and estrogen does
mitotic activity ceases not only focus on the ovaries and the
- Day 21-24: stroma becomes edematous endometrium
- Day 22-25: stromal mitosis apparent It also has its effect on the body causing a
Day 20-24: window of implantation rise in the basal body temperature one
(endometrium must be ready for basis in predicting ovulation
successful implantation) Progesterone has a thermogenic effect
Pinopods protrusions of the apical causes the increase in basal body
surface in to the lumen; helpful temperature and the feverish feeling during
during the time of implantation pregnancy
Continuous growth of spiral arteries The moment the temperature rises and that
at this time for continuous temperature is sustained after 14 days
nourishment of the endometrium in chances of pregnancy has likely happened
preparation for the decidua
Day 22 I m read for o da in
the endometrium

Decidualizarion
- Happens because of preogesterone
If corpus luteum is rescued and
there is continued secretion of
progesterone, the endometrium will
be decidualized
LH and HCG maintains the corpus
luteum; basis for the pregnancy test
3
Placenta and Placental Hormones: PDV
FERTILIZATION
N ri ion of he de eloping embryo
- Birth 2,000,000 oocytes
- Adolescence 400,000 Endocrine f nc ion secretes hormones

- Reproductive period 400 undergo EMBRYOBLAST make up the inner cell


maturation -> ovulation mass which becomes the embryo, umbilical
cord, and
Primary follicles amnion
st
1 meiotic division: arrested in prophase of
(completed just before ovulation) Remember: TOE (Trophoblast:Outer,
nd
2 meiotic division halted in metaphase Embryoblast:Enner)
(completed upon fertilization of ovum)

- Oocytes must be fertilized within 12 hours IMPLANTATION


(cannot survive for more than 24 hours) - 6-7 days after fertilization
- happens in the Ampulla (*Fallopian tube is
7-8 cm) Stages:

A. Apposition blastocyst adherence to the upper


Upon fertilization, the oocyte undergoes many part of the posterior uterine wall (superficial
changes: adherence)

o Embryonic pole (contains the embryoblast)


attaches first
o Implantation window adhesion of
trophoblast and endometrium is maximal
and is only present at
this time

vitro fertilizations
***Dra. said that there is a step called
Adherence hich i j a deeper implan a ion
1. Zygote - formed by fusion of 2 haploid
gametes (46 chromosomes) B. Invasion penetration of the blastocyst to the
- union of the egg and the sperm inner 1/3 layer. Cells of the trophoblast of the
2. Blastomere further divisions (myosis and implanting embryonic pole proliferate to form 2
mitosis) make the zygote go through different layers:
stages as it
travels in the fallopian tube (2 cell stage, 4 cell, 8 1. Syncitiotrophoblast outer layer facing
cell, 16 cell) It will happen three days in the maternal tissue; fused
fallopian tube. 2. Cytotrophoblast remaining unfused
O Transported by peristaltic and cellular components
ciliary movement
3. Morula solid ball of cells formed; Remember: SOCI (syncitio:outer; cyto:inner)
enters uterine cavity 3 days after fertilization or
once it reaches the 16 cell stage ** Trophoblast invasion does not penetrate beyond the
- the one that enters the uterine cavity and inner third of myometrium
rd
implants in the upper 3
(Ectopic/Tubal pregnancy implants in the Germinal cell

fallopian tube) Camp

On FM

* later on it will accumulate more fluid between - Trophoutronectin

the cells/blastomeres or when there is fluid Cytotrophoblast
- Trophoblastic glue
accumulation it becomes the blastocyst
Facilitates separation of tissue
at delivery
4. Blastocyst: implants on endometrial lining
(6-7 days after fertilization)
Syncitiotrophoblast
- the fluid accumulation will cause Secretory cell
preferential flow (the cells to be pushed on one
side). One side is called inner cell mass which
C. Other developments after implantation
will become the embryon and the outer cell
Prelacunar & lacunar stages
mass is the placenta and trophoblast.
Trophoblast invasion
- Most of the blastocyst cells make up
Villous development
the TROPHOBLAST (outer wall that becomes
Development of chorion and decidua
the fetal membranes and the placenta)
1. Prelacunar stage
TROPHOBLAST (functions)
- Characteristic feature: solid
syncitiotrophoblast mass with finger-like extensions
invading the endometrium
A achmen of bla oc o he er
1
Placenta and Placental Hormones: PDV
- fe al a me he hape of a h man
2. Lacunar stage
-DAY 8: Formation of lacunae (confluence of 5. Development of Chorion and Decidua
vacuoles in the syncitiotrophoblast) a. Decidua basalis
- Decidua at implantation site/the placenta
-Trabeculae: separating syncitiotrophoblast - Chorionic villi at this site will proliferate and form
- DAY 12: Uterine epithelium closes over the chorion frondosum / leafy chorion (fore runner of
implantation site the placenta)
*Completed syncitiotrophoblast covered by - Very vascular
layer of incomplete cytotrophoblast
b. Decidua capsularis
3. Trophoblast invasion of the - Envelops the blastocyst after completion of
endometrium implantation
- Covers/encapsulates the embryo
- Signal of invasion: proliferation and - Covers the side facing the endometrial cavity
migration of cytotrophoblast at the bottom of the shell - Chorionic villi here degenerates & becomes
- Significance of this event: avascular because of pressure due to growth
o Further invasion of the blastocyst blood is compromised = chorion laeve / smooth
o Helps the maternal vessels adapt to chorion (to avoid misspelling, remember that chorion
pregnancy conditions laeve is spelled like leave, but the vowels are
o Anchorage of the developing alphabetically arranged LAEVE)
placenta
- Trophoblast is seen as an irritant and c. Decidua parietalis
causes release of hormones that promote - Rest of the decidua that is not in contact with
endometrial stroma blastocyst
proliferation and enlargement = formation of
decidual cells ** There is this vestigial cavity between the decidua
- Syncitiotrophoblast invasion causes the capsularis and parietalis.
maternal endometrial vessels to disintegrate
causing maternal blood to enter through the - After 14 16 (15) weeks the smooth Chorion and
lacunae (remember that these were formed Decidua Capsularis fuse together with the Decidua
during the lacunar stage ) Parietalis due to the growth of the fetus and
becomes Decidua Vera
o The syncitio now becomes the - Clinical Significance: Before 15 weeks,
capillary walls and forms new lacunae -> complete intrauterine infection can set in due to the
perfusion of communication between the two (ascending
the entire lacunar system infection). After 15 weeks, the fetus is protected from
o Spiral arteries erode due to deeper the ascending infection.
invasion = increase intralacunar Con ac be een he moo h Chorion and
pressure = FIRST REAL maternal Decidual surface of uterine wall allows
circulation of the placenta paraplacental exchange.

4. Villous development (Sabi ni Williams 24/E)


The decidua parietalis and basalis are composed of
- Lacunar system is transformed into the intervillous three layers.
space 1. Zona Compacta surface
- Primary villi: longitudinal cell columns that protrude 2. Zona Spongiosa middle; with remnants of
into the lacunae glands and numerous small blood vessels
- Secondary villi: transformed when mesenchymal 3. Zona Basalis Contains the endometrial gands
cells from extraembryonic chorionic plate invade the and stroma
villi The ona compac a and pongio a oge her form
- Tertiary villi: marked by the appearance of capillaries the zona functionalis. The basal zone MUST remain
in villous stroma after delivery and give rise to new endometrium
because without it the woman becomes infertile.
o Complete fetoplacental circulation: established at
The Ni ab ch la er i a one of fibrinoid
th
5 week degeneration in which invading trophoblasts meet
the decidua basalis. If the decidua is defective, as in
placenta accreta, the Nitabuch layer is usually
ab en Thi la er hich epara e he ba ali from
the placenta and the myometrium.

(Sorry, may tin ro i a di o pero hi lang inabi


ni a oooo, e an ko ano ng hi b i abo
the Fibrion/ Nitabuch layer. So I got that statement
from Williams)

DECIDUALIZATION
Fun Fact: - Change that occurs in the endometrium in
Embryo 8 eek and belo ; adpole looking response to blastocyst implantation
Fetus - 8 weeks and above
2
Placenta and Placental Hormones: PDV
- The endome ri m ran form in o he decid a rd
(3 trimester) exchange due to high degree of
because of hormones. with fetal vascularisation & large
- Decidualized endometrial cells are called branching exchange surface
decidual cells angiogenesis - prominent sites for hormone
Remember: Decidua is the endometrium of the production
pregnant woman - high degree of fetal
Con ain a fe T cell b no B cell (rea on h capillaries & dilated
trophoblast is not recognized by the endometrium) sinusoids (responsible for
D. Ba ali : con ain ma ernal macrophage /c slowing down blood flow to
are able to phagocytose cell debris and immune Terminal villi provide ample time for
complexes w/ fetomaternal exchange; area
- Walls of spiral arteries have no nonbranching of greatest diffusional
macrophages and are thus invaded by the Angiogenesis exchange)
trophoblastic cells (decrease in - MAIN site of fetomaternal
- Macrophage function inhibited by VEGF and exchange
progesterone increase in
placental GF) (O2, CO2, H2O
PLACENTA transfer)
- Major source of nutrients of the fetus in - reduction in terminal villi =
utero may lead to fetal hypoxia
- Produce hormones that maintain the
pregnancy
- Main function: oxygenate blood from fetus
- Insight into prenatal life
- Helpful in caring for neonate
- Plan the future care for mother/child
- Fetal side of the placenta: glistening, covered
by amnion
- Maternal side: rough ; attached in the D.
Basalis
- Main cell comprises of the Trophoblast
** Basically, villi are present in the placenta to
Characteristics of Placenta: facilitate exchange of nutrients, oxygen, etc between
the mother and the fetus. Thus, blood flow and BP
- HEMOCHORIAL control is very important in the intervillous space.
o HEMO: Maternal blood (from spiral arteries) is
localized only to the outsides of the endothelium of - Major mechanism in the pathogenesis of
the maternal vascular system intrauterine growth restriction (IUGR) with
- Directly bathes the syncitiotrophoblast absence of reverse end-diastolic umbilical flow
(where exchange of gases & nutrients occur) (ARED)

o CHORIO: chorion-placenta separated from - Myofibroblasts are present in stem and anchoring
fetal blood by endothelial wall of fetal capillaries villi to help relax the capillaries when there is an
o Fetal capillary blood confined in the villous increase in pressure in IV space
core
o There is no mixing of maternal and fetal blood NONVILLOUS PARTS OF PLACENTA
(may come in close contact but never mixing)
- Separated by the placental barrier 1. Extravillous trophoblasts
composed of the ff layers: - Responsible for invasion of maternal tissues and
Syncitiotrophoblast lining the vessels
intervillous space
C o rophobla (Langhan cell )
Trophoblastic basal lamina
Connective tissue
Fetal endothelium

VILLOUS
Villi Type Function
- Mechanically support the
structures of the villous trees
Stem Villi - Negligible part in
fetomaternal exchange and
endocrine activity
- Growth centers of villous
Immature trees
intermediate (placentones) 2. Fibrinoid
villi - principal sites of exchange - Nonfibrous, noncellular, homogenous
during first 2 trimesters
Mature - contain capillaries, arterioles, 3. Decidualized endometrial stroma (see decidua
intermediate venules on page 1)
villi - important for fetomaternal
3
Placenta and Placental Hormones: PDV
PLACENTONE THEORY OF SCHUHMANN & FACTORS REGULATING BLOOD FLOW TO
WEHLER Placentone: PLACENTA
fetomaternal circulatory unit is composed of 1 1. Blood pressure of mother increased BP of
villous tree + related centrifugally perfused mother will decrease blood flow to the placenta; Low
intervillous space BP poor circulation Hypoxic baby
The arrows represent the maternal blood at it enters 2. Intrauterine pressure will also decrease blood
the center of the villious tree and leaves through the flow to placenta
clefts between neighboring villious trees. 3. Type of uterine contraction
Ex. Hypertonic contractions (one after
- One or a few villous trees = 1 placental lobule/ another)/Tetanic No relaxation = no pooling of
maternal cotyledon blood
- 15-20 lobes divided by Fibro septae 4. Factors affecting arteriolar wall PTH and
- to check: put on a table and look for Endothelin I (chorionic vessels)
missing segments (incomplete can cause maternal
hemorrhage/infection/suninvolution) ABNORMAL:
H-mole (Hyatidiform mole)

VASCULOGENESIS & ANGIOGENESIS IN


PLACENTA
- Normal vessel formation is regulated by the ff:
VEGF (Vascular Endothelial Growth Factor), partial
pressure of Oxygen in the fetoplacental vessels
Imbalance in an of he e ma lead o
preeclampsia/ IUGR

- Vessel formation involves 2 processes:


- Placenta develops; with fertilization; with
implantation
1. Vasculogenesis de novo formation from
- No fetus
mesodermally derived precursor cells
- Pregnancy is a lot bigger than expected because of
2. Angiogenesis creation of new vessel
the rapid growth of the placenta
branches from pre-existing ones
- Grows every 2 days (twice the amount)
Branching angiogene i : Principal
- Swollen and avascular
type; occurs during development of the villi =
- Vesicles that are grape like
primitive
- Proliferation of the trophoblastic cells
capillary network; mainly
stimulated by VEGF
Nonbranching angiogene i :
Coincide with development of
mature intermediate villi
rd
(3 trimester) and terminal villi;
lasts until term

REGULATORS OF A & V
1. VEGF
- potent stimulator of growth and survival of
blood vessels
o High levels in early pregnancy promotes - Small caliber compared to the rest of the umbilical
formation of widely branched low resistance cord (normal length: 55 100 cm). If more than that
capillaries (branching angiogenesis) it can twist and wrap around the fetus/cord coil
o Decreased levels in second half of sudden fetal death
pregnancy ***Always monitor the fetal movement thus report
o Affected by O2 levels; low O2 levels changes in fetal movement to see the location of the
stimulate upregulation of VEGF -> umbilical cord
compensatory formation of blood vessels
- Umbilical cord i co ered i h Whar on jell and
2. PlGF contains 2 arteries and 1 vein. The absence of 1
- High levels in second half of pregnancy, during the artery is the most common vascular anomaly in the
period of the most dramatic nonbranching human being but it is compatible with life.
angiogenesis

** Balance between VEGF & PlGF = balance


between branching & nonbranching angiogenesis
** if VEGF > PlGF: predominance of branching
angiogenesis = highly branched, convoluted, short,
terminal villi (seen in preplacental & uteroplacental
hypoxia)
** if PlGF > VEGF: predominance of
nonbranching angiogenesis = long filiform terminal
villi
(seen in postplacental hypoxia) - Umbilical cord is capable of having blood vessel
anomalies. Some are tortious and we call these
varix/varices or some call it as a pseudoknot.
4
Placenta and Placental Hormones: PDV
Underneath it is just an accumulation of blood increased fetoplacental
vessels which are swollen similar to the concept of flow impedance
varicose veins in pregnant women.

FETAL CIRCULATION
- Uteroplacental arteries are branches
of myometrial arteries. As they enter the decidua,
they are also
called spiral arteries (due to spiral course)
- Deoxygenated blood is carried to the
placenta via 2 umbilical arteries in the umbilical cord
- This is because of the redundancy of the umbilical - Single umbilical vein carries oxygenated
cord, some twists/ screw like along the length of the blood back to fetus
umbilical cord. The tighter this twist becomes - Umbilical vessels enter placenta and branch
untoward events can happen to the fetus. into capillary networks
- Exchange of substances passively diffuse
from capillary networks to intervillous space
- Maternal blood bathes
syncitiotrophoblast -> oxygenated blood returns
from placenta to the fetus
through the single umbilical vein
- Maternal oxygen supply has a strong impact
on villous growth and differentiation
o Low maternal O2 may lead to
underdevelopment of the villous and cause fetal
hypoxia
- Long cord and had undergone twisting or torsion. - No actual mixing of fetal and maternal blood
Mother will verbalize that no fetal movement is felt. just diffusion and osmosis
Upon examination,no more cardiac activity. PLACENTAL AGING
Evacuation/Induced labor is important because - Premature placenta may affect fetal growth
leaving the fetus in utero will lead to DIC. Cesarean - As it ages, there is also an increasing impairment
section is not needed unless there are some other of function
maternal indications.
Characteristics/Features of aging:

o Decrease syncytial thickness


o Lo of Langhan cell
o Decrease stroma
o Increased capillaries
o Thickened basement membrane
o Obliteration of blood vessel villi
o Fibrin deposits
- Pathologic placenta
o Hydropic degeneration of placenta (occurs
in molar pregnancies)
- Nuchal Cord coil sudden fetal death (most
common) ***

PLACENTAL MEMBRANES
Different Origins of Fetal Hypoxia 3 distinct layers:
1. Amnion
Hypoxia Causes 2. Chorion laeve/smooth chorion
maternal anemia, 3. Decidua capsularis
Preplacental
cyanotic maternal
hypoxia
cardiac diseases, Trophoblast is subdivided into:
mother, placenta, &
pregnancy in high a. Basal chorion cells surrounding the
fetus are hypoxic
altitude implantation pole
Uteroplacental o Becomes the chorion frondosum (tree
Impairment in
Mother (normal) like) with appearance of the 1st villi
uteroplacental circulation
Placenta and fetus b. Capsular chorion cells surrounding the
(preeclampsia)
(hypoxic) anti-implantation pole
IUGR w/o umbilical end- o Degenerates by 3rd week, intervillous
Postplacental
diastolic flow space obliterates, trophoblastic cells fuse to
Fetus (hypoxic) Mother
form chorion laeve (smooth chorion)
(normoxic) Placenta
Poorly developed
( h pero ia
terminal villous AMNION
- higher O2 levels
capillaries, absent - Composed of inner layer of epithelial cells
than normal)
capillary branching = - Easily separated from underlying chorion
5
Placenta and Placental Hormones: PDV
- Contains Amniotic Fluid - Contains 2 arteries & 1 vein (left umbilical vein;
- Avascular - no blood vessels; obtains nutrition right vein disappears early in life)
and O2 from surrounding chorionic fluid, amniotic - Artery deoxygenated blood; Vein
fluid, and fetal oxygenated
surface vessels **Chorionic vessels branches of these
- Intramniotic lipid synthesis umbilical vessels seen on the surface of the
- Greater tensile strength than chorion placenta
- to maintain pregnancy until 9 months - central wider vessels and periphery
- if weak (ex. infection) there might be a smaller
rupture Amnionitis
- Amnionic epithelium flat, cuboidal
to columnar cells - Intraabdominal portion of the duct atrophies and
o Exclusive source of PGE2 di appear . If i remain : Meckel di erticulum
o Source of interleukins - Average length: 55cm (range: 30-100cm)
(regulateprostaglandin synthesis) Diameter: 0.8 2cm
Prostaglandins: significant role in
initiation and maintenance of uterine Possible pathologies:
contractions
Intraamnionic infections induce False knots: folding and tortuosity of vessels
production of IL -> increase PG longer than umbilical cord
production Pro r ion of Whar on jell
o Produce leukotrienes Constriction of some portions of the cord
o Produces Endothelin I potent Torsion of umbilical cord
vasoconstrictor (if increased, it will * Usually in very long cords
compromise circulation with fetus IUGR Ma ca e fe al dea h
(Intraunterine Growth restriction), amniotic Mi concep ion: if he mbilical cord i rro nding
fluid homeostasis; promotes H2O transfer the neck, it will cause death of the baby and is an
across epithelia of other organs immediate indication to do a caesarean section.
o Contains carbonic anhydrase isoenzymes NOT TRUE! For it to actually cause death, it has to
(responsible for regulating pH of amniotic fluid to be tightly wrapped around the neck.
7.0) Cord coil
- E racell lar ma ri i called Whar on jell
- Contain oxytocin receptors
- Membrane that contains amniotic fluid PLACENTAL HORMONES:
- Solute and water transport to maintain
A.F> Hemostasis 1. Estrogen
- Produces hormones: Pregnancy is hyperestrogenic state
o Parathyroid hormone related protein Syncitiotrophoblast
Vasodilator (must in balance with Endothelin Ovaries 2-4 weeks post-ovulation
I) 7th Week > 50% placenta
o Enkaphalinase
- Modulates chorionic vessel tone and blood flow
Precursors
Source of Amniotic Fluid: DHEAs (main)
1. Possible secretory process of amnionic epithelium Androstenodione
2. Filtration of fluid from maternal vessels via Testosterone
deciduas parietalis and chorion laeve
3. Filtration from fetal vessels or in the chorionic Decreased Estrogen
plate and via umbilical cord Fetal Anencephaly
4. Urination of fetus Maternal Adrenal Dysfunction
5. Filtration from intracorporeal fetal vessels via Maternal ovarian androgen producing
fetal skin tumors
**Functioning GI, GU, Pulmonary, Integumentary to Placental sulfatase deficiency
have normal amount 1000cc / 1L towards term Placental aromatase deficiency
Hydramnios more than 1L may be cause by Glucocorticoid Treatment
problems in circulation/GI/Atresia/obstruction and Deficiency in fetal LDL synthesis
less is oligohydramnios may have problems in
Decreased fetal adrenal utilization of
GU/renal agenisis/obstruction
LDL
UMBILICAL CORD
2. Progesterone
- 18 days: embryo is a flattened disc between
Source: Syncitiotrophoblast
amnion and yolk sac
Utilization of Maternal LDL
- Middle of 3rd month: amnion obliterates the
exocoelom, fuses with chorion laeve and covers the
* when there is ovulation there is Trophoblast
bulging placental disc and the lateral surface of the
formation and implantation there is Corpus Luteum
body stalk
production source of progesterone fro the next 6 -
- Body stalk called funis or umbilical cord
7 weeks. After that, the placenta takes over.
- Must be in the central or para central area to get
better blood flow
3. Human Chrorionic Gonadotrophins (HCG)
- Fetal umbilicus to the fetal surface of the
Syncitiotrophoblast
placenta
Alpha & Beta Subunits

6
Placenta and Placental Hormones: PDV
Structural related to LH, FSH and TSH
Alpha sub-units identical
Beta sub-units are distinct
Beta subunit: basis for pregnancy kit (5
drop of rine in he indo of he ki
= 1 line means negative and 2 lines
means positive) The kit contains
antibodies coagulation happens with
HCG
Hormone of pregnancy
Increases between 9-10 weeks AOG
down at term
Amount is directly related to no of
Trophoblast
Doubles every 2 days if negative, try
again after 2 days

Significance: Increased HCG means increased


TSH thus the thyroid gland enlarges
symptoms of hyperthyroidism. Same correlation
goes to an enlarged ovaries because of FSH
thus we need monitoring.

• Functions
Rescue & maintenance of corpus
Luteum
Stimulate Leydig cells of the fetal
testes
Induces ovulation (Infertile women)
• Increased HCG
Multiple pregnancies
Fetal hydrops
H-mole
Choriocarcinoma

4. Human Placental Lactogen (HPL)


Chorionic Somatotropin
Chorionic Growth Hormone
Syncitiotrophoblast
Detected on 2nd or 3rd week of
fertilization
Structurally related to Human growth
hormone and human prolactin
• HPL Synthesis
Stimulated by insulin
Inhibited by PGE2 & PGF2
alpha
Greatest hormone in humans
Present within 5-10 days after
conception
Concentration rises till 34-36
th th
weeks AOG *peak 6 7
month
Proportional to placental mass
• Metabolic actions of HPL
Lipolysis and increased
circulating free fatty acid
reason why it is a Diabetogenic
Anti-insulin
Increased maternal insulin
CHON synthesis thus no fetal
starvation
A.A. for transport of fetus thus
no fetal starvation

********There are others but you can just read on


i . I i and I q o e Nice o Kno . If o are
not nice, then you know what to do. :P

7
MATERNAL PHYSIOLOGY: PDV
MATERNAL ADAPTATIONS TO PREGNANCY Due to a more rapid increase in length than
width
REPRODUCTIVE TRACT Shape pushes uterus out of pelvic cavity
displace the intestines laterally and
A. Uterus superiorly, almost the tip of liver
1. Increase in size (attributed to the ff o Dextrorotation (right) caused by the
changes:) rectosigmoid colon on the left side of the pelvis
o Standing causes extension of the pelvic inlet axis.
To accommodate the growing fetus,
The abdominal wall supports the uterus maintains
placenta, amniotic fluid
the relation between the long axis uterus and axis
- At term, uterine contents = 5L 20L,
of the pelvic inlet.
weight: 1100g
o Supine uterus falls back t the vertebral column
70 g and is almost solid,
and adjacent great vessels.
except for a cavity of 10 mL or less
o Stretching of myometrium
By the end of pregnancy 500 to 1000 times greater o Fundal
o Round & broad ligament are located already
Thinning of the wall (1-2cm at thickness at term) midway the uterus due to its ascent to the
Hypertrophy of smooth muscle (myometrial) cells abdomen because of the tilting of the
(hypertrophy > hyperplasia) dextrorotation
- Production of new myocytes is limited o Consistency soft uterus if because of pregnancy
- because of the stretch it becomes more and solid if tumor of the musculature
of hypertrophy Vascularity causes softness
Increase in fibrous & elastic tissues Bimanual examination - One hand at the
- Adds strength to uterine wall vagina/cervix one hand at the abdomen top (one
o Increase in size and interal + one external)
number of lymphatics & blood vessels Proce ed o ge he ol me
Hypertrophy of nerves Upper limit fundus to note the number of weeks
o Asymmetrical enlargement Size is important in knowing if the baby is growing
= more pronounced at the FUNDUS well SGA or too big; rapid growth; GDM
- More rapid growth at the part of the Tape measure put one end at the symphysis pubis
uterus surrounding the placental site another is on the top part (fundus)
- Early pregnancy the fallopian tubes - correspondence in height in cm and weeks
and ovarian and round ligament attach ex.. 18 weeks = 18 cm but after 36 it will become
only slightly below the apex of the smaller
fundus. 20 weeks at level of umbilicus
- Later months, located above the middle 16 weeks 2 finger breadths below the umbilicus
In ectopic pregnancies: growth still takes 24 weeks 2 -3 finger breadths above the umbilicus
place because of the
hormones Appendix pushed up but this does not mean that we
Stimuli for change: do not check on the possibility of Appendicitis if
- Before 12 weeks: Hormonal - with right lower quadrant pain
(estrogen & progesterone)
- Polyamines (spermidine, spermine, Myoma can cause the uterus to enlarge
putrescine) 3. + Hegar sign
- After 12 weeks: Softening of the isthmus +
mechanical distension by uterine disappearance -> replaced by the lower uterine
contents is responsible thus it still segment
stretches outside labor Reappears after delivery

occur entirely in response to mechanical distention 4. Contractions


by the products of conception, because similar Early pregnancy: painless, irregular
uterine changes are observed with ectopic contractions because of the stretch in muscles
pregnancy o 2nd trimester: contractions detected by
Myocyte Arrangement bimanual palpation
- uterine musculature: 3 strata Braxton - Hicks contractions: appear
Hoodlike layer arches over the fundus and extends unpredictably, sporadically, nonrhythmic
to the various ligaments - Strong palpable irregular contractions (5
Middle layer dense network perforated in all 25mm Hg)
directions by the BV. Forms most of the - Encountered usually during last two weeks
uterine wall. Each cell has a double of pregnancy
c r e like an 8 crucial because - May occur every 10 to 20 minutes
the cells contract after delivery, they - May cause false labor pains
constrict penetrating blood vessels Factors affecting contractility:
and thus act as ligatures. - Estrogen increased function of the
Internal Layer w/ spincter like fibers around the FT biochemical contractile system of uterine
orifices and internal cervical os. smooth muscle
- Progesterone maintains high membrane
2. The uterus becomes an potential to block myometrial activity
abdominal organ (from being a pelvic - Biochemical control: increase contraction
organ) of actomyosin, increase muscle excitability
Few weeks piriform/ pear through Ca, PGE2 and PGF2
- ANS innervations
Becomes ovoid/globular (end of 12th week)
1
MATERNAL PHYSIOLOGY: PDV
* Know the difference between the contractions: Estrogen * sa lecture both E
before labor and towards labor and P daw but more of P
meaning ferning i n normal in
5. Uteroplacental Blood Flow
Placental perfusion is dependent on total uterine pregnancy
blood flow
Increase progressively during pregnancy 7. Others
Estimates range from 500 to 750 mL/min o Basal cells near the squamocolumnar junction
Nonpregnant woman is 5000 mL/min - prominent in size, shape, and staining
Uterine veins reduced elastin content and qualities (Estrogen
adrenergic nerve density increased venous induced)
caliber and distensibility.. Arias-Stella reaction
Uterine artery diameter doubled by 20 weeks o - both endocervical gland hyperplasia and
17 -estradiol has been shown to promote hypersecretory appearance, which makes
uterine artery vasodilation and reduce uterine the differentiation of these and atypical
vascular resistance glandular cells on Pap smear particularly
difficult

B. Cervix C. Ovaries
* speculum bivalve instrument used for IE 1. Amenorrhea (due to arrested
1. + G de (as early as 1 follicular maturation and ovulation)
mo after conception)
Softening of the cervix due to: (parang lips) 2. + Corpus Luteum (until 6-7wks
- Increased vascularity & edema AOG)
- Hyperplasia & hypertrophy of the Produce progesterone
cervical glands = leukorrhea Releases relaxin remodels CT of repro tract +
(excessive secretions) allow successful pregnancy & labor
- can also be caused by hormonal upon decrease in hormone it will be rescued by
HCG
2. Cyanosis of the cervix Estrogen and Progesterone -
Not all bluish cervix are pregnant; cervical no release of FSH and LH;
cyanosis may also be caused by oral source is placenta kaya raw
contraceptives parating mataas ang level
Violicious in color 3. Extrauterine decidual reaction
(On and beneath the surface of the ovaries is
3. Rearrangement of collagen-rich common)
connective tissue - Elevated patches of tissue bleed easily and
For maintenance of pregnancy, dilatation during may (resemble freshly torn adhesions)
labor, repair following parturition Similar decidual reactions are seen on the uterine
Decreases collagen and proteoglycan serosa and other pelvic, or even
concentrations and increases water content extrapelvic, abdominal organs
- regulated in part by localized estrogen o These areas arise from subcoelomic mesenchyme
and progesterone metabolism (progesterone stimulation and histologically
appear similar to progestin-
4. Cervical Glands stimulated intrauterine endometrial stroma)
Marked proliferation: by the end of pregnancy -
occupy up to one half of the entire cervical mass 4. Increased caliber of ovarian veins
Cervical extension, or eversion, of the from 0.9 to 2.6 cm at term
proliferating columnar endocervical glands
(tissue tends to be red and velvety and bleeds 5. Pregnancy Luteoma
even with minor trauma) Overexaggeration of the luteinization reaction of
normal ovary
5. Mucoid plug Not true malignancies but are large masses
Produced by endocervical mucosal cells due to o Difficult to differentiate from other ovarian
hypertrophy and hyperplasia of the glands neoplasms
Rich in immunoglobulins and cytokines o May cause maternal virilization, female fetus
Acts as a barrier w/in cervix, prevents bacteria USUALLY not affected (due to protective
from entering uterine cavity through the vagina role of trophoblast
o During contraction, mucoid plug is converting androstenedione to estrogen)
extruded = bloody show Self-limited; resolves after delivery

Stimuli for these changes: estrogen &


progesterone 6. Hyperreactio Luteinalis (Theca-
* anti-bacterial and immunologic Lutein Cyst)
- benign ovarian lesion; usually bilateral cystic
6. Changes in cervical mucus (as ovaries are moderately to massively
seen on a glass slide) enlarged
Beading/crystallization (progesterone induced) Due to exaggerated physiologic follicle stimulation
Ferning (arborisation of crystals) Tumor related to an increase in hCG
Result of amniotic fluid leakage - May also be seen in hyperthyroid patients due to
structurally similarity between TSH
and hCG
2
MATERNAL PHYSIOLOGY: PDV
Usually found in large placenta, multiple fetuses and Exchange happens in terminal villi composed of
anti-D alloimmunization layers of syncy., basal lamina, and endothelial cells
May also cause virilization during pregnancy (due to Late pregnancy, diffusion barrier thins 5 mm surface
high testosterone and androstenedione) 12m2
- Sx: temporal balding, hirsutism, Large amounts of Progesterone, Estrogen, hCG,
clitoromegaly hPL and other secretory products are released by
Self-limited; resolution follows delivery syncytiotrophoblast directly into intervillous space
Maternal circulation
D. Fallopian Tubes Placental hormones largely responsible maternal
physiology
1. Little hypertrophy of muscle layer
2. Flattened epithelium BREASTS
3. FT torsion (may result from In preparation for lactation
increasing size of the uterus in the presence of 1. Breast tenderness & paresthesias
ovarian cyts) (early pregnancy) ***

E. Vagina and Perineum After 2nd month

1. Increased vascularity and 2. Enlargement of mammary glands


hyperemia o At first missed menstrual period
In skin & muscles of perineum and vulva o Due to hypertrophy and hyperplasia of glands in
preparation for lactation
2. + C ad c o Increase in fatty tissue
Violet color of the vagina
Due to increased vascularity 3. Delicate veins become visible
4. Nipples: deeply pigmented, enlarge,
3. Vaginal changes that prepare for more erectile
delivery:
Increase in mucosal thickness After first few months
Loosening of connective tissue 5. Expression of colostrums (end of
Hypertrophy of smooth muscle cells first trimester)
Hypertrophy of the papillae of vaginal epithelium Thick yellowish fluid expressed from nipples after
fine, hobnailed appearance gentle massage
1st evidence of secretory activity
4. Increased volume of cervical
secretions within vagina 6. Broader and more deeply pigmented
Thick white discharge areola
pH 3.5-6 due to increased production of lactic acid happens after # 5
from glycogen in epithelium (by forms a mottled secondary areola
Lactobacillus acidophilus
Bartholin gland cyst 1-cm common 7. Hypertrophic glands of Montgomery
Sebaceous glands found scattered through the
PLACENTA (APMC only not discussed) areola

Anchor developing fetus to the uterine wall **Extensive increase in breast size may also cause
exchange of nutrients and fetal wastes striations on the skin of the breast similar to the
Direct homeostatic adj. to meet changing fetal needs striations in the abdomen
by secreting hormones and others into mat. **No relation between prepregnant breast size and
Circulation volume of milk production
Disc shaped measures 22cm in diameter and
2.5cm thick at the end of pregnancy SKIN
Chorionic plate: facing fetus
Penetrated at center AVA -> tree-like vili which is A. Blood Flow
rooted at chorionic plate and extended to basal plate 1. Increased metabolism in pregnancy =
(maternal decidual + extravillious excess heat production = increased cutaneous
syncytiotrophoblast) blood f low to release heat
60 -70 villuos trees 100,000 intermediate and
terminal villi B. Abdominal Wall
1. Striae gravidarum
Length 90 km with finer branches of arterioles and
venules terminate grape-like (sinusoidal dilated Stretch marks
capillaries) Begins at midpregnancy
Entire villous tree ensheathed in a continuous layer Reddish, glistening, silvery lines that represent the
of syncytiotrophoblast overlay a discon. layer of cicatrices from previous striae, slightly
cytotrophoblastic cells. depressed streaks commonly found
Chorionic plate fused at edges with basal plate to in abdominal skin and some in the breasts & thighs
form hollow cavity Risk factors: weight gain during pregnancy, younger
Intervillous spaceperfused with Maternal blood maternal age, family history
enter spiral erteries branch off the radial arteries * change in the Biometrical components of the skin
in the myometrium and exits by way of * disruption of the skin weight
uteroplacental veins 2. Diastasis recti
o Separation of rectus muscles at
the midline
3
MATERNAL PHYSIOLOGY: PDV
o Due to inability of the abdominal
wall to withstand the tension

C. Hyperpigmentation (may also be caused by


oral contraceptives)
1. Linea nigra
Midline of the abdominal skin (linea alba) is
hyperpigmented to a brownish- black color
2. Cholasma or melasma gravidarum
Mask of pregnancy
Irregular brownish patches of varying size on the
face and neck A. Weight Gain
caused by melanocyte-stimulating hormone 2nd o Most of the weight is contributed by
month until term the uterus and its contents plus
hypervolemia and extravascular
3. Pigmentation of areola & genital skin fluid increase.
Disappear/regress after delivery o Increase in cellular water and
deposition of new fat and protein
Stimuli: (maternal reserves)
Increased melanocyte-stimulating hormone
(elevated at 2nd month of pregnancy until o increase BMR
term) o Weight gain: (average: 12.5kg or
Estrogen & progesterone (melanocyte stimulating 27.5lb) more than normal DM
effect) o Causes:
Uterus + contents
D. Vascular Changes Breasts
1. Angiomas (vascular spiders) Increase in blood volume
Minute, red elevations on the skin (face, neck Increase in extravascular
upper chest, arms) with radicles branching ECF
out from a central lesion Increase in cellular water
Aka nevus, angioma or telangiectasis and fat/protein deposition =
* can be seen in liver diseases maternal reserves
o 1st trimester: 2 lbs gain (5lbs wt loss
2. Palmar Erythema = normal)
Due to hyperestrogenemia o 2nd trimester: 11 lbs (1-2lbs/wk)
rd
Disappear after pregnancy o 3 Trimester 11 lbs
3. Increased cutaneous blood flow
serves to dissipate excess heat generated by B. Metabolism Changes
increased metabolism
1. Water
* important to do differential diagnosis because
these changes can still happen in other serious o There is increased water retention,
diseases mostly attributed to a fall in plasma
ex. Cervix discoloration OC pills osmolality induced by resetting
threshold for thirst and vasopressin
action.
o Water content (of the fetus)
at term: 3.5 L. + 3.0 L more as a
result of increased maternal blood
METABOLIC CHANGES volume.
o The minimum amount of extra water
the woman can hold: 6.5 L.
o Increase of venous pressure below
the level of the uterus as a
consequence of partial occlusion of
the vena cava produces
accumulation of fluid in the ankles
and legs.

2. Proteins

o The fetus and placenta = 4 kg with a


protein content of 500 g.
o Another 500 g. is added to the
uterus as contractile proteins, to the
breasts as found in the glands, and
to the maternal blood as hemoglobin
and plasma proteins.
o Daily supplement of protein is
needed growth or fetus and
mother
o Higher concentrations in fetal
4
MATERNAL PHYSIOLOGY: PDV
o Largely regulated by the placenta improve across pregnancy
not only concentrates AA into the because of the increased
fetal circulation +protein synthesis, HDL-cholesterol
oxidation, and transamination of concentration likely inhibit
some nonessential AA LDL oxidation thus
protecting the endothelium
3. Carbohydrates Increased CV disease may
be related to other factors
o Normal pregnancy is characterized other than
by mild fasting hypoglycemia, hypercholesterolemia
postprandial hyperglycemia and o LDL-C peaks at about 36 weeks
hyperinsulinemia. (consequence of hepatic effects of
Increased insulin response to estradiol)
glucose. o HDL-C peaks at 25 weeks but
nd
Increased peripheral uptake of decreases by the 32 week.
glucose. o Lactation speeds decrease change
Suppressed glucagons in levels
o Increased concentration of free fatty nd
acids from lipolysis (enhanced by 5. Increased leptin levels (peak in 2
hPL) facilitate increased tissue trimester; plateau in term 2 to 4 times
resistance to insulin. higher in nonpregnant)
o Insulin sensitivity in late normal o Secreted by adipose tissue

pregnancy 45 to 70% lower than o Regulate body fat and energy
non pregnant expenditure
o Increase is partially
o Nitrogen Balance due to increased weight gain
Increased with gestational age o Deficiency:
Urinary excretion of 3- 1. Anovulation and infertility
methylhistidine did not change 2. Increase causes
no maternal muscle preecplampsia and GDM
breakdown o Leptin is also produced by placenta
Turnover rate of nonessential o May play a role in regulation of
serine decreases across growth
gestation o Placental weight is correlated with
o Accelerated Starvation leptin levels (UC blood
Post-prandial: elevated and measurement)
sustained glucose to a fasting o Leptin and adiponectin cytokine
state: decreased plasma involved in fetal growth
glucose and some AA
Plasma concentrations of FFA, 6. Increase ghrelin levels (Increase in
Triglycerides and cholesterol mid-pregnancy and decrease until term)
are higher o Produced by the placenta
Pregnancy-induced switch in o Role in fetal growth and cell
fuels from glucose to lipids proliferation
Prolonged fasting ketonemia o Regulates GH secretion

4. Fats
o Concentrations of all lipids are 7. Electrolyte and mineral Metabolism
usually increased in pregnancy
(includes lipoproteins and NORMAL: 1000 mEq of Na and 300 mEq of K
apolipoproteins)
o Increased insulin-resistance and o Large accumulations of sodium and
estrogen stimulation during potassium that decreases in the
pregnancy Maternal plasma.
Hyperlipidemia o Increase in their glomerular filtration,
st nd
o 1 and 2 trimester maternal fat but the excretion of these
accumulation increase in lipid electrolytes are unchanged as a
synthesis and food intake result of increased tubular
rd
o 3 trimester Fat storage decrease: resorption
enhanced lipolytic activity o Serum concentrations are
(increased hPL effect) and decreased slightly because of
decreased lipoprotein lipase activity expanded plasma volume
reduces Triglyceride uptake into o Progesterone counters
adipose tissue aldo erone effec on na ri e i
favors maternal use of lipids and kaliuresis.
o Hyperlipidemia o CALCIUM - plasma levels decline,
follows lowered plasma Albumin
Triacylglycerol and cholesterol
concentrations decrease in the
levels in VLDL, LDL and HDL
rd amt. of circulating protein-bound
increased during 3 trimester
nonionized calcium
Associated with endothelial
Serum ionized Ca levels
dysfunction
remain unchanged
Endothelium-dependent
Fetus demands on maternal
vasodilation responses
5
MATERNAL PHYSIOLOGY: PDV
Ca: doubing of maternal - Expansion is both plasma > erythrocytes (average
intestinal Ca absorption by 450mL)
1,25-dihydroxyvitamin D3 - Moderate erythroid hyperplasia in bone marrow
o MAGNESIUM: plasma levels - Reticulocyte count is slightly elevated
decline. Lowered plasma proteins
cause little of the minerals that are - Slightly elevated reticulocyte count during
bound. pregnancy. Followed by increased levels of
rd
o PHOSPHATE: within non pregnant erythropoietin peaking at 3 trimester and
range. The renal threshold for corresponds to maximal erythrocyte production.
inorganic phosphates excretion is - delicate/may cause heart failure: Heart may not be
elevated in pregnancy due to able to handle BV at times of greater increase in
increased calcitonin. BV/CO
o IODINE: Increase during normal (immediately after delivery) maximal Work Load
pregnancy Contraction increase pump
Maternal Thyroxine (T4) Postpartum extra fluid in the circulatory system
production increases to comes back into circulatory system; increase in
maintain maternal venous return and decrease in intraabdominal
euthyroidism and to transfer pressure
thyroid hormones to the First week puerperium: mobilization of fluid from
fetus early in the gestation interstitium to vascular compartment; increases
before the fetal thyroid blood flow back to heart
functioning
Fetal Thyroid hormone B. Hemoglobin and Hematocrit
increases during the 2nd
half of pregnancy -- this o Decreases slightly during pregnancy.
contributes to increased o Due to increase in plasma voume (dilution
maternal iodine effect) physiologic anemia
requirements because o Whole blood viscosity also goes down with it
iodide crosses the placenta o Hgb average: 12.5 mg/dL; can go as low as
Primary route of Iodine 11.0 mg/dL (if low: iron deficiency rather than
extractions is through the pregnancy hypervolemia)
kidney in the early part of
pregnancy the iodide C. Iron
glomerular filtration rate 1. Storage Iron
increases by 30 to 50 o Total iron content in women: 2-2.5 g.
percent o Hemoglobin and Myoglobin
Placenta has the ability to o Iron stores 300mg
store Iodine
o IRON: pregnancy induces minimal
change in iron metabolism. 2. Iron Requirements

o The iron requirement for normal


HEMATOLOGIC CHANGES pregnancy is 1000 mg.
o 300 mg.: actively transferred to the fetus
A. Blood Volume and placenta
o 200 mg.: lost through various routes (GI)
- Maternal blood volume increases during pregnancy o As blood erythrocyte volume expand,
(to about 40-45% more than the usual). Increase in more iron is needed, Thus another 500
both. mg is required, because 1 mL. RBC
- A fetus is not essential in the production of contains 1.1 mg at an expanded volume
hypervolemia, a hyadatidiform cyst also produces of 450 mL.
increased ECV. o The requirement becomes increased,
especially during the second half. It now
Functions: become about 6-7 g a day from
Meet the demands of the enlarged uterus exogenous sources.
Oxygen delivery
Protect both mother and fetus from the 3. Pueperium
deleterious effects of impaired venous return
supine/erect o 500 to 600mL of predelivery whole blood
Safegaurd the mother against adverse are lost with vaginal delivery of a single
effects of parturition increase blood loss fetus
Usual blood loss in vaginal o CS or with twins: 1000mL
delivery = 500-600ml o Bleeding after birth: Originating from the
Usual blood loss in caesarean placental implantation site, the placenta
delivery or twins = 1000ml itself, the laceration/episiostomy, and in
the lochia.
st
- Begins to increase during the 1 tri. By 12
menstrual weeks, plasma volume expands by IMMUNOLOGICAL FUNCTIONS
approximately 15%
nd
- Most rapid during the 2 trimester; slower during - Pregnancy reduces the effects of a variety of
rd
3 trimester to plateau during the weeks cellular and humoral mediated immune
re pon e in order o accommoda e he foreign
semiallogenic fetal graft
6
MATERNAL PHYSIOLOGY: PDV
- proinflammatory and antiinflamatory depending o During labor and the early puerperium,
on the stage values may become markedly elevated,
Stages: attaining levels of 25,000/ L
1. Proinflammatory early pregnancy
- during the implantation and NORMAL: 5,000-12,000/µL.
placentation, the blastocyst must break
through the uterine cavity. Trophoblast o 3rd trimester, the percentages of
must replace endothelium and vascular granulocytes and CD8 T lymphocytes
S. muscle of the BV to have good B. are significantly increased, along with a
flow these processes causes concomitant reduction in the
invading, dying and repairing needing an percentages of CD4 T lymphocytes and
inflammatory envi. monocytes

2. Antiinflammatory - Midpregnancy B. Inflammatory Markers


- fetal Gand D anti-inflammatory state o Leukocyte Alkaline Phosphatase (LAP)
is needed is increased beginning quite early in
pregnancy
3. Influx of Immune Cells - Partuition o 1000-fold increased response of C-
- in the myometrium to promote reactive protein (acute-phase serum
recrudescence of an inflammatory reactant) increasing during labor
process o Elevated ESR (Erythrocyte
Sedimentation Rate) due to increased
- Important inflammatory component globulins and fibrinogen
rd
S pre ion of Th and Tc 1 cell hich o 2nd and 3 tri: C3 and C4 are also
decreases secretion of IL-2 interferon-y and TNF-B increased
S pre ed Th1 prerequisite for o Procalcitonin - a normal precursor of
pregnancy continuation calcitonin, increase at the end of the 3rd
- Explains pregnancy related remission of trimester and through the first few
autoimmune disorders (R. arthritis, Multiple postpartum days. Elevated in severe
Sclerosis and Hashimoto thyroiditis - Th1 bacterial infections but remain low in
mediated) Th1 suppression failure causes viral infections and nonspecific
preeclampsia inflammatory disease
preg la ion of Th2 increase IL-4, IL-6
and IL-13 C. Coagulation and Fibrinolysis
Cer ical m c peak Ig A and G o The coagulation cascade is activated
st
IL-B in cervical and vaginal mucus 1 during normal pregnancy.
trimester : 10 fold greater o Increased concentrations of all clotting
* WBC increase indicator in infection (5-10k/mm) factors except XI and XIII
- sometimes goes over 10k o Clotting time no significant change
considering the substantive
physiological increase in plasma volume
in normal pregnancy, such increased
concentrations represent a markedly
augmented production of these
procoagulants
o Progressive increases in the level and
rate of thrombin generation throughout
gestation. These returned to
preconceptional levels by 1 year after
pregnancy
o The percentage in high-molecular
weight fibrinogen is unchanged
contributing to the increased ESR
o Levels of coagulation factors can be
duplicated by the administration of
estrogen plus progestin contraceptive
tablets
o Tissue plasminogen activator (tPA)
converts plasminogen into plasmin,
which causes fibrinolysis and produces
fibrin-degradation products such as D-
dimers decreased fibrinolytic activity

o Platelets:
A. Leukocytes Decreased slightly during pregnancy
o 213,000/ L compared i h
nd
o 2 tri and after - depressed 250,000/ L in nonpregnan con rol
polymorphonuclear leukocyte women partially due to
chemotaxis and adherence functions hemodilutional effects.
Relaxin impairs the Increased platelet consumption,
Neutrophil activation leading to a greater proportion of
o Depressed Leukocyte functions = younger and therefore larger
improvement of some autoimmune platelets
7
MATERNAL PHYSIOLOGY: PDV
Midpregnancy: Increase production o Ventricular function during pregnancy is
of thromboxane A2, which induces normal (as estimated by the Braunwald
platelet aggregation (Because of ventricular function graph)
splenic enlargement) 1. Remodelling (please compare to
o Regulatory Proteins: Williams because it says differently
- natural inhibitors of coagulation: compared to this)
Proteins C and S and antithrombin o Due to high levels of estrogen and
Resistance to Activated C, progesterone
along with a decrease in free o Increase in preload + increased blood
protein S and factor V volume = increase in left atrial diameter
(neutralizing factor Va and VIIIa) o Increased aortic distensibility +
Increase in factor VIII decreased VR = decreased afterload
Constant antithrombin o Response to circulating levels of
o Spleen: Estrogen and Progesterone
Enlarges by 50% - by the end of o Increase in preload develops in concert
pregnancy with the increment in BV increase in
Cause of this splenomegaly is left atrial diameter
unknown, but it might follow the o 5 months after hypertrophy = mild
increased blood volume and/or residual persists
the hemodynamic changes of
pregnancy B. Cardiac Output
o Arterial blood pressure and vascular
CARDIOVASCULAR SYSTEM resistance are decreased, while ECV,
maternal weight and metabolic rates
o The most important changes in the CVS increase.
is apparent during the first 8 weeks of o Increased in 6 weeks
pregnancy. o 4.8 L/min before fertilization and 7 after
st
o Brachial systolic blood pressure, 1 tri
diastolic blood pressure, and central o Increase in CO accompanied by a
systolic blood pressure are all modest decrease in mean blood
significantly lower 6 to 7 weeks from the pressure - from a decline in diastolic
last menstrual period pressure
th
o CO is increased at 5 week and reflects o CO is HIGHER in the RECUMBENT
a reduced systemic vascular resistance position; because in the supine position,
and an increased HR the uterus impedes cardiac venous
o Increase Resting Pulse (10 bpm) return from the IVC Cardiac filling
- P. Volume expansion and may be reduced
Preload (bet. 10 20 weeks) o CO becomes appreciably greater at the
nd
o Ventricular performance during 2 stage of labor with forceful expulsion.
pregnancy is influenced by the decrease o In multifetal pregnancies, there is also
in the systemic vascular resistance and increase in CO predominantly because
changes in pulsatile arterial flow of greater SV and HR.
o Vascular Resistance: Left atrial diameter and left
Human Vascular endothelial cells ventricular end-diastolic diameter
express estrogen receptors and in are also increased due to
response to Estradiol rapidly augmented preload
increase vasodilating NO and PGI2 The increased heart rate and
inotropic contractility imply that
A. Heart cardiovascular reserve is reduced
o The heart is displaced to the left and
upward as a result of diaphragmatic C. Hemodynamic Function in Late Pregnancy
enlargement
o It is also somewhat rotated on its long o Increases in HR, SV, and CO.(although
axis Apex moved laterally Larger increase in CO LV func as measured by
cardiac silhouette in radiograph Stroke work remains similar to
o Some degree of pericardial effusion nonpregnant)
o Slight deviation of electrical axis to the o Decrease in Systemic vascular and
left (ECG) because of altered position Pulmonary vascular resistance.
but no characteristic changes in ECG o Decrease Colloid Osmotic pressure
o Alterations in cardiac sounds: o No change in pulmonary capillary wedge
Exaggerated splitting of the first pressure, and central venous pressure
nd
heart sound; no change in 2 ;
and a loud easily heard 3
rd D. Circulation and Blood Pressure
sound. o Brachial Artery pressure: Sitting <
Systolic murmur (90%) Lateral recumbent supine
intensified at inspiration; o BP returns to baseline at term
disappears shortly after delivery; o Labor: increase in CO and BP
more volume handled by the o After labor: CO initially increases then
heart. Diastolic murmur decreases within the first hour to reach
cardiac disease baseline 2 weeks postP
o Atrial pressure decreases to a nadir at
Soft Diastolic murmur (10%)
24 -26 weeks and rises after
from the breast vasculature
o Decrease: Diastolic > Systolic
8
MATERNAL PHYSIOLOGY: PDV
o Antecubital venous pressure = - increase (severe preeclamsia)
unchanged due to increased cardiac strain from induced
o Supine: femoral V. pressureincrease afterload
o Venous BF in the legs is retarded except o ANP
when in lateral recumbent - participate in extracellular fluid
o Blood stagnation in the lower extremities expansion and in increased plasma aldosterone
during latter pregnancy - occlusion of
the pelvic veins and inferior vena cava o CNP (C-type NP)
by the enlarged uterus - predominantly noncardiac tissues
o Elevated venous pressure returns to - fetal bone growth regulator
normal when the pregnant woman lies
on her side and immediately after G. Prostaglandin
delivery dependent edema frequently o Increase production of prostaglandin
experienced and to the development of play a central role in regulating vascular
varicose veins in the legs and vulva, as tone, BP and Na balance
well as hemorrhoids + predispose to o Renal Medullary Prostaglandin E2
deep-vein thrombosis. synthesis increases during late
pregnancy and presumed to be
1. Supine Hypotension natriuretic
o Supine Hypotensive Syndrome o PGI2 (prostacyclin)
- compression of great vessels by - prostaglandin of endothelium
uterus increased during late
o Uterine arterial pressure < Brachial pregnancy and regulates BP
artery affects fetal HR pattern and platelet function
Increase Venous return compressed IVC (supine) - Implicated in angiotensin
below the level of the uterus edema and resistance
varicose veins - Ratio with thromboxane in urine
Othrostatic hypotension and blood related to preeclampsia pathogenesis
Increase volume and increased blood vessel and
blood con ainer = pre re no increa ed generall H. Endothelins
low in 2dn trimester o Endothelin-1 is produced in the endothelial
Ex. 80-85 MAP cells and vascular smooth muscle to
Preecplamsia increase in BP regulate vasomotor tone by inducing
vasoconstriction.
E. Renin, Angiotensin II and Plasma Volume o It has been identified in the amnion, amniotic
o RAA axis controls salt and water volume. fluid, deciduas and placental tissue.
All components of the system are o It is stimulated by angiotensin II, Arg-
increased during pregnancy. vassopressin, and thrombin
o Renin is produced by maternal kidney o It can stimulate secretion of ANP,
and placenta aldosterone and catecholamines.
o Increased Renin substrate (angiotensin)
produced in fetal and maternal liver I. Nitric Oxide
o Actual hormones are secreted by the o Potent vasodilator endothelial cells
maternal kidney and the uteroplacental o Imp. Mediators of placental vascular tone
unit; increase attributed to increased and dev.
st
estrogen secretion (most important 1 o Abnormal associated with preeclampsia
trimester)
o Sensitivity to angiotensin II = increased
because of vessel wall refractoriness RESPIRATORY TRACT
(lost after delivery of placenta 15 to 30 Facts:
mins) The diaphragm rises about 4 cm. during
o Prostaglandin vascular response to pregnancy
Angiotensin II Subcostal angle widens as transverse
o Exogenous progesterone does not diameter of the thoracic cage widens by +
restore angiotensin II refractoriness to 2 cm.
women with gestational hypertension, + 6 cm. to the thoracic circumference. But
this can be done with infusion of its this adaptation not sufficient to prevent
major metabolite, 5 - residual volume created by think elevated
dihydroprogesterone diaphragm
Greater diaphragmatic excursion
F. Cardiac Natriuretic Peptides
o ANP and BNP (B-type NP) secrete by
cardiomyoctes in response to stretch
o Unchanged despite increase in plasma
volume
o Provokes Natriuresis, Diuresis and
Vasculer SM relaxation

o BNP and NT pro-BNP (amino-terminal


pro-brain NP)
screening for depressed LV
systolic function (for preggy and non-preggy)

9
MATERNAL PHYSIOLOGY: PDV
o Increased respiratory effort in turn
reduction in PCO2
- induced by Progesterone in a
lesser degree by Estrogen

Proge erone appear o ac cen rall , here i


lowers the threshold and increases the sensitivity of
the chemoreflex response to CO2 increases
Ventilatory drive
* Progesterone is also sensitive to PO2 hypoxic
A. Pulmonary Function state (high altitude/exercise)
1. Functional residual capacity (FRC) decreases * Hyperventilation high circulating concentrations
by approximately 20 to 30 percent or 400 to 700 mL of progesterone
during pregnancy o Respiratory Alkalosis
Composition: - Minute ventilation increases during
a. Expiratory reserve volume which pregnancy
decreases 15 to 20 - To compensate moderate
percent or 200 to 300 mL decrease in plasma bicarbonate from 26 mmol to 22
b. Residual volume which decreases 20 to mmol minimal increase in the pH shifts the
125 percent or 200 to 400 mL. oxygen dissociation curve to the left increases the
FRC and re id al ol me decline d e o diaphragm affinity of maternal hemoglobin for oxygen (Bohr
elevation, and significant reductions are observed by effect)
the sixth month with a progressive decline across - BUT offset because the slight pH
pregnancy increase also stimulates an increase in 2,3-
2. Inspiratory capacity (the maximum volume that diphosphoglycerate in maternal erythrocytes
can be inhaled from FRC) shifts the curve back to the right reduced Pco2
- increases by 5 to 10 percent or 200 to 250 mL from maternal hyperventilation aids carbon dioxide
during pregnancy (waste) transfer from the fetus to the mother while
3. Total lung capacity (the combination of FRC and also aiding oxygen release to the fetus
inspiratory capacity)
- unchanged or decreases by less than 5 percent at * Gas exchange across the placenta similar to gas
term exchange in lungs th arterial supply to the placenta
is low O2 high CO2 after equilibration with maternal
4. Tidal Volume and Resting Minute Ventilation blood umbilical venous blood has taken on O2 and
Increase (but RR unchanged) delivered CO2
o Increase in RMV * Difference bet. Uterine and umbilical venous blood
a. Stimulated by progesterone is due to the high rate of O2 extraction and CO2
b. Low expiratory reserve volume production by the syncytiotrophoblast and by the
c. Compensated respiratory alkalosis relatively large areas of the placenta that are
5. Peak expiratory flow rates unavailable for exchange.
- increase progressively * Despite low PO2and the metabolic activity of
6. Lung compliance syncytio., the O2 content in umbilical venous blood
- unaffected is the similar to maternal arterial blood because of
7. Airway conductance higher affinity of hemoglobin in fetal blood
- increased and total pulmonary resistance reduced * PCO2, Bicarb and pH are all the same in umbilical
- possibly as a result of progesterone venous blood
8. Maximum breathing capacity and forced or Although the CO is increased, the lowered Hgb
timed vital capacity content of blood accounts for poor oxygen delivery =
- not altered appreciably PHYSIOLOGIC ANEMIA OF PREGNANCY (lowered
9. Critical closing volume atrial oxygen tension)
- (the lung volume at which airways in the dependent
parts of the lung begin to close during expiration) URINARY SYSTEM
unclear if it is higher in pregnancy
10. Total pulmonary resistance A. Kidneys
- reduced o The size of the kidney increases slightly
- Due to progesterone during pregnancy. (About 1.5 cm.
- Hormones of pregnancy will relax airway longer than normal)
smooth muscle = increase functional dead space = o Afferent and efferent and glomerular
decrease airway resistance arterioles are major sites of resistance in
blood flow.
B. Oxygen Delivery
1. Maternal hemoglobin 1. GFR and RBF is INCREASED.
nd
- (in increased pH) has greater affinity for oxygen - Where GFR can rise up to 25% 2
thus increasing total oxygen-carrying capacity. week after conception and 50% by the
nd
2. Maternal atriovenous oxygen difference beginning of the 2 trimester
- decreased. Effects for hyperfiltration:
1. Hypervolemia-induced
C. Acid Base Balance hemodilution
o Physiologic Dyspnea - lowers the protein
- Increased TV that lowers the blood concentration and oncotic pressure of
PCO2 slightly plasma entering the glomerular
microcirculation
2. Increase in Renal Plasma Flow
10
MATERNAL PHYSIOLOGY: PDV
- 80% increase before the Failure of a pregnant woman to excrete
st
end of the 1 trimester concentrated urine after 18 hours of
Ele a ed GFR per i n il erm fluid restriction DOES NOT
increase urination NECESSARILY signify renal damage
- Nonselective and selective inhibition of
NO synthase isoforms attenuate the 3. Urinalysis
renal hemodynamic changes o Glucosuria in pregnancy is not
immediately abnormal; increased
GFR + low reabsorptive capacity of
the tubules account for glucose in
the urine.
o Hematuria difficult labor and
delivery trauma to the bladder
and urethra
o Proteinuria is not always evident
- Excretion rate of more than 150
mg/day aforementioned hyperfiltration
and possible reduction of tubular
reabsorption, significant proteinuria
2. Marked GFR during puerperium during pregnancy is usually defined as a
st
1 post partum day from the reduced protein excretion rate of at least 300
glomerular capillary oncotic pressure mg/day
- Measuring Urine Protein:
3. Reversal of the gestational hypervolemia 1. Dipstick BUT renal
and hemodilution concentration and dilution of urine is not
- still evident on the first postpartum day, accounted for
eventuates by the second week postpartum 2. 24- hour affected by urinary
tract dilatation errors related to both
4. Relaxin retention and timing (remaining urine may
- mediating both increased GFR have formed hours before collection). Advice
and renal blood flow pt. hydration and lateral recumbent for 45
- increases endothelin and nitric to 60 minutes. Discard first
oxide production in the renal circulation renal collection. Final hour place in recumbent
vasodilation and decreased renal afferent and again at the end of this hour final
efferent arteriolar resistance increase in renal collected urine is incorporated into
blood flow and GFR. the total collected volume.
- increase vascular gelatinase 3. CHO/ Crea ration promising
activity renal vasodilation, glomerular approach data can be obtained quickly
hyperfiltration, and reduced myogenic reactivity and collection errors avoided. BUT CHO per
of small renal arteries unit of Crea excreted ir not constant
- enchances NO production by an
C. Ureter (HYDRONEPHROSIS / HYDROURETER)
endothelin B receptor-mediated mechanism
o With the uterus rising out the pelvis, the
5. Urinary flow and sodium excretion ureters may be compressed laterally
- Late in pregnancy average less than half displaces and compresses at the pelvic brim
the excretion rate in the supine position than in o There is unequal degree of dilatation
the recumbent position. because of the compression on the:
- Increased excretion of nutrients via the LEFT: Sigmoid colon
urine. Such nutrients include amino acids and RIGHT: Greater compression by
water-soluble dextrorotation of the uterus + R. ovarian
vitamins. Vein complex is dilated and lies
obliquely over the R. ureter
B. Renal Function Test o Can also be caused by elevated
Progesterone levels
1. The plasma concentration of both urea o Ureteral elongation accompanies distention
and creatinine (0.7 to 0.5 mg/dL) frequently thrown into curves especially
- often decrease in pregnancy the smaller one which become angulated
attributed to the increased GFR. Much is lost o Estrogen: growth promoting
in the urine. o Progesterone: generalized atony of the UT
musculature slowing musculature
2. Creatinine Clearance o Hypertrophy in ureter
- Increased 30% (Nonpregnant: 100 115 o Return dapat within 6 weeks postpartum
mL/min)
D. Bladder
- Decreased serum level
- very useful test to estimate renal function. o Few before 12 weeks
o Increased uterine size, the hyperemia that
o During the day, women tend to affects all pelvic organs,
accumulate the water in the o Hyperplasia of bladder muscle and
form of dependent edema. connective tissues elevate the trigone
o During the night at recumbent thickening of its posterior, or intraureteric,
mobilize fluid with diuresis margin marked deepening and widening
URINE IS DILUTE of the trigone.

11
MATERNAL PHYSIOLOGY: PDV
o There are no mucosal changes other than o During labor, however, and especially after
an increase in the size and tortuosity of its administration of analgesic agents, gastric
blood vessels emptying time may be appreciably
o Bladder pressure from 8 to 20 cm H2O and prolonged = Do not feed
to compensate for reduced bladder capacity o Danger of Gen. Anesthesia regurgitation
absolute and functional urethral length and aspiration of food-laden or highly acidic
increase (6.7 and 4.8 mm respectively) gastric contents
o Increase pressure (70 93 cmH2O) o Decreased motility = prolonged transit time
Pressure effect pushed by the uterus = better ferrous & Ca & H2O absorption
polyuria o Can cause increased
Happens because: flatulence/constipation/ bloated
st
1. 1 trimester enlarged uterus o Mechanical displacement on rectosigmoid
rd
2. 3 tri pushing of the presenting area = constipation and hemorrhoid
part/fetus formation
o Toward the end of pregnancy
-(particularly in nulliparas in whom the F. Liver
presenting part often engages before labor) o Does not enlarge
the entire base of the bladder is pushed o Increase in hepatic arterial and portal
forward and upward the normal convex venous blood flow
surface into a concavity o Total ALP activity can double - the increase
As a result, difficulties in diagnostic and is contributed by the heat-stable placental
therapeutic procedures are greatly ALP isoenzyme, Serum Aspartate
increased. transaminase (AST), alanine transaminase
- pressure from the presenting part impairs (ALT) and y-glutamyl transpeptidase (GGT)
blood and lymph drainage from the bladder and bilirubin levels are usually lower in
base, often rendering the area edematous, pregnancy
easily traumatized, and possibly more o Serum albumin concentration decreases. By
susceptible to infection + marked late pregnancy, albumin concentrations may
hypervascularity be near 3.0 g/dL (approximately 4.3 g/dL in
o Stasis UTI/pyelonephritis nonpregnant women)
o Total body albumin levels are increased,
Gastrointestinal Tract o However, because of pregnancy-associated
increased plasma volume. Serum globulin
A. Pregnancy Gingivitis levels are also slightly higher
o Gums are hyperemic and softened and may o Leucine aminopeptidase (proteolytic liver
bleed when brushing enzyme) whose serum levels may be
o Subsides postpartum increased with liver disease. Its activity is
o Due to increased Estrogen markedly elevated in pregnant women. The
o ep li gra idar m pyogenic granuloma; increase, however, results from a
focal and highly vascular swelling of the pregnancy-specific enzyme(s) with distinct
gums; spongy with raised nodule, ptyalism substrate specificities
and excessive salivation o Pregnancy-induced aminopeptidase has
oxytocinase and vasopressinase activity that
B. Stomach and intestines occasionally causes transient diabetes
o Displaced by the enlarging uterus insipidus
o Vomiting
o Appendix, for instance, is usually displaced G. Gallbladder
upward and somewhat laterally as the o Reduce Gallbladder contractility
uterus enlarges. At times, it may reach the increased residual volume
right flank o Progesterone potentially impairs gallbladder
o Progesterone decreased motility contraction by inhibiting cholecystokinin-
o Estrogen decreased Hydrochloric acid mediated smooth muscle stimulation, which
o Slow emptying increase SI absorption is the primary regulator of gallbladder
(ferrous and calcium) and LI absorption of contraction.
water increased constipation o Impaired emptying, subsequent stasis, and
an increased bile cholesterol saturation
C. Pyrosis (heartburn) increased prevalence of cholesterol
o Common during pregnancy and is most gallstones in multiparas
likely caused by reflux of acidic secretions o cause intrahepatic cholestasis and pruritus
into the lower esophagus gravidarum from retained bile salts
o Although the altered stomach position o Intrahepatic cholestasis has been linked to
probably contributes to its frequency, high circulating levels of estrogen, which
o Lower esophageal sphincter tone also is inhibit intraductal bile acid transport
decreased (due to progesterone) Increased progesterone levels and genetic
o Intraesophageal pressures are lower and factors have been implicated in the
intragastric pressures higher in pregnant pathogenesis
women. At the same time, esophageal
peristalsis has lower wave speed and lower ENDOCRINE SYSTEM
amplitude A. Pituitary glands
o Enlarges during pregnancy (138%)
D. Gastric emptying time o Size can compress the optic chiasma to
o Unchanged during each trimester reduce visual fields but impaired vision is
rare
12
MATERNAL PHYSIOLOGY: PDV
o Due to Estrogen-stimulated hypertrophy and o Promotes mammary alveolar cell RNA
hyperplasia of the lactotrophs sysnthesis, galactopoiesis and production of
o Maternal serum prolactin levels parallel the casein, lactalbumin, lactose and lipids The
increasing size amniotic fluid also contains appreciable
o Gonadotrophs decline in number, and amounts of prolactin, probably secreted by
corticotrophs and thyrotrophs remain the decidua
constant. o Impairs water transfer from the fetus into the
o Somatotrophs are generally suppressed due maternal compartment, thus preventing fetal
to negative feedback by the placental dehydration
production of growth hormone.
o Involutes rapidly thereafter and reaches E. Oxytocin and ADH
normal size by 6 months postpartum o Secreted from the posterior pituitary.
o Pituitary gland not necessarily essential o Levels of antidiuretic hormone, also called
because women without it can still take vasopressin, do not change
supplements and have a normal pregnancy
o Posterior lobe increase in oxytocin Please note: Further discussion daw nasa
production especially nearing term Chapters 21 (p. 426) and 36 (p. 672) Nakakalula
o Increase vascularity is not much na kasi idagdag isang chapter nanaman yun.
enlargement Ischemia bec. of
hemorrhage hypopituiotarism/ knock out F. Thyroid
st
blood supply panpi i ari m (Sheehan ) o 1 tri - increase of thyroxine- binding globin
(TBG) in response to Estrogen
B. Growth Hormone o Thyroidal activating substances are also
st
o At the 1 trimester, from the maternal made in the placenta.
pituitary gland o The thyroid is controlled by both thyrotrophin
o GH in the serum and AF are at non- and hCG during normal and abnormal
pregnant values (0.5 to 7.5 ng/mL). pregnancies
o 8 weeks secreted by the placenta is o Increase clearance of iodide and losses to
detectable the fetus during gestation, may result to a
o 17 weeks placenta is the source of GH iodine deficiency state.
th
o Serum values of GH increase by the 10 o Hyperplasia of the glandular tissue +
st
week and plateau after 28 weeks. (from 10 vascuarity. (volume increase 12 mL 1 tri to
to 14 ng/mL) 15 mL at del) inversely proportional to serum
th th
o GH in the AF peaks by the 14 15 week, thyrotropin
th
and reach baseline by the 36 week. o Increased hCG production indeed causes
activation of the thyroid.
C. Placental growth hormone o Minimal transfer of thyroxine from mother to
o (differs from pituitary growth hormone by 13 child somewhere in advanced age.
amino acid residues) o Thyrotropin-releasing hormone (TRH) is
o secreted by syncytiotrophoblast in a secreted by the hypothalamus stimulates
nonpulsatile fashion thyrotrope cells of the anterior pituitary
o to have some influence on fetal growth as release thyroid-stimulating hormone (TSH)
well as preeclampsia development or thyrotropin.
o major determinant of maternal insulin o TRH levels are not increased during normal
resistance after midpregnancy. pregnancy. However, this neurotransmitter
o Maternal serum levels correlate positively does cross the placenta and may serve to
with birthweight but negatively with fetal- stimulate the fetal pituitary to secrete
growth restriction and uterine artery thyrotropin
resistance fetal growth still progresses in o Pregnancy causes TG to increase
the complete absence of this hormone. THproduction = mild hyperthyroid state
o Although not absolutely essential, the o Full blown lang sa DM
hormone may act in concert with placental
lactogen and somatolactogens to regulate
fetal growth

D. Prolactin
o Prolactin has a 10-fold increase in the
maternal blood (150 ng/mL)
o After delivery, there is a paradoxical
decrease even in breastfeeding women
o Pulsatile burst of prolactin in response to
sucking.
o Estrogen can increase the activation of the
lactotrophs in the pituitary
o TRH increases prolactin
o Serotonin also increases prolactin secretion.
o Its principal function is Lactation
o Early in pregnancy initiate DNA synthesis
and mitosis of glandular epithelial cells and
presecretory alveolar cells of the breast
o Increases prolactin and estrogen receptors
in the cell

13
MATERNAL PHYSIOLOGY: PDV
o Increased production of active vitamin D is
likely due to placental production of either
PTH or a PTH-related protein (PTH-rP)
o Outside pregnancy and lactation, PTH-rP is
usually only detectable in serum of women
with hypercalcemia due to malignancy
o During pregnancy, PTH-rP concentrations
increase significantly - synthesized in both
fetal tissues and maternal breasts
o Intensified levels are attributed to the fact
that maternal calcium stores are low.
o PTH is blocked by estrogen.
o Physiological hyperparathyroidism of
pregnancy.
- Calcium and magnesium increase the
biosynthesis of calcitonin.
- Opposes the action of PTH by lowering
plasma calcium.
- Since pregnancy causes calcium stress,
levels of calcitonin are generally higher.

o Calcitonin
- C cells that secrete calcitonin are derived
Explanation to the figure above: embryologically from the neural crest and are
1. Maternal changes include a marked and early located predominantly in the perifollicular areas of
increase in hepatic production of thyroxine-binding the thyroid gland
globulin (TBG) and placental production of human - Increased by gastric hormones (Gastrin,
chorionic gonadotropin (hCG). Pengastrin, Glucagon and pancreozymin + food
2. Increased thyroxine-binding globulin increases ingestion)
serum thyroxine (T4) concentrations.
3. hCG has thyrotropin-like activity and stimulates I. Adrenal Glands
maternal free T4 secretion inhibits maternal
secretion of thyrotropin. Except for minimally 1. Cortisol
increased free T4 levels when hCG peaks, these o Considerable increase in cortisol serum
levels are essentially unchanged. concentration but much is bound to
4. Fetal levels of all serum thyroid analytes increase transcortin - cortisol-binding globulin
incrementally across pregnancy. o Adrenal secretion rate of this
5. Fetal triiodothyronine (T3) does not increase until glucocorticoid is probably decreases
late pregnancy o Metabolic clearance rate is lower
because its half-life is doubled
G. Iodine o Estrogen and OC change in level
o Increase
o If low intake low thyroxine and increased o During early pregnancy, the levels of
TSH circulating adrenocorticotropic hormone
o For the fetus, early exposure to thyroid (ACTH), also known as corticotropin, are
hormone is essential for the nervous reduced strikingly As pregnancy
system, and progresses, ACTH and free cortisol
o Iodine deficiency is the most common levels rise equally and strikingly This
preventable cause of impaired neurological apparent paradox is not understood
development completely
Probable reasons:
H. Parathyroid gland and Calcitonin a. some suggest that the
o Acute or chronic decreases in plasma higher free cortisol levels observed in pregnancy
calcium or acute decreases in magnesium result from a
stimulate parathyroid hormone (PTH) re e ing of he ma ernal feedback
release. mechanism to higher levels
o increased calcium and magnesium levels b. might result from tissue
suppress PTH levels. refractoriness to cortisol
o Action on: Bone resorption, intestinal c. antagonistic action of
absorption, and kidney reabsorption is to progesterone on mineralocorticoids Elevated
increase extracellular fluid calcium progesterone
concentrations and decrease phosphate elevated free cortisol homeostasis
levels o Elevated maternal cortisol and
o Fetal skeleton miniralization requires 30g of aldosterone secretion are necessary to
rd
Ca: 3 tri maintain the normal increase in plasma
o Ca absorbed gradually increases volume during late pregnancy
rd
400mg/day at 3 tri
o Ca absorption is mediated by elevated 2. Aldosterone
maternal 1,25 dihydroxyvitamin D -- this o 15 weeks maternal adrenals secrete
occurs despite decreased levels during early increased aldosterone
rd
pregnancy of PTH, which is the normal o 3 tri 1mg/day
stimulus for active Vit D prod. o IF Na intake is restricted Aldosterone is
st
o PTH at 1 tri is low and increases after elevated
14
MATERNAL PHYSIOLOGY: PDV
o Considerable increase in secretion in Pubic joints
order to protect the mother from the o Some symphyseal separation likely
natriuretic effect of preogesterone and accompanies many deliveries, those
ANP. greater than 1 cm may cause significant
o Levels of renin and angiotensin II pain
substrate normally are increased, o Marked lordosis is already characterized
especially during the latter half of by anterior neck flexion and slumping of
pregnancy rise to increased plasma the shoulder girdle which may produce
levels of angiotensin II, which acts on traction to the median and ulnar nerves -
the zona glomerulosa of the maternal --- causes pain and discomfort +
adrenal glands and accounts for the numbness
markedly elevated aldosterone secretion o Normal relaxation of pelvic joints
o Increased aldosterone - protection a. Due to hormones of pregnancy
against the natriuretic effect of b. Allow for expansion during childbirth
progesterone and atrial natriuretic c. Joint laxity and mobility may
peptide cause lower back pain in later
o Modulating trophoblast growth and pregnancy
placental size o Postpartum tailbone pain
- Due to the tailbone being flattened
3. Deoxycorticosterone and p hed o ard a bab head
o Increased rise near 1500 pg/mL at term passes through the pelvic outlet
(>15 fold) during labor and delivery
o Represents increased kidney o Expansion of rib cage
production caused by estrogen o Shoulders become imbalanced
stimulation Muscles that elevate shoulders/
o Increased in fetal > maternal blood internal rotators = tighten
Muscles that depress
4. Androgen shoulders/external rotators =
o Maternal plasma levels of both weaken
androstenedione and testosterone are o Carpal tunnel syndrome compression
increased during pregnancy of the medial nerve as it passes thorugh
o Both androgens are converted to the wrist
estradiol in the placenta, which o Upper extremities
increases their clearance rates Swelling, fluid retension and
o Conversely, increased plasma sex increased BV can compress
hormone-binding globulin in pregnant tissues in extremities especially
women retards testosterone clearance rd
at 3 tri hand pain,
production rates of maternal numbness upon waking up
testosterone and androstenedione
during human pregnancy are increased. EYES
o The source of this increased C19-
steroid production is unknown, but it o Even with pituitary hypertrophy, the optic
likely originates in the ovary. tracts are compressed, but no significant
o Little or no testosterone in maternal change in vision is observed.
plasma enters the fetal circulation as o Decrease in intraocular pressure due to
testosterone. Even when massive increased vitreous outflow.
testosterone levels are found in the o Decreased corneal sensitivity.
circulation of pregnant women, as with Thickness may also change as a
androgen-secreting tumors, consequence of edema.
testosterone concentrations in umbilical o Krukenberg spindles are brownish red
cord blood are likely to be undetectable opacities on the posterior surface of the
This results from the near complete cornea. Hormonal effects are causing
trophoblastic conversion of testosterone this phenomenon.
to 17 -estradiol o Transient but reversible loss of
o Maternal serum and urine levels of accommodation reflex.
dehydroepiandrosterone sulfate are
decreased during normal pregnancy CNS
because of increased metabolic
clearance through extensive maternal o Problems with attention, concentration
hepa ic 16 -hydroxylation and placental rd
and memory significantly at 3 trimester
conversion to estrogen o Difficulty in sleeping, frequent
awakenings, reduce sleep efficiency and
MUSCULO-SKELETAL fewer hours of sleep
o Progressive lordosis is a characteristic o Mean blood flow in the middle and
feature of pregnancy. posterior cerebral arteries decreased
o The body is compensating for the (progressively from 147 and 56 mL/min
anterior position of the enlarging uterus. when nonpregnant to 118 and 44
The lordosis shifts the weight back over mL/min late in pregnancy, respectively)
the lower extremes.
o Increased mobility in the following SLEEP
joints: o 12 eek ge a ion and e ending
Sacroiliac through the first 2 months postpartum,
Sacrococcygeal
15
MATERNAL PHYSIOLOGY: PDV
women have difficulty with going to
sleep, frequent awakenings, fewer hours
of night sleep, and reduced sleep
efficiency
o The frequency and duration of sleep
apnea episodes were reported to be
decreased significantly in pregnant
women compared with those postpartum
o In the supine position, however, average
Pao2 levels were lower
o The greatest disruption of sleep is encountered
postpartum and may contribute to postpartum
blues or to frank depression

16
OB: Fetal Growth and Development
Red mentioned by Dra. Coloma

Yellow highlight lumabas sa samplex

FETAL GROWTH & DEVELOPMENT Death inside uterus with AOG of >20 weeks
Intrauterine Fetal Death (IUFD)
nd
I. Determination of Gestational Age/ Menstrual Still birth 2 half/ later part of pregnancy
Age Placenta previa low lying placenta; causes
It is the time elapsed since the first day of the last bleeding
menstrual period. Embryo term used up to 10 weeks AOG
Time that precedes conception. Fetus AOG >10weeks
Starting time:
- 2 weeks before ovulation and fertilization II. Morphologic Growth
- Nearly 3 weeks before implantation. A. Zygote and Blastocyst Development
AVERAGE: 280 days, or 40 weeks from the first first 2 weeks after ovulation and then fertilization
day of the last menstruation. undergoes implantation 6-7 days after fertilization
corresponds to 9 1/3 months or 10 units 58-cell blastocyst form the 5 embryo producing
containing 28 days each. The unit of 28 days has cells (the inner cell mass)
been referred to as a LUNAR MONTH of pregnancy. Remaining 53 cells form the placenta
trophoblast
Computing for the Age of Gestation (AOG): aka Blastocyst form in which it will implant in
menstrual age the uterine cavity; part of the product will
in weeks, not months become the fetus; part will become the
based on LMP placenta

LMP: April 20-23,2014 B. Embryonic Period


rd
Today: Aug.6, 2014 It begins at the start of the 3 weeks after
Month No. of days for the month ovulation/fertilization which coincides with the time
April 10 the next menstruation would start.
(April has 30 days, since April 20 From cell mass endoderm, ectoderm &
was the LMP, subtract 20 from 30) mesoderm different organ system derived from
May 31 these layers
June 30 primitive chorionic villi form
July 31 lasts 8 weeks
August 6 (since today is Aug 6) organogenesis takes place
Total: 108 days/ 7= 15-16 weeks AOG 2
nd hCG levels may now be measured (+ pregnancy
trimester test)
The embryonic disc, body stalk are defined. The
Computing for Expected Date of Delivery (EDD) chorionic sac is about 1 cm. in diameter.
Naegele R le
1. Subtract 3 months or add 9 months from the Week Development
given date 3 fetal blood vessels in the chorionic
2. Add 7 days to the first day of LMP. villi appear
embryonic disc, body stalk are
LMP: April 20,2014 defined
4 20 chorionic sac: 1cm in diameter
-3 +7 4 cardiovascular system has formed
1 27 Jan 27, 2015 is the EDD & true circulation established within
This is not the exact date, it is only an the embryo and between embryo
approximation and chorionic villi
Delivery must be more or less +/- 2 weeks from the chorionic sac assumes a 2-3
the EDD cm diameter, and the embryo is 4-5
If baby is delivers > 2 weeks the EDD post mm. in length
term baby Middle Primitive heart
of 4 Arm and leg buds, and the amnion
More conveniently, pregnancy is divided into begins to unsheathe the body stalk
three trimesters (each 13-14 weeks long), to be the future umbilical cord
because obstetrical milestones are easier to 6 the embryo is now about 22-24
plot. mm. in length
Trimester dividing pregnancy into 3 groups;
Fingers and toes
used as reference periods
heart completely formed
Sonographic estimate of gestational age is
head is larger than trunk
usually a few days later than that determined by
the last period Arms bend at the elbow
Term pregnancy: 37 weeks and above Upper lip and external ears are
Fetus starts from union of sperm and egg visible
which forms zygote
<20 weeks gestation abortion C. Fetal Period
>20 weeks gestation preterm birth Occur at about 8 weeks after fertilization, or 10
Death inside uterus with AOG of <20weeks weeks after onset of LMP.
miscarriage/ missed abortion Fetus is about 4 cm. long

1
OB: Fetal Growth and Development
Red mentioned by Dra. Coloma

Yellow highlight lumabas sa samplex

period of growth and maturation of the structures many neonate born at this age die because
formed in the embryonic period terminal sac required for gas exchange have not yet
there is increase in size and functionality of the formed; some may survive with intensive neonatal
different systems care
Crown-to-rump length (CRL)/ sitting height CRL= 21cm
more accurate in determining AOG than standing
th
height due to the variability to maintain arms and 26 week
legs extended nociceptors present all over the body
Length is a more accurate criterion of gestational Neural pain system is developed
age than weight
th
28 week
th
11 week CRL= 25 cm
round head= ½ CRL weight= 1100 g.
Eyes closed Thin reddish skin covered in vernix caseosa
external genitalia not distinguishabke (mixture of fatty secretions of fetal sebaceous glands
physiological herniation of the gut: and desquamated epidermal cells)
intestines are located outside abdominal cavity Disappearance of papillary membrane; eyes
within abdominal end partially open
of the umbilical cord peak of isolated eye blinking
neonate born at this age has 90% survival without
th
12 week physical or neurological impairment
Uterus can be palpable above the pubic
nd
symphisis. 32 week
CRL approximately 6-7 cm. CRL= 28 cm
Centers of ossification have appeared in fatal weight= 1800 g
bones skin surface still red and wrinkled
Fingers and toes are well differentiated toenails present
Skin and nails + scattered rudiments of hair testes start to descend
External genitalia are beginning to show evidence birth of this age has good survival rate as long as
of gender given proper care
th
Spontaneous fetal movements; respond to stimuli 36 week
intestines normally located within abdomen CRL= 32cm
Weight= 2500g
th
14 week Because of deposition of subcutaneous fat, the
gender can be determined by inspection body has become more rotund, and wrinkles
of genitalia disappear
birth at this age has excellent chance of survival
th
16 week
th
CRL approximately 12 cm 40 week
Weight of about 110 g Average CRL is 36 cm. and weighs about 3600 g.
Eye movements at 16-18 weeks (coincides with Fetus is fully develop and fully functional
midbrain maturation) testes in scrotum
forearm and lower legs are crossed, fingers chest is prominent, breast protrude
flexed fingernails extend beyond fingertips
bone ossifications allow identification of bones pinae of ear contain cartilage
thru x-ray
III. Fetal-Maternal Communication System
th
20 week PLACENTA
midpoint of pregnancy has 2 parts: Decidua maternal part; chorion
fetus weighs more than 300g frondosum fetal part
Skin becomes less transparent and downy lanugo The vessels that go into the placenta go into
hairs covers the entire body the chorion frondosum it affixes itself into
Scalp hair the endometrium and then it will attach itself
fetal breathing movements become regular to the decidua basalis (invasion)
moves about every minute; active 10-20% of the Gross interface between baby and mother
time At the final structural level, they interface
CRL= 16cm at the intervillous space
Intervillous space-- space between the
th
24 week villi which is a fetal structure but what
fetus now weighs approximately 630 g. goes into this space is maternal blood
Wrinkled skin, with fat deposition. therefore it is where the interface is
Head is still quite large, with eyebrows and The transfer oxygen: MOTHER to FETUS
eyelashes fairly recognizable. The release of CO2 and metabolic wastes:
CANALICULAR PERIOD of the lungs, where the FETUS to MOTHER
bronchi and bronchioles including the alveoli Achieved by way of the nutritive component the
develop placental arm of the FMCS
Fetal lung pneumocytes begin production
of surfactant
2
OB: Fetal Growth and Development
Red mentioned by Dra. Coloma

Yellow highlight lumabas sa samplex

There is no direct communication between the PHYSIOLOGY OF PLACENTAL TRANSFER


fetal and maternal blood because of the lining of the A. Chorionic Villus
chorion frondosum Substances that transfer from mother to fetus
Fetal capillaries contain the fetal blood in should traverse the following:
the chorionic villi 1. Syncitiotrophoblast
Maternal blood remains in the intervillous 2. Villous stroma
spaces 3. Fetal capillary wall
No gross intermingling of the macromolecular
constituents of the two circulations Throughout pregnancy, the syncitiotrphoblast
Eliminates wastes to mother actively or passively permit, facilitate and adjust the
Receives nutrients for the baby total amount of nutrients that enter the fetus.
O2 & CO2 exchange very vital; essential
function of respiration; assumes function of the lungs After midpregnancy, the Langhans cells
Assumes function of the GI (absorption of (cytotrophoblasts), lining the innermost layer of the
nutrients) & GU (elimination of waste) chroionic villi, will decrease and will consist primarily
Blood that passes through the GI and GU of syncitiotrophoblasts.
of the fetus are still deoxygenated blood
so less nutrients are extracted and less B. Regulation of Placental Transfer
wastes eliminated The syncitiotrophoblast is the fetal tissue interface
The placenta also ages efficient only up to a of the whole system.
certain point; eventually degenerates decreased Maternal-facing surface: Complex microvillus
efficiency structure.
>40 weeks AOG adverse effects on Fetal-facing surface: Site of transfer to the
baby due to less efficient placenta intravillous space through which the fetal capillaries
traverse.
THE INTERVILLOUS SPACE
Contains maternal blood (primary unit of Variables of Maternal-Fetal Substance Transfer:
maternal-fetal transfer) 1. Concentration of the substance in the
blood from maternal spiral arteries directly bathes maternal plasma; extent to which is it is
the trophoblasts. protein bound.
To transfer nutrients to the fetus, blood enters the 2. Rate of maternal blood flow
intervillous space and the nutritive substance is 3. Area available for exchange
delivered to the syncitiotrophoblasts. 4. Physical properties of the tissue barrier
The same route is used to eliminate the fetal between the blood in the intervillous space
metabolic wastes. and capillary.
Thus the chorionic villi and intervillous space 5. Biochemical machinery of the palcenta
function as the fetal lung, GIT and kidney. 6. Amount of substance metabolized by the
Although renal function is not yet maximum, fetal placenta
urine comprises a major portion of the amniotic fluid 7. Area of exchange across fetal capillaries
after 16 weeks AOG 8. Concentration of substance in the fetal
blood.
Fetal urination: 9. Specific binding or carrier proteins in the
2.2 mL/h at 22 weeks fetal or maternal circulation.
10 mL/h at 30 weeks 10. Rate of fetal blood flow through the villus
50 mL/h at term capillaries.
The residual volume of the placenta at term is
about 140 mL. This increases by twice before C. Mechanism of Transfer
dlivery. * study what substances are transferred via simple
Uteroplacental blood flow is estimated 700-900 diffusion and facilitated diffusion
mL/min near term. Simple Diffusion occurs among substances with
The blood pressure of the intervillous space is molecular mass <500Da (These include oxygen,
less than uterine artery pressure, but greater than CO2, water, most electrolytes and anesthetic gases)
uterine venous pressure. Insulin, steroid hormones and thyroid hormones
Uterine venous pressure is greatly dependent on cross the placenta but at very slow rates.
the relative position of the mother. Large molevular weight substances do not pass
the placenta unless it is mediated by a specific
FETAL CAPILLARIES trophoblast receptor mediated mechanism (IgG)
Contains the fetal blood.
The hydrostatic pressure of the fetal capillaries D. Transfer of Oxygen and Carbon Dioxide
traversing the chorionic villi is NOT different from The transfer of oxygen is blood flow limited
that of the intervillous space. The average oxygen saturation of the intervillous
During normal delivery, the blood pressure of the space is about 65-75% with partial pressures of 30-
fetal capillaries should parallel the pressure of the 35 mmHg.
amniotic fluid and intervillous space. Otherwise, the the placenta is highly permeable to carbon
capillaries would collapse and fetal blood flow would dioxide, which traverses the chorionic villus by
cease. diffusion more rapidly than oxygen
Despite the low PO2 the fetus still survives
because of the following mechanisms:
3
OB: Fetal Growth and Development
Red mentioned by Dra. Coloma

Yellow highlight lumabas sa samplex

1. High cardiac output


2. Increased oxygen carrying capacity of fetal D. Free Fatty Acids and Triacylglycerols.
hemoglobin. newborn fat= 50% of body weight
3. Higher concentration of hemoglobin Neutral fats (triacylglycerols) does not cross the
compared to the adult. placenta. Lipoprotein lipase, present ONLY on the
maternal side, will hydrolyze neutral fats to allow
Near term, the PCO2 in the umbilical arteries glycerol to enter.
averages about 50 mmHg, about 5 mmHg more Glycerol will be acetylated in the fetal liver into
than maternal blood. triglycerides.
Fetal blood has less affinity for CO2 thus favoring LDL can be taken in as an alternate mechanism
release to the maternal blood. to acquire fatty acids and amino acids.
LDL particle taken up by receptor mediated
IV. Fetal Nutrition endocytosis.
During the first 2 months, the embryo consists
mainly of water. As gestation progress, more solids LDL is hydrolyzed by the lysosomal enzymes in
are being added. the syncitium to give rise to:
Because of the small amount of yolk, the embryo- 1. Cholesterol for progesterone synthesis
fetus is dependent on the mother for nutrition during 2. Free amino acids including essential ones.
the first 2 months. 3. Essential fatty acids, such as linoleic acid.
During the first few days after implantation,
blastocyst nutrition is derived from the interstitial fluid E. Amino Acids
of the endometrium and surrounding maternal The placenta can concentrate large amounts of
tissue. amino acids
They are then transferred to the fetal circulation
Three major storage depots from the mother: by simple diffusion
1. Liver AA concentration in umbilical cord plasma >
2. Adipose tissue maternal villous or arterial plasma
3. Muscles transport influenced by gestational age and
Together with the storage-hormone environmental transport such as heat stress,
insulin, they are intimately involved in the hypoxia and under- and over-nutrition, as well as
metabolism of the nutrients absorbed from hormones such as glucocorticoid, GH and leptin
the maternal gut
F. Proteins
A. Glucose in Fetal Growth Proteins transfer is limited. Except for IgG.
Minimize use of the glucose by the mother so that IgG crosses the placenta via endocytosis and
is can be given to the fetus. trophoblast Fc receptors
Glucose is the major nutrient for fetal growth and IgM can be found in the fetal blood only if there is
energy. an infection.
hPL block glucose uptake and use of glucose,
and promotes the mobilization of fatty acids by G. Ions and Trace Metals
maternal tissues. Iodide transport is a carrier-mediated process and
is energy requiring as well.
B. Glucose Transport Zinc is greater in fetal plasma; Copper is greater
GLUT-1 and GLUT-3 primarily facilitate glucose in maternal plasma
uptake by the placenta and are located in the Important copper-requiring enzymes are
plasma membrane of the synctiotrophoblast neccesarry for fetal development
microvilli Deficiency in copper results in inadequate
GLUT-1 expression is essential for collagen cross-linking thus diminishing
decidualization; it increases as pregnancy advances tensile strength in tissues
and is induced by almost all growth factors
Fetal size is not only dependent on age, but also H. Heavy Metals
on efficiency of nutrient transport, availability, and a Achieved using a heavy metal binding protein
number of cofactors. metallothionein-1 which can sequester zinc, copper,
Fetal Macrosomia: Hyperinsulinemia, increased lead and cadmium.
levels of selected growth factors, increased GLUT Cadmium from cigarette smoke is sequestered by
expression in the syncitiotrophoblast. metallothionein that is why it cannot be found in fetal
circulation.
C. Leptin Copper is also sequestered by metallothionein-1.
contributes to angiogenesis, hemopoiesis, That is why it is less in fetal blood.
osteogenesis, pulmonary maturation, and
neruroendocrine, immune and reproductive I. Calcium and Phosphorus
functions. actively transported from mother to fetus
expressed in synctiotrophoblast and fetal vascular calcium: for fetal skeletal mineralization
endothelial cells parathyroid hormone related protein: acts as
rises approximately 34 weeks and are correlated surrogate PTH; expressed in cytotrophoblast
with fetal weight
abnormal levels: preeclampsia and growth
disorders
4
OB: Fetal Growth and Development
Red mentioned by Dra. Coloma

Yellow highlight lumabas sa samplex

J. Vitamins 2. Lesser/posterior fontanel


Concentration of Vitamin A (retinol) is greater in triangular shaped; intersection of the
the fetal blood. sagittal and lambdoid sutures.
Vitamic C is transferred through a carrier 3. Temporal/ Caesarian fontanel
mediated process. junction of the lambdoid and
Vitamin D metabolites are greater in maternal temporal sutures
plasma, hydroxylation takes place in the placenta.
Palpation and localization of these fontanels can
V. FETAL ORGAN SYSTEM DEVELOPMENT determine the position and presentation of the baby.

FETAL HEAD DIAMETERS OF THE NEWBORN HEAD:


Very important in obstetrics, since it is an 1. Occipitofrontal (11.5 cm) From a point just
essential feature in labor above the root of the nose to the most
Adaptation between the fetal head and the prominent part of the occipital bone; seen if
mo her pel ic o le bab head i no fle ed
composed primarily of skull with face occupying a 2. Biparietal (9.5 cm) Greatest transverse
small part diameter of the head; from one parietal bone
to the other.
3. Bitemporal (8 cm) Greatest distance
between two temporal bones.
4. Occipitomental (12.5 cm) Chin to most
prominent portion of the occiput.
5. Suboccipitobregmatic (9.5 m) line drawn
from the middle of large fontanel to the
undersurface of the occipital bone where it
joins the neck; shortest diameter that can
pass through during normal cephalix
delivery; seen when fetal head is completely
flexed
6. Trachelobregmatic (9.5) line drawn from
the bregma to the undersurface of the fetal
mentum/ mandible

Plane of occipitofrontal diameter greatest


circumference of the head; averages 34.5cm
Plane of suboccipitobregmatic diameter
smallest circumference; 32cm

MOLDING: Intrapartum process; Consider the


thin layer of fibrous tissue which allows a degree of
considerable shifting and sliding as the head adapts
to the shape and size of the pelvis.
Fetopelvic disproportion leading indication
for caesarean delivery

FETAL BRAIN
SKULL From midpregnancy onwards, it is possible to
Composed of 2 frontal, 2 parietal, 2 temporal, identify fetal age from characteristic external
upper portion of occipital bone and wings of the appearance
sphenoid Myelination of the ventral roots of the
These bones are separated by sutures. cerebrospinal nerves and brainstem begin at about
1. Frontal suture between 2 frontal bones th
6 month.
2. Sagittal between 2 parietal bones It is completed at birth.
3. 2 coronal between the frontal and The lack of myelin and ossification of the fetal
parietal bones skull allows the physician to see the fetal brain
4. 2 lambdoid between posterior margins throughout gestation.
of the parietal bones and upper margin
of the occipital bone CNS & SPINAL CORD;
In vertex position, all except the temporal Neuronal proliferation and migration proceed
suture can be palpated (useful in monitoring along with gyral growth and maturation
position and presentation of fetal head) Myelination of the ventral roots of the
cerebrospinal nerves and brainstem begins at
When several sutures meet, they form an approximately 6 months but most myelination occurs
irregular space covered in a membrane after birth
the FONTANEL. 20-24 weeks complete neuronal
1. Greater/anterior fontanel (Bregma) proliferation and migration
lozenge- shaped space between the nd
2 half of gestation organizational events
coronal and sagittal structures. proceed with gyral formation and
5
OB: Fetal Growth and Development
Red mentioned by Dra. Coloma

Yellow highlight lumabas sa samplex

proliferation, differentiation, and migration of CARDIOVASCULAR SYSTEM


cellular elements * study fetal blood circulation favorite questions in
The spinal cord extends along the entire length of quizzes
the vertebral column in the embryo, but after it grows blood from mother goes through the placenta and
more slowly goes to the fetus via umbilical cord
By 24 weeks, the spinal cord fetal blood does not need to enter the pulmonary
extends up to S1 vasculature to be oxygenated; most RV output
At birth to L3 bypasses the lungs
Adult to L1 fetal heart chambers work in parallel, not in series
Spinal cordmyelination begins at the midgestation effectively supplies brain and heart more highly
and continues through the first year of life. oxygenated blood
th
8 week: synaptic responses that promote trunk umbilical cord has 3 vessels: 2 arteries and 1 vein
and neck flexion. oxygen and nutrients are delivered from the
AMNIOTIC FLUID placenta by the single umbilical vein
In early pregnancy, it is an ulatrafiltrate of O gena ed blood en er he bab
maternal plasma; usually comes from amniotic umbilicus, then goes up to the liver and
membrane= 50ml umbilical vein is then divided into 2
Consists of ECF that diffuses from the fetal skin umbilical vein is divided into:
nd
reflecting the composition of fetal plasma by the 2 1. Ductus venosus
triemster - Major branch of the umbilical vein
As cornification of the skin begins at about 20 - Transverses the liver to enter/join the
weeks AOG, the fetal urine becomes a major IVC directly
component of amniotic fluid (fetal kidneys start - Carries highly oxygenated blood directly
producing urine at 12 weeks) to the heart
Fetal urine consists of increased quantities of 2. Portal sinus/ portal vein
urea, creatinine and uric acid. It also contains - Carries blood to the hepatic veins
desquamated epithelials, lanugo, vernix and various primarily on the left side of the liver, and
secretions. oxygen is extracted
Amniotic fluid contains glycerophospholipids, Deoxygenated blood from liver flows to
pulmonary cells, desquamated fetal cells, vernix, IVC, which also receives less oxygenated
lanugo and various secretions. blood from the lower body
The volume of amniotic fluid increases with age: ventricles work in parallel
10 mL. per week after 8 weeks well oxygenated blood enters LV, which supplies
60 mL per week after 21 weeks brain and heart
Peaks at 34 weeks less oxygenated blood enters RV, supplies the
Midpregnancy: 400ml rest of the body
peak volume: 1000ml (36-38weeks) well oxygenated blood tends to course along the
>42 weeks AOG= decrease in AFV medial aspect of the IVC
less oxygenated blood flows along the lateral
Functions: vessel wall
1. Cushions the fetus for musculocutaneous Once blood enters the RA, upper interaterial
growth and trauma protection. septum (crista dividens) shunts well
2. Maintains temperature oxygenated blood from the medial side of
3. Minimal nutritive function the IVC through the foramen ovale into the
4. Contains EGF and EGF like growth factors left heart then to the heart and brain
5. Promote growth and differentiation of lungs After extraction of oxygen, the less
and GIT by swallowing and inspiring oxygenated blood then returns to the RA
amniotic fluid. through the SVC
Less oxygenated blood SVC RA & RV
Ultrasound is usually used to measure AF The SVC courses inferiorly and anteriorly as
index it enters the RA, ensuring that less well-
- How to measure: uterus is divided oxygenated blood returning from the brain
into 4 equal quadrants AF is and upper body will be shunted into the RV
measured vertically in single The ostium of the coronary sinus lies just
deepest pocket of each quadrant superior to the tricuspid valve so that less
add the 4 values to ger AFI oxygenated blood from the heart returns to
- Interpretation: the RV
< 5cm oligohydramnios (may indicate fetal lung 90% of less oxygenated blood exiting the
hypoplasia) RV is shunted through the ductus arteriosus
> 24cm polyhydramnios to the descending aorta
AF can also determine maturity of the lung The remaining RV output returns to the
- Movement of AF in and out of fetal placenta through the 2 hypogastric arteries
lungs= increase in
The blood picks up oxygen and nutrients in
glycerophospholipids & pulmonary
the placenta and is recirculated through the
cells
umbilical vein
- Movement is regulated by fetal
breathing movements

6
OB: Fetal Growth and Development
Red mentioned by Dra. Coloma

Yellow highlight lumabas sa samplex

blood in the RV is 15-20% less saturated than inadequate closure there will be a
blood in the LV due to the shunting of less O2 blood problem and must close within 1 year to
to the RV completely close
- Ductus arteriosus functionally closes
STREAMING OF FLOW 10-96 hours after birth; anatomically
Streaming effect high oxygenated blood closes 2-3 weeks after birth; eventually
passes though the medial side and less becomes ligamentum arteriosum
oxygenated blood passes though the lateral - Ductus venosus functionally closes
side and they are all brought together by 10-96 hours afterbirth; anatomically
the IVC to the RA. By the presence of the closes 2-3 weeks after birth
crista dividens, the streaming is maintained (walang sinabi sa Williams tungkol sa closure ng
such that highly oxygenated blood goes ductus arteriosus pero sabi sa group discussion
through the foramen ovale into the LA and pareho lang sila ng ductus venosus)
then to the LV then goes into the 3. Fetal cardiac output is higher
circulation. - To compensate for the high affinity of
The less oxygenated blood goes to the RA oxygen to HgbF
then to the RV then goes to the pulmonary. HgbF tightly binds O2 more so
The lower part of the pulmonary has a lesser O2 is available to the
structure that connects to the descending circulation fetal heart
aorta, which is the ductus arteriosus. compensates by increasing
Through it, the less oxygenated blood goes cardiac output to meet the
to the descending aorta. demand
On the other hand, less oxygenated blood
from the upper part of the body goes into
the SVC, then goes into the RA then to the FETAL CIRCULATION
RV.
The referred blood from the ductus
venosus goes to through the foramen ovale
to the LA. It then goes to the LV and from
this it goes to the ascending aorta which
supply the coronaries of the heart and then
it goes up to the neck to supply the brain
Heart and brain priority; need more highly
oxygenated blood
Upper part of the descending aorta
supplies the lungs and the viscera (since
lungs are not functional, they receive the
less oxygenated blood)

Circulatory Changes at Birth


umbilical vessels, ductus arteriosus, foramen
ovale and ductus venosus normally constrict or
collapse due to shift in pressure
expansion of lungs and closure of ductus
arteriosus blood from RV enters the pulmonary
vasculature to become oxygenated before it returns
to the left heart
ventricles now work in series, not in parallel
more distal portions of the hypogastric arteries
undergo atrophy and obliteration within 3-4 days
after birth and become umbilical ligaments
intraabdominal remnants of umbilical vein form
ligamentum teres
ductus venosus constricts 10-96 hours after birth
and closes by 2-3 weeks formation of ligamentum
venosum
umbilical arteries become umbilical ligaments

Difference from Adult:


1. Functions in parallel (adult heart works in
series)
- There is a pathway for the oxygenated
blood and pathway for deoxygenated
blood
2. Interconnection of fetal circulation is
possible because of the 3 shunts:
- Foramen ovale closes immediately at
birth; in some instances if there is
7
OB: Fetal Growth and Development
Red mentioned by Dra. Coloma

Yellow highlight lumabas sa samplex

fetoplacental blood volume at term= 125ml/ kg of


fetal weight
Hepmoglobin has a timing of production,
depending on the chains that it contains.

Fetal Hemoglobin
YOLK SAC: Gower 1, Gower 2 and Portland
LIVER: HbF (fetal hemoglobin)
hemoglobin produced by fetus
binds more O2 (12-15G/DL)
Lesser binding with 2,3-DPG= higher
affinity of Hgb for O2
Persistence of HgF in newborns of
diabetic women is usually due to hypomethylation of
the q
gene (genes are not turned off continued
production of HgbF)
BONE MARROW: adult-type hemoglobin A
SHUNT 1 Foramen Ovale hemoglobin produced when baby is born
SHUNT 2 Ductus Arteriosus binds 2,3-DPG more avidly than HgbF= lowered
affinity of Hgb for O2
In pregnancy, maternal 2,3-DPG levels
are increased
As term approaches, less HgbF and more
HgbA are produced (at term, 3/4 are Hgb A)

Functional difference between HbA and


HbF. Fetal RBC that contain HbF bind
more oxygen. Because HbA binds more
rapidly with 2-3DPG reducing its capacity
for oxygen binding.
The switch from HbF to HbA is driven by
glucocotricoids.

Coagulation Factors
With the exception of fibrinogen, the fetus
produces adult-type procoagulant, fibrinolytic and
anticoagulant proteins by 12 weeks.
Fetal leves of factor II, VII, IX, X, XI as well as
prokallikerin, protein S & C, antithrombin and
plasminogen are low (low vitamin K dependent
FETAL BLOOD dependent factors)
st
1 site: Yolk sac/ Mesoblastic Period (can be seen in Requirement of Vit K to reduce risk of
ultrasound up to 10 weeks) hemorrhage at birth to prevent hemorrhagic
function: early source of blood cells up to disease of the newborn because coagulation factors
8 weeks decrease further first few days after birth
nd
2 site: Liver/ Hepatic Period Fetal fibrinogen appears 5 weeks. It forms a less
8-32weeks compressible clot and the fibrin monomer has lower
Final site: Bone Marrow/ Myeloid Period degree of aggregation.
32 weeks and above
The fir RBC relea ed in o fe al circ la ion are Fibrin stabilizing factor or factor XIII is decreased
usually macrocytic and nucleated. Occupying a significantly compared to the adults.
volume of 180 fL, but decreases to 105-115 fL at Bleeding is not significant during cordocentesis,
term. because amniotic fluid thromboplastins and factors
Hemoglobin concentration at midpregnancy is in he Whar on jelly facilitate clotting at the
about 12 g/dL, which rises to about 18 g/dL by term. puncture site.
Fetal erythrocytes have shorter life span due to
large size but lengthens to approximately 90 days at Fetal Plasma Proteins
term. 1. Albumin
RBC production is chiefly controlled by fetal 2. LD
erythropoietin 3. AST
It is produced in response to bleeding, labor and 4. GGT
isoimmunization (hypoxic stress) 5. Alanin transferase
Liver is the production site before the renal cells These proteins increase with gestational age.
mature At birth, the concentration of plasma proteins of
After birth, erythropoietin may not be detectable the fetus parallel that of the mother.
for up to 3 months

8
OB: Fetal Growth and Development
Red mentioned by Dra. Coloma

Yellow highlight lumabas sa samplex

IMMUNOGLOBULINS Hydramnios due to accumulation


A. IgG of AF; common if swallowing is
Transport to the fetus begins by 16 weeks. inhibited
A large bulk is acquired by the last 4 weeks of Esophageal atresia most common
pregnancy. anomaly associated with increase
Adult values are not attained until 3 years of age AF
Pass freely from the mother to the fetus (all IgG Average of 200-760 mL/day swallowed AF.
will be coming from the mother) Swallowing also serves to remove insoluble
transfer of IgG from mother to fetus may also be debris that is shed into the amniotic sac and
harmful hemolytic disease of the fetus and sometimes abnormally excreted into it.
newborn results from D-antigen alloimmunization Intrinsic factor is detectable by 11 weeks
(Rh incompatibility) stomach emptying stimulated primarily by volume
Rh (-) mother, Rh (+) baby no effect on first Pepsinogen by 16 weeks
nd
pregnancy; 2 pregnancy is affected because Movement of AF in the GIT stimulates growth and
mother has already developed antibodies development of the alimentary canal
st
Exception: 1 pregnancy may be affected/ Impaired swallowing= impaired growth and
immunoglobulins of the mother will act on the development of GIT
fetus if there will be a break in the barrier of the MECONIUM
fetal and maternal circulation (ex. Tearing of passed after birth and sometimes during labor
the placenta in placenta previa, invasive Consists not only of AF debris, but of products of
procedures done to the mother that may break secretion such as:
the barrier) 1. Glycerophospholipids in the lungs
Prophylactic anti-Rh prophylaxis is given to the 2. Desquamated fetal cells
mother; any bleeding to an Rh (-) mother will 3. Lanugo scalp hair
warrant administration of anti-Rh (Rhogam) 4. Vernix
The dark greenish black appearance is due to
B. IgM biliverdin pigments.
Not transported from mother to fetus. can be passed due to normal bowel peristalsis in
Very little IgM is produced by the fetus, and that mature fetus or from vagal stimulation
which is produced may include an antibody to the Arginine Vasopressin hypoxia stimulated;
maternal T lymphocytes. stimulated colonic smooth muscle to contract,
produced in response to antigenic stimulus resulting to intraamnionic defecation
Increased levels may be found in newborns with Why is there meconium passage?
congenital infections/ TORCH viruses like rubella, - Normal bowel peristalsis (can signify
CMV, or toxoplasmosis maturation of the GI tract)
At around 32 weeks, some
C. IgA fetuses can already produce
Acquired in the colostrums of breast fed infants meconium
Provides adequate mucosal protection from - Hypoxia/fetal distress (stimulates
enteric organisms pituitary gland to produce Arginine
helpful in preventing diarrhea in newborn Vasopressin stimulates contraction of
the smooth muscles of the colon
LYMPHOCYTES meconium passage/ intraamniotic
B lymphocytes appear in the liver by 9 weeks, defecation)
and become apparent in the blood and spleen by 12 LIVER
weeks AOG. serum liver enzyme levels increase with
T lymphocytes in the fetal thymus are released by gestational age
14 weeks AOG. System of conjugation of bilirubin is immature
Lack of responsive B lymphocytes to polyclonal when the fetus is younger.
activators and lack of proliferating T cells result to The small amount conjugated is excreted through
poor response to immunization and bacterial the biliary tract into the intestines and is ultimately
capsular polysaccharides oxidized to biliverdin.
th
12 week: unconjugated bilirubin is excreted into
MONOCYTES the amniotic fluid and transferred across the
are able to process and present antigen when placenta.
tested with maternal antigen-specific T CELLS bidirectional placental transfer excess
unconjugated bilirubin from mother readily passes to
GASTROINTESTINAL TRACT the fetus and the AF; conjugated bilirubin is not
10-12weeks: Swallowing begins; coincident with exchanged to any significant degree between
active peristalsis of the small intestines and ability to mother and fetus
transport glucose actively fetal cholesterol most from hepatic synthesis;
stimuli for swallowing: thirst, gastric emptying, large demand for LDL by fetal adrenal glands
change in AF composition; fetal taste buds may also Glycogen is present ay low levels in the liver. But
play a role as the fetus nears term, the values rise at least two
Volume of swallowed amniotic fluid is smaller to three times than normal adult liver glycogen.
than the total volume, so it doe n r n o . Values fall precipitously after birth.
AF regulation is substantially affected by fetal
swallowing.
9
OB: Fetal Growth and Development
Red mentioned by Dra. Coloma

Yellow highlight lumabas sa samplex

PANCREAS ANATOMICAL MATURATION


Insulin can be identified by 9-10 weeks and Three stages:
detectably in fetal plasma at 12 weeks. 1. Pseudoglandular stage
The fetal pancreas has the ability to respond to - growth of intrasegmental bronchial tree
hyperglycemia by secreting insulin - between 6-16 weeks
Glucagon is apparent by 8 weeks - the lungs appear like glands.
Most pancreatic enzymes are produced by 16 2. Canalicular stage
weeks, and lipases are present as early as 14 - 16-26 weeks
weeks. - bronchial cartilage plates extend
peripherally
URINARY SYSTEM - each terminal bronchiole give rise to
2 primitive urinary system: pronephros and respiratory bronchioles subsequently
mesonephros dividing into multiple saccular ducts
metanephros forms the final kidney 3. Terminal Sac stage
The pronephros involutes by 2 weeks. - begins at 26 weeks
The mesonephros produced urine at 5 weeks will - respiratory bronchioles give rise to
degenerate by 11-12 weeks. primitive alveoli or terminal sac
Between 9-12 weeks, the ureteric buds and - extracellular matrix develop from
nephrogenic blastema unite to form the proximal to distal lung segments
metanephros. - type II cells produce surfactant
th
By the 14 week, the Loop of Henle becomes - most importance stage because alveoli
functional and reabsorption occurs is fully developed and produces
More nephrons are added until 36 weeks, and surfactant
formation will continue until after birth. 4. Alveolar stage:
fetal kidneys produce urine but have limited ability - begins at 32 weeks
to concentrate and modify pH - alveolar epithelial lining thins to improve
The urine is HYPOTONIC to plasma, and has low gas exchange
electrolyte concentration - an extracellular matrix develops from
Less Na, K, Cl proximal to distal lung segments until
More urea, creatinine, uric acid term
Causes AF to also be hypotonic At birth only 15% of the alveoli are formed, this
High renal vascular resistance and low filtration continues until about 8 years of childhood.
fraction If there is an insult before stage 3 occurs,
Receive only 2-4% of CO as compared to the 15- there will be a decreased growth and
18% of the newborn. development of the pulmonary tree. Baby
may survive but will present with pulmonary
KIDNEYS hypoplasia.
Not essential for survival in utero
important in the control of AF volume and LUNGS
composition Air sacs expand no sooner than 28 weeks (before
nephrons are formed until 36 weeks AOG (In 28 weeks, surfactant expand the lungs)
preterm, it continues after birth) full maturation starts at 32 weeks
After 20 weeks, if there is problem with the Morphological or functional immaturity at birth
kidney oligohydramnios because majority of AF Respiratory Distress Syndrome (RDS)
comes from fetal urine complication: factors that accelerate fetal lung maturity:
inadequate AF will result to poorly developed Corticosteroids (Betamethasone,
fetal lungs pulmonary hypoplasia Dexamethasone)
RBF and urine formation is controlled by: - Usually given when preterm baby is
1. RAA System expected to hasten the process of
2. Sympathetic nervous system the maturation of the lungs to
3. Prostaglandins greatly improve respiratory
4. Kallikrein compliance of baby to decrease
5. ANP incidence of RDS
GFR increases with gestational age - Cortisol natural stimulus for lung
< 0.1 mL/min 12 weeks maturation and augmentation of
0.3 mL/min 20 weeks surfactant synthesis
Formation of urine that appear in the bladder - Used in neonatal surfactant therapy
begins by 12 weeks. (prenatal& postnatal) reduced
7-14 mL/day by 18 weeks incidence of RDS
27ml/hr or 650 mL/day at term - Recommended to give steroids at
- Decreased urine formation may 24-34 weeks
result to fetal hypoxic state - Maximum period of effectivity is 26-
32 weeks
PULMONARY SYSTEM - Duration of effect is only 1 weeks (If
The presence of surface-active materials baby is delivered 1 week after
(surfactant) in the AF is indicative of fetal lung administration of surfactant, there is
maturity. no effect)

10
OB: Fetal Growth and Development
Red mentioned by Dra. Coloma

Yellow highlight lumabas sa samplex

- Multiple courses of steroid will have ENDOCRINE GLANDS


detrimental effect A. Pituitary Gland
Indicators of lung maturation: The two components of the hypophysis develop
Examine amniotic fluid separately:
Lecithin/ sphingomyelin ration (L/S ratio)
- 1:1 (34weeks AOG) 1. Anterior pituitary gland/
- 2 ( 36 weeks AOG) ADENOHYPOPHYSIS
- >2 (mature lungs) de elop from oral ec oderm or Ra hke po ch
Phosphatidylglycerol Differentiates into 5 cell types which secrete 6
Foam shake test protein hormones:
- Indirectly measures a. Lactotrophs produce prolactin (PRL)
phosphatidylglycerol b. Somatotrophs produce growth
Lamellar body count hormones (GH)
- Reliable test to asses fetal pulmonary c. Corticotrophs produce corticotropin
maturity (ACTH) (detected as early as 7 weeks)
d. Thyrotrophs produce thyrotropin or
SURFACTANT thyroid stimulating hormone (TSH)
rd
become highly available in the 3 trimester e. Gonadotrophs produce FSH and LH
Keeps the terminal sacs from collapsing because GH, LH and ACTH have been
they must be kept expanded despite the pressure indentified by 13 weeks.
imparted by tissue-to air interface
In utero, there is water to alveoli interface 2. Posterior pituitary gland/
alveoli constantly being bathed in AF NEUROHYPOPHYSIS neuroectoderm
During delivery, when baby takes first breath Well developed by 10-12 weeks
air goes into alveoli and if there is no Oxytocin and arginine vasopressin (AVP)
surfactant, the lungs will collapse
Low surfactant RDS INTERMEDIATE LOBE
surface active material present in mature lungs cells of this structure begin to disappear before
formed by Type II pneumocytes term and are absent from the adult pituitary
Multivesicular bodies that produce lamellar Prod c ion of -MSH and -Endorphins
bodies spread in alveolus and prevent
collapse B. Thyroid
Lamellar bodies where surfactant is It is able to synthesize hormones by 10-12 weeks
formed AOG, and TSH, thyroxine and TBG are
Capacity of fetal lungs to produce demonstrable by 11 weeks.
surfactant establish lung maturity Normal fetal levels of free T3 and T4 and TBG
lack of surfactant RDS increase steadily throughout pregnancy.
Composition: By 36 weeks:
Lipid (glycerophospholipid) 90% TSH is higher than adult levels
- Phosphatidylcholines (lecithins) T3 is lower than adult levels
80%: Major component of surfactant T4 is similar to adult levels
- Dipalmitoylphosphatidylcholine Fetal thyroid hormone is essential to normal
principle active component of surfactant development of all tissues, especially the brain.
- Phosphatidylglycerol 8-15%; second Placenta prevents passage of maternal thyroid
most surface active hormones to the fetus by rapidly deiodinating
glycerophospholipid; has stabilizing maternal T4 to T3 to form reverse T3, a relatively
feature which prevent lung collapse inactive thyroid hormone.
- Phosphatidylinositol 4% Maternal T4 prevent antenatal
Proteins 10% cretinism in fetuses with thyroid
- surfactant A (SP-A) major apoprotein; agenesis
its synthesis is increased by treatment Changes immediately after birth:
of fetal lung tissue with CAMP Atmospheric cooling activates secretion of
analogues, epidermal GF and TSH to increase T4 levels maximal 24-36
triiodothyronine hours after birth
Simultaneous elevations in T3

C. Adrenal Glands
RESPIRATION They are larger than the standard adult size.
th
Movements can be detected by the 11 week The bulk is made up of the inner or fetal zone of
AOG. Wherein the lungs must be able to move adrenal cortex and involutes rapidly after birth
enough to provide itself with oxygen and eliminate
CO2 Sources: Dra. Coloma, Williams 24thEd, Notes from
respiratory muscles develop early higher years, LALALA-LALA, samplex
th
4 month: respiratory movements detected
(content of AF moves in and out of respiratory tract)
meconium passage poses danger in babies
because it may be aspirated to the lungs

11
OB: Fetal Growth and Development
Red mentioned by Dra. Coloma

Yellow highlight lumabas sa samplex

1
DIAGNOSIS OF PREGNANCY Usually most noticeable at 1st few
1. Presumptive signs and symptoms that weeks of pregnancy
involve other organ systems but most Easy fatigability is normal in pregnancy
prominent in reproductive tract due to increased BMR
2. Probable o However, always check with cardio and
3. Positive 100% chance that the woman is pulmo first to be sure that it is not caused by an
pregnant underlying cardio or pulmo condition
PRESUMPTIVE EVIDENCE Differential Diagnosis:
Symptoms Medicine daw (Kapagod e. Alam daw yan ng
A. NAUSEA w/ or w/o VOMITING Nanay niya. Ikaw? Alam ba ng nanay mo yun?)
Begins at 6 weeks Higher metabolic rate
Peak: 60 70 days (9-10 weeks) = peak of Big tummy
hCG (hCG ne er ero )
First 2-3 months: 50% of pregnant D. PERCEPTION OF FETAL MOVEMENT
women have peculiar distaste for food, food (Quickening)
idiosyncrasies, and other GI disturbances Quickening: first perception by the mother of
o Morning sickness (may also occur other fetal movement
times of the day) o Slight flutter or some brisk movement w/in
Correlates with the amount of circulating serum abdomen
hCG Primigravidas: usually felt at 18-20 weeks
Hyperemesis gravidarum: persistent o May mistake this as peristalsis or
vomiting aggravated by inability to take in food spasms of the GI system
o Can lead to severe dehydration Multigravidas: usually felt at 16-18 weeks
and ketonuria Increases in intensity & frequency as
pregnancy progresses
Management: At about 20 weeks, depending on maternal
Frequent, small feedings habitus, an examiner can begin to detect fetal
Avoidance of fatty food; light, dry, low fat diet movements.
Ice chips Differential Diagnosis:
Anti-emetics occasionally Peristalsis
For severe vomiting (dehydration) who are Taong magaling magkontrol ng muscle
unable to take in food and anti-emetics, hospitalize Hoks lang yun spelled as (Etch-O-A-X)
and correct fluid and electrolyte imbalance ***tapos biglang segue ng:
o Get serum electrolyte levels 1. Pseudosyosis false pregnancy very
Do an ultrasound to check for the desperate of pregnancy that they feel all these
presence of conditions that may increase hCG changes
levels 2. Ectosyesis ectopic pregnancy
1. Hyperplacentosis (multiple gestations or
molar pregnancies) E. BREAST SYMPTOMS
2. Fetal hydrops Estrogen: stimulate mammary duct system
3. Mother carrying a fetus with DS Progesterone: stimulate alveolar components
4. H-mole: gestational trophoblastic disease Breast tenderness & engorgement (esp in
Increases hCG to hundred thousands early pregnancy)
No baby is involved o Mastodynia tenderness; tingling to
5. Polycystic ovary frank pain (first weeks of pregnancy)
A subunit of hCG can interact with LH and cause Described by some px as tightness of heaviness of
hCG to increase the breasts
6. Choriocarcinoma **Less obvious changes in multiparas breasts may
contain a small amount of milky material/colostrums
Differential Diagnosis: for months or years after the birth or their last child,
Nakakasuka raw katabi mo especially if child was breastfed.
Hormone problems
GI problems Signs
F. CESSATION OF MENSTRUATION
B. DISTURBANCES IN URINATION - the endometrium will not shed
2nd and 3rd months: most prominent during - prolonged release of Progesterone
this time because the Corpus Luteum does not die
o Uterus is still pelvic organ and is quite rescued by hCG
adjacent to the bladder Earliest sign of pregnancy
Enlarging uterus puts pressure on urinary Not a reliable indicator of pregnancy (esp
bladder causing the ff: if px does not have a regular menstrual cycle)
o Irritability o Dribbling o Nocturia o Highly suspect pregnancy: 10 or more
o Frequent UTI Differentail Diagnosis days after expected onset of menses
Disappears as uterus rises (end of the 1st o 2nd missed period enhances suspicion
trimester) and becomes an abdominal organ o note the amount, interval and duration of normal
Symptoms may reappear again near menses for comparison
term when presenting part emerges Many non-pregnant factors may cause a
delay in menstruation:
C. FATIGUE Irregular cycles
Emotional Stress
1
Chronic disease Perceptible elevation of body temperature
Drugs for longer than 3 weeks
Endocrine disorders o Due to thermogenic effect of progesterone
Lactation
Genitourinary tumors Differential Diagnosis:
Luteal phase body temperature increase by 0.3
Uterine bleeding after conceptions may occur 0.5oC of the basal body temperature in the
occasionally and may be mistaken for follicular phase
menstruation (25%) - Luteal > follicular phase temperature
o Usually 1 or 2 episodes of bloody Hyperthyroid
discharge in the 1st month of pregnancy
o Usually lesser in amount and paler in J. SKIN PIGMENTATION CHANGES
color Due to an increase in estrogen and progesterone ->
o Physiological; due to blastocyst stimulates Melanocyte Stimulating Hormone ->
implantation hyperpigmentation
o Resolves spontaneously 1. Cholasma (melasma gravidarum)
o Mask of pregnancy
Differential Diagnosis: o Irregular brownish patches of varying size on
Stress Hypothalamic face & neck
Hormonal changes o Usually over forehead, bridge of nose,
Increase in hCG H-mole cheekbones, neck
Anovulatory (irregular cycles/gap) remember ha o More prominent in those with dark complexion
the cycle depends on ovulation and this causes the o Intensified by exposure to sunlight
production of progesterone and then stays for 2 o Disappear or regress after delivery
weeks on without ovulation the woman only gets
Estrogen stimulation then the shedding of the 2. Linea Nigra
Endometrium depends on how much that supply o Midline of the abdominal skin (linea alba
can ppor he prolifera ing pha e ( a ahead of line from symphysis pubis to xiphoid process) is
our Gyne) hyperpigmented to a brownish-black color
- ex. PCOS hormonal imbalance there is o Due to the stimulation of melanophores by
no dominant follicle and then the cohort stays as is increased MSH secondary to increased Estrogen
small non progressive that is why the
appearance of ha im la e (pin ol ni a bigla 3. Striae gravidarum or stretch marks
kasi may tumunog na phone) o Begins at midpregnancy or late
pregnancy when there is increased tension to skin
G. ANATOMICAL BREAST CHANGES o Reddish, slightly depressed streaks
Enlargement of the mammary glands commonly found in abdominal skin (some may
Delicate veins become visible (6-8wks be found in breasts and thighs)
after conception) Indicates recent rupture of muscle fibers
Nipples: deeply pigmented, enlarged, May turn silvery white after delivery
more erectile/everted o Due to the separation of the underlying
Areola: enlarged and more deeply pigmented collagen tissue & appear as irregular scars
Striations may be present if breast o Risk factors: weight gain during pregnancy,
size is extensive younger maternal age, family history
Hypertrophic glands of Montgomery
o Seen as small elevations scattered in the 4. Spider telangiectesia
areola o Vascular and stellate marks
o Minute, red elevations on the skin (face, neck,
Expression of colostrum (16th week) upper chest, arms) with radicles branching out from
o Thick, yellowish fluid expressed from breasts a central lesion
by gentle massage o Due to high levels of estrogen
REMEMBER! There is no relation between pre- o Blanch when compressed
pregnant breast size & volume of milk production o Palmar erythema associated sign
during lactation
Differential Diagnosis: **These changes may be absent in pregnant women.
Tumor - Enlargement They may also be present in non-pregnant
Oral Contraceptive - Tenderness women who are taking estrogen-progestin
contraceptives
H. CHANGES IN VAGINAL MUCOSA
Occurs on the 6th week **There are other states where in the Estrogen is
+ Chadwick s sign increased such as:
o Dark-bluish or purplish-red color of the Liver diseases not able to breakdown the
vagina Estrogen manifestations of Estrogen increase
- includes increase in cervical softening but can
also be due to Estrogen-Progestin contraceptives Differential Diagnosis:
o Due to increased vascularity Use of cosmetics
- Photon damage in the skin
Differential Diagnosis: Reaction to skin whitening products
Oral Contraceptives

I. THERMAL CHANGES PROBABLE EVIDENCE

2
Therefore, fern pattern in seen from day 7 to 18 of
Signs the cycle due to estrogen production
o Progesterone: lowers NaCl concentration =
A. ABDOMINAL ENLARGEMENT inhibit fernin
- After the day 21, there is beading or cellular
At 12 weeks: uterus fundus felt at appearance
symphysis pubis - During pregnancy, progesterone effect
At 16-22 weeks: growth is more rapid as predominates despite enormous estrogen produced
uterus rises out of pelvis into the abdomen - Therefore, if thin, fern-patterned mucus is seen =
At a certain period in pregnancy, tape pregnancy is UNLIKELY
measure height is equal to the age of gestation
(16-32 weeks) E. BRAXTON HICKS CONTRACTION
o The measurement from symphysis False labor pain
pubis to the uterine fundus (w/ empty bladder) = Strong, PALPABLE, VISIBLE, irregular
AOG contractions
o Makes fundic height very useful in (5-25mmHg)
gauging AOG especially if px cannot recall LMP or Usually felt most in the last two weeks of
no ultrasound is available pregnancy
o Ex. at 20 weeks = 20cm (28 weeks)
Abdominal enlargement more o May occur early in pregnancy but not as
pronounced in multigravida then primigravida strong
o Multigravida: Distended, more lax, o Increase in number when abdomen is
flaccid, pendulous abdomen due to previous massaged/stimulated
pregnancies Of short duration (true labor: long duration,
- Especially if (+) hx of multifetal more pain, regular)
pregnancies, polyhydramnios, fetal macrosomia
F. BALLOTEMENT
B. CHANGES IN UTERINE SHAPE, 20th week: Amniotic fluid volume is greater
SIZE, CONSISTENCY than fetus
Internal and external
fetus Procedure (internal ballottement):
o Growth is limited to anteroposterior 1. Insert fingers in vagina
diameter 2. Place upward pressure on fundus
o 12 weeks: uterus is somewhat globular (average 3. Use other hand to palpate
diameter: 8 cm) Findings: You will feel a rounded structure that
On bimanual examination: uterus is will hit/bounce back on your finger
elastic or exceedingly soft compared to firm, External ballottement: When examiner moves
doughy non pregnant uterus uterus side to side (with both palms on each side of
uterus), feels like something is floating/bouncing
+ He a against the palms
o 6-8 weeks Not a positive sign because if there is a
o Softening of the isthmus and fundus mass in the uterus, you will also elicit the same
(compared to the firm cervix) reaction.
- Softening is very significant that
uterus and cervix seem to be separate organs G. OUTLINING OF THE FETUS
- Makes it compressible on bimanual Myomas or ovarian overgrowths may be
examination mistaken for the fetal head
+ G de
o As early as 4 weeks H. POSITIVE ENDOCRINE TESTS
o Softening of the cervix due to increased hCG (detected as early as 8-9 days after
vascularity of tissue ovulation)
o produced by fetal trophoblasts: maintains
C. CHANGES IN THE CERVIX the corpus luteum, the main site of progesterone
6-8 weeks: softening production before the placenta kicks in
o May also be caused by estrogen- progestin o hCG in serum/urine basis for pregnancy
contraceptives tests
- hCG is a heterodimer composed of two
o Firm cervix: nose/nasal cartilage dissimilar subunits ( and )
External cervical os & cervical canal o increase from day of implantation (doubling
become patulous (open/distended) to allow time: 1.4-2 days)
insertion of fingertip o peak: 60-70 days
o Internal os should remain close o concentration declines slowly until lowest
level is reached at about 16 weeks
D. CHANGES IN CERVICAL MUCUS pregnancy tests are targeted to detecting
Mucus crystallization the subunit of hCG
o Necessary for production of fern pattern o subunit: similar to LH, FSH, TSH not
nonspecific though reliable indicator of pregnancy that s why the
o Depends on an increased NaCl concentration subunit is used
* Cervical mucus is relatively rich in NaCl when * Although each immunoassay detects a slightly
estrogen is produced (not progesterone). different mixture of hCG variants, its free subunits,

3
or its metabolites, all are appropriate for pregnancy - Visualization of cardiac activity and fetal pole
testing - Also used to measure CRL

Sandwich-type immunoassay. o Fetal echocardiography


a monoclonal an ibod again he -subunit is - Demonstrated as early as 6-8 wks AOG
bound to a solid phase support o Doppler ultrasound
The attached antibody is then exposed to and - Heard at 10 th week
binds hCG in the serum or urine specimen. A o Stethoscope
second antibody is then added, binds to another - Easiest & simplest way
i e on he hCG molec le, and and iche he th
bound hCG between the two antibodies. - Disadvantage: hear FHT at 18 week
In some assays, the second antibody is linked to Other sounds heard:
an enzyme, such as alkaline phosphatase. 1. Uterine soufflé
When substrate for the enzyme is added, a color Soft, blowing sound synchronous with maternal
develops. The color intensity is proportional to the pulse
amount of enzyme and thus to the amount of the Heard in the later months of pregnancy
second antibody bound. Produced by the passage of blood through the
This is a function of the hCG concentration in the dilated uterine vessels
test sample. Heard most distinctly near the lower portion of
The sensitivity for the laboratory detection of hCG the uterus (hypogastric area of abdomen)
in serum is as low as 1.0 mIU/mL using this 2. Funic soufflé or umbilical cord soufflé
technique Sharp, whistling sound that is synchronous with fetal
extremely sensitive immunoradiometric assays, the pulse
detection limit is even lower Caused by the rush of blood through the umbilical
arteries
**False positive hCG results are rare with the May not be heard consistently
sandwich type immunoassay 3. Sound from movements of fetus
o Possible causes of false +: 4. Maternal pulse
Presence of circulating serum factors Pulsation of aorta is unusually loud
(heterophilic antibodies usually seen in women who 5. Gurgling gas in mother s GIT
work with animals) that may interact with hCG Sounds from maternal internal peristalsis
antibody
Elevated hCG levels may also reflect molar B. PERCEPTION OF FETAL MOVEMENT BY
pregnancy and its associated cancers EXAMINER
Other rare causes of positive assays without Active fetal movement may be seen and felt
pregnancy are: by the
(1) exogenous hCG injection used for weight loss examiner
(2) renal failure with impaired hCG clearance In advanced pregnancies, fetal movements
(3) physiological pituitary hCG can be visualized by inspection
(4) hCG-producing tumors that most commonly o Visible bowel movements are rarely seen
originate from gastrointestinal sites, ovary, bladder, Usually occurs late in pregnancy (after 20th
or lung week)
o Detected earlier with real times sonography
Home pregnancy tests: 75 percent
sensitivity, high C. RECOGNITION OF THE EMBRYO OR FETUS
false negative rate BY ULTRASOND TECHNIQUES
o Detection limit: 12.5mIU/mL Detects fetal heart activity
Commercially available tests involve At 5 weeks, gestational sac may be seen
the ff: agglutination-inhibition, radio-immunoassay, o 2mm sac = 16 days from ovulation or 10 days
enzyme- linked absorbent assay (ELISA), from implantation
immunochromatography At 6 weeks, fetus w/in sac & fetal heart beat
detected
6-12 weeks: CRL measurement is
POSITIVE EVIDENCE predictive of gestational
A. IDENTIFICATION OF FETAL HEART TONES Preferred is transvaginal sees signs of
Auscultation of fetal heart tones apart from pregnancy by 4-5 weeks
mother s own pulse is a positive sign that there is a - gestational sac is seen in the decidua change in
viable pregnancy the endometrium for the pregnancy state form a
o How to do it: Get maternal pulse and FHT sac like thing 5-6 weeks a white rim forms which
simultaneously Maternal pulse and fetal heart tone is the gestational sac
must not be synchronous - yolk sac and embryo, as time goes by
- Maternal thyrotoxicosis or fever can mimic the fetal separates
heart tone due to increased maternal HR - in the UTZ there will be a flicker which is
o Usually FHT is faster than the mother s (110-150 he hi e hing (Y p, hindi ni a ina abi ano yun)
beats per minute) - look for the sac in other areas because it
might be ectpic
Different ways to listen to FHT (arranged
from the earliest to the latest): ** J read he book for hi proce r o read
o Ultrasound (real time sonography) back (I g e placen a )
- Heard at the 5th weeks ** X-ray the pelvic bone covers the fetal bones
- Most accurate o mahirap rin i o.

4
11. Ultrasound may show
Other information that could be verified by ultrasound oligohydramnios/anhydramnios and particulate
presence of blighted ovum matters floating in scanty AF
number of foetuses
ectopic gestation Radiographic Evidence nice to know - not used
presenting part anymore
fetal anomalies 1. Spalding s sign: overlapping of the fetal
hydramnios skull bones due to liquefaction of the brain
detection of IUGR 2. Robert s sign: gas bubbles in the fetus
3. Exaggeration of fetal spine curvature
Differential Diagnosis
Other conditions may result in enlargement of the Extra from the book:
abdomen and may be mistaken for a pregnancy Sonographic Recognition of Pregnancy
o myomas (esp fundal solitary subserous variety) Transvaginal sonography
o adenomyosis early pregnancy imaging and is commonly used to
o solid ovarian masses accurately establish gestational age and confirm
o hematometra pregnancy location
First sonographic evidence of pregnancy: A
PSEUDOCYESIS gestational sac a small anechoic fluid collection
Imaginary or spurious pregnancy within the endometrial cavity
o Have nausea, vomiting, Seen with transvaginal sonography by 4 to 5
quickening, sensation of fluid coming out (amniotic eek ge a ion
fluid) Fluid collection, however, can also be seen within
o Feel majority of signs and symptoms but the endometrial cavity with an ectopic pregnancy
they are not pregnant and is termed a pseudogestational sac or
Usually seen in women who are nearing pseudosac
menopause or those with a strong desire to become Further evaluation may be warranted if this is the
pregnant only sonographic finding, particularly in a patient
Other risk factors: with pain or bleeding.
o Obese, emotional problems (strong desire to Normal gestational sac implants eccentrically in the
become pregnant), > 35 y/o endometrium (whereas a pseudosac is seen in the
Women with pseudocyesis are usually midline of the endometrial cavity)
hard to convince that they are not pregnant Other potential indicators of early intrauterine
o Do an ultrasound! (most convincing method) pregnancy:
o Bimanual exam may also be done Anechoic center surrounded by:
- Uterus will be seen as small & (-) positive 1. single echogenic rim the intradecidual sign
signs of pregnancy 2. two concentric echogenic rings surrounding the
gestational sac the double decidual sign
IDENTIFICATION OF FETAL LIFE & DEATH sonography yields equivocal findings the so-
In 50% of cases, cause of death is called pregnancy of unknown location, then serial
unexplainable serum hCG levels can also help differentiate a
Pregnancy tests are not reliable because normal intrauterine pregnancy from an extrauterine
trophoblasts continue to produce hCG for several pregnancy or an early miscarriage
days or weeks after fetal demise Visualization of the yolk sac
Ultrasound to confirm nonviability - a brightly echogenic ring with an
o If not available do bimanual pelvic exam & anechoic center
auscultation of FHT with stethoscope confirms with certainty an intrauterine location for
Decrease in uterine size the pregnancy and can normally be seen by the
o Due to collapse/shrinkage of fetal skeleton middle of the fifth week.
o Collapse of skull w/ liquefaction of fetal brain after 6 weeks, an embryo is seen as a linear
structure immediately adjacent to the yolk sac, and
Clinical Findings that indicate non-viability of the fetus cardiac motion is typically noted at this point
1. Cessation of fetal movements or an Up o 12 eek ge a ion, he cro n-rump length
appreciable decrease in activity is predictive of gestational age within 4 days
2. Cessation of vomiting for hyperemetic px
3. Weight decreases/returns to normal
4. Cessation of fetal movement usually
the most noticeable sign in fetal demise that
occurs in the period after quickening
o Position changes due to amniotic fluid may
be mistaken for fetal movements
5. Loss of breast turgor & engorgement
6. If previously hypertensive, BP decreases.
7. Decrease in fundic height
8. Internal exam: soft collapsible fetal skull
may be felt through soft cervix that has already
started to dilate
9. No fetal heart tone
10. Tobacco stained amniotic fluid highly
suggestive of fetal demise

5
TETRALOGY, TERATOGENS AND FETOTOXIC CRITERIA FOR DETERMINING TERATOGENICITY
AGENTS

Birth defects

- Nearly 70% of birth defects do not have an


obvious etiology, and those with identified
cause are more likely to be genetic than
teratogenic

Tetralogy
- Study of birth defects and their etiology
Teratogen
- came from Greek work teratos, meaning
monster
- Any agent that acts during embryonic or fetal
development to produce a permanent
alteration of form or function Although all criteria may not be required to establish
- May be a drug or any chemical substance, a teratogenicity, the following tenets must be
physical or environmental factor (heat or considered:
radiation), a maternal metabolite (such as in
PKU or diabetes), a genetic abnormality of an
infection
- Causes structural abnormalities
Hadegen The defect has been completely characterized
- Is an agent that interferes with normal – done because various genetic and
maturation and function of an organ environmental factors may produce
- Named after the god Hades similar anomalies
Trophogen – it is easiest to prove causation if a
- An agents that alters growth rare exposure produces a rare defect,
when at least 3 cases with the same
exposure have been identified and
Both Hadegen and trophogen affect
when defect is severe
development in the fetal period or after birth ,
The agent must cross the placenta
when exposures are often difficult to
– Transport must be sufficient quantity
document
to directly influence embryonic or fetal
Teratogen is used to refer to these types of
development or to alter maternal or
agents
placental metabolism to exert an
Hydroxyprogesterone caproate
indirect effect
- Only medication approved between 1996 and o Placental transfer depends
2011 specifically for pregnancy for recurrent on:
preterm birth prevention 1. Maternal metabolism
Pregnant women are excluded protein binding and
from drug trials to protect the storage, molecular size,
embryo and fetus from potentially electrical charge and lipid
harmful effects of a medication solubility
and because pregnancy 2. Placental metabolism
physiology affects the medication. CYP 450 enzyme
Trofurantoin and Sulfonamides systems
- Appropriate for first trimester use if no suitable Exposure must occur during a critical period
alternative is available of development:
- First-line agents for treatment and prevention 1. Preimplantation period
st
of UTI beyond 1 trimester – 2 weeks from fertilization to
implantation
– Kno n a he all or none
period
– As zygote undergoes
cleavage, an insult
damaging large number of
cells causes embryonic
death
– Compensation is possible if
only few cells are damaged
2. Embryonic period
nd th
– 2 -8 week
– Organogenesis
– most crucial period with
regard to structural
malformations
1
3. Fetal Period
– >8weeks
– Continued maturation and
functional development
– Certain organs remain
vulnerable (ex. Brain
development susceptible
to environmental influences
such as alcohol exposure)
A biologically plausible association is
supportive
– Because birth defects and
medication exposures are
both common, they may
be temporarily but not
causally related
Epidemiological findings must be consistent
– Important criterion for
teratogenicity: 2 or more
high-quality
epidemiological studies
report similar findings
The suspected teratogen causes defect in
animal studies
– Human teratogenicity is Teratogens
more likely is an agent
produces adverse effects - Act by disturbing specific physiological
in animals process, which may in turn lead to abnormal
Exception: thalidomide potent cellular differentiation, altered tissue growth,
human teratogen but produces no or cell death
defects in several animal species - Exposure may result in multiple effects
Failure to employ these tenets and criteria has because pathophysiological processes may
contributed to erroneous conclusions be induced by various cell types and tissues
regarding the safety of some widely used - May produce similar phenotypic abnormalities
drugs. because different teratogens may disturb
similar processes
Low-risk teratogens

- Medications that produce defects in fewer Fetal genome


than 10 per 1000 maternal exposures
- Ex. Corticosteroids, lithium, trimethropin and - Genetic composition has been linked to
methimazole susceptibility to teratogenic effects of specific
medications
- Ex. Fetuses exposed to hydantoin are more
likely to develop anomalies if homozygous for
a gene mutation that results in abnormally low
levels of epoxide hydrolase

Disruption of Folic Acid Metabolism

- Fetal neural tube defects, cardiac defects and


oral clefts can be a result of folic acid
metabolic pathway disturbances

Folate

- Essential for methionine production, which is


required for gene methylation and thus
production of proteins, lipids and myelin
Anticonvulsants such as
phenytoin, carbamazepine,
valproic acid and phenobarbital
– May either impair folic acid
absorption or act as folic
acid antagonist

Paternal exposures

- Exposures to drugs and environmental


influences may increase risk of adverse fetal
outcome
- Proposed mechanisms:
2
induction of a gene mutation or
chromosomal abnormality in
sperm
– 64 days germ cells
mature into sperm
– Exposure during the 2
months before conception
could cause gene
mutations
Epigenetic pathways suppress
germ-cell apoptosis or interfere
with imprinting
During intercourse, the
developing embryo is exposed to
a teratogenic agent in seminal
fluid
- Exposure to mercury, lead, solvents,
pesticides, anesthetic gases or
hydrocarbons associates with early
pregnancy loss

General rule: Since no adequate and well-


controlled studies in pregnant women for most
medications, and because animal
reproduction studies are not always predictive
of human response, any medication in
pregnancy must be carefully considered and
only used if clearly needed.

Known and Suspected Teratogens:

1. Alcohol
- Most potent and prevalent teratogen
- One of the most frequent nongenetic cause of
mental retardation
- Leading cause of preventable defects in the
US
Fetal alcohol syndrome
– alcohol-related fetal defects
– include dysmorphic facial
features, pre- or postnatal 2. Anticonvulsant medications
growth impairment, and CNS - No anticonvulsant drugs are considered truly
abnormalities that may be afe in pregnanc
structural, neurological or - Most medications used to treat epilepsy has
functional been proven or suspected to confer an
Fetal alcohol spectrum disorder increased risk for fetal malformations
– an umbrella term that Several older anticonvulsants
includes the full range of produce a constellation of
prenatal alcohol damage that malformations similar to fetal
may not meet the criteria for hydantoin syndrome
fetal alcohol syndrome Valproic aicd
Other alcohol-related major and minor – confers the greatest risk;
birth defects include cardiac and renal significantly increases risk for
anomalies, orthopedic problems and malformations
abnormalities of the eyes and ears – children with in utero
- There is unknown minimum amount of alcohol exposure are reported to
required to produce adverse fetal have significantly lower IQ
consequences scores at age 3 compared
Binge drinking believed to pose with those exposed to
particular high risk for alcohol-related phenytoin, carbamazepine or
birth defects and increases risk for lamotrigine
stillbirth Topimarate
– Among the newer agents, it is
reported to confer a risk for
orofacial clefts at least
fivefold higher than in
unexposed pregnancies
- Most frequently reported anomalies:
– Orofacial clefts
– Cardiac malformations
– Neural tube defects
3
animal species, it is associated with
3. ACE inhibitors & ARBs multiple abnormalities
- ACE inhibitors Abnormalities include
– considered fetotoxic and result in hydrocephalus, eye
ACE-inhibitor fetopathy anomalies, skeletal
– causes fetal hypotension and renal abnormalities and embryo
hypoperfusion, with subsequent death
ischemia and anuria – its active metabolite is detectable in
Normal renal development depends plasma for up to 2 years following its
on the renal-angiotensin system discontinuation
Reduced perfusion may cause fetal- Women of childbearing
growth restriction and calvarium potential who discontinue this
maldevelopment, whereas medication should consider
oligohydramnios may result in cholestyramine treatment/
pulmonary hypoplasia and limb washout, followed by
contractures verification that serum levels
- ARBs are undetectable on 2 tests
– Since it has same mechanism of performed 14 days apart
action as ACEI, concerns regarding
fetotoxicity has been generalized to 6. Antimicrobial Drugs
include this entire mediation class - Aminoglycosides
– Preterm infants treated with
4. Antifungal Medications gentamicin or streptomycin have
- Fluconazole developed nephrotoxicity and
– Has been associated with a pattern of ototoxicity
congenital malformations resembling - Chloramphenicol
the autosomal recessive Antley-Bixler – Gray baby syndrome was described
Syndrome in neonates who received this
– Abnormalities include: oral clefts, medication
abnormal facies and cardiac, skull, Preterm infants were unable
long bone and joint abnormalities to conjugate and excrete the
reported only with chronic high-dose drug manifested abdominal
treatment in the first trimester (400- distention, respiratory
800mg daily) abnormalities, an ashen-gray
color and vascular collapse
5. Anti-inflammatory Agents - Nitrofurantoin
- NSAIDs – First trimester exposure include a
– Includes both aspirin and traditional fourfold increased risk for hypoplastic
NSAIDs such as ibuprofen and left heart syndrome and
indomethacin micropthalmia/ anopthalmia and a
– Low dose aspirin, 100mg daily or twofold increased risk for clefts and
lower, does not confer increased risk atrial septal defects
for constriction of the ductus - Sulfonamides
arteriosus or for adverse infant – Often combined with trimethroprim
outcomes and used to treat various infections
– High dose aspirin and NSAIDs should during pregnancy such as MRSA
be avoided particularly during the – First trimester exposure increases risk
third trimester for anencephaly and left ventricular
– NSAIDs may cause adverse fetal outflow tract obstruction, choanal
effects when taken in late pregnancy atresia, and diaphragmatic hernia
Indomethacin may cause – Displaces bilirubin from protein
constriction of the fetal ductus binding sites which may worsen
arteriosus resulting in hyperbilirubinemia if given near time
pulmonary hypertension; may of preterm delivery
also decrease fetal urine - Tetracyclin
production and thereby – Associated with yellowish-brown
reduce amniotic fluid discoloration of deciduous teeth when
volume presumed due to used after 25 weeks
an increase in vasopressin
levels and vasopressin 7. Antineoplastic Agents
responsiveness - Cyclophosphamide
Fetal ductal constriction is – Alkylating agent that inflicts a
more likely when drug is chemical insult on developing fetal
rd
taken in the 3 trimester for tissues and leads to cell death and
longer han 72 ho r d ra ion heritable DNA alterations in surviving
- Leflunomide cells
– Pyrimidine-synthesis inhibitor used to – Increases pregnancy loss and
treat rheumatoid arthritis reported malformations include
– Contraindicated in pregnancy skeletal abnormalities, limb defects,
because when given at or below cleft palate and eye abnormalities
human-equivalent doses in several
4
– Surviving infants may have growth – Exposure of a female fetus may
abnormalities and developmental cause virilization and may result in
delays ambiguous genitalia similar to that
– Environmental exposures increase encountered in cases of congenital
risk for spontaneous abortion adrenal hyperplasia
- Methotrexate – Labioscrotal fusion first trimester
– folic acid antagonist exposure
– a potent teratogen – Phallic enlargement later fetal
– used for cancer chemotherapy, exposure
immunosuppression, nonsurgical - Danazol
treatment of ectopic pregnancy and – Used to treat endometriosis,
as an abortifacient premenstrual syndrome and
– can cause defects known collectively fibrocystic breast disease
known as fetal methotrexate- – 40% of exposed female fetuses were
aminopterin syndrome includes virilized
cranio n o i i h clo er-leaf skull, – May cause dose-related pattern of
wide nasal bridge, low set ears, clitoromegaly, fused labia and
micrognathia and limb deformities urogenital sinus malformation
2
– 50mg/m dose given to treat ectopic - Diethylstilbestrol
pregnancy – In utero exposure increases risk for
Dose is increased to induce vaginal and cervical intraepithelial
elective abortion neoplasia
- Tamoxifen – Has been associated with genital tract
– Nonsteroidal selective estrogen- abnormalities in exposed fetuses of
receptor modulator (SERM) both genders
– Used as an adjuvant to treat breast Women may have
cancer hypoplastic, T-shaped uterine
– Women who become pregnant while cavity, cervical collars, hoods
on therapy or within 2 months of its septa and coxcombs and
discontinuation should be appraised withered fallopian tube;
for long-term risk of diethylstrilbestrol slightly higher rates of earlier
(DES)- like symptoms menopause and breast
– Exposed infants should be monitored cancer
for carcinogenic effects for up to 20 Men may develop
years epididymal cyst, microphallus,
- Trastuzumab hypospadias, cryptorchidism
– Recombinant monoclonal antibody and testicular hypoplasia
directed to the human epidermal
growth factor 2 (HER 2) protein 11. Immunosuppressant Medications
– Used to treat breast cancers that over - Corticosteroids
express HER2 protein – Associated with clefts in animal
– Associated with oligohydramnios, studies
anhydramnios and fetal urine faiure – Prednisolone active metabolite of
– May result in fetal pulmonary prednisone; inactivated by placental
hypoplasia, skeletal abnormalities and enzyme 11-beta-hydroxysteroid
neonatal death dehydrogenase 2 and does not
effectively reach the fetus
8. Antiviral Agents - Mycophenolate Mofetil
- Ribavirin – Potent immunosuppressant used to
– Nucleoside analogue prevent rejection in organ-transplant
– Component therapy for hepatitis C recipients
infection – Also used for autoimmune disease
– Causes multiple defects in multiple such as lupus nephritis
animal species – May cause abortion
– Women are recommended to use two – Surviving infants may have
forms of contraception while on this malformations (usually ear
therapy and delay childbrearing for 6 deformities)
months following drug administration
- Efavirenz 12. Radioiodine
– Nucleoside reverse transcriptase - Radioactive iodine -131 is used to treat
inhibitor used to treat HIV infection thyroid cancer and thyrotoxicosis and for
– CNS abnormalities following human diagnostic thyroid scanning
exposure - Contraindicated in pregnancy because it
readily crosses the placenta and is then
9. Endothelin-Receptor Antagonists concentrated in the fetal thyroid gland by 12
- Teratogenic for mice; no human data weeks
available - Causes irreversible fetal hypothyroidism and
may increase risk for childhood bearing
10. Sex Hormones thyroid cancer
- Testosterone and Anabolic Steroids

5
13. Lead associated with pulmonary
- Associated with fetal-growth abnormalities hypertension of the newborn
and with childhood development delay and - Antipsychotic Medications
behavioral abnormalities – Exposed neonates have manifested
- There is no lead exposure that is considered extrapyramidal muscle movements
safe in pregnancy and withdrawal symptoms, including
agitation, abnormally increased or
14. Mercury decreased muscle tone, tremor,
- Prenatal exposure causes disturbances in sleepiness, feeding difficulty and
neuronal cell division and migration and lead respiratory abnormalities
to a range of defects from developmental
delay microcephaly and severe brain damage 16. Retinoids
Principal concern for prenatal mercury - Vitamin A derivatives
exposure is the consumption of - Among the most potent human teratogens
certain species of large fish - Isotretinoin, acitretin and bexarotene highly
Advice not to eat shark, swordfish, teratogenic when orally administered
king mackerel, or tilefish, and - Inhibits neural-crest migration during
consumption of albacore tuna should embryogenesis resulting in neural crest
be limited to 6ounces per week defects termed retinoic acid embryopathy
retinoic acid embryopathy
15. Psychiatric Medications involve the CNS, face, heart
- Lithium and thymus
– Associated with Ebstein anomaly - specific anomalies: ventriculomegaly,
– Exposure near delivery findings maldevelopment of facial bones and cranium,
typically persist 1-2 weeks and may microtia or anotia, micrognathia, cleft palate,
include hypothyroidism, diabetes conotruncal heart defects and thymic aplasia
insipidus, cardiomegaly, bradycardia, or hypoplasia
ECG abnormalities, cyanosis and - Isotretinoin
hypotonia – First trimester exposure is associated
Ebstein anomaly with high rate of pregnancy loss and
– a cardiac abnormality fetal malformations
characterized by apical
displacement of the
tricuspid valve
– often results in severe
tricuspid regurgitation
and marked right atrial
enlargement, which
confer significant
morbidity
Fetal Echocardiography
should be considered for
pregnancies exposed to - Acitretin
lithium in the first trimester – Was introduced to replace etretinate
- Selective Serotonin- and Norepinephrine- which is a lipophilic retinoid with such
Reuptake Inhibitors a long half-life that birth defects
– As a class, not considered major resulted more than 2 years after
teratogens therapy was discontinued
– Exception: paroxetine associated - Bexarotene
with increased risk for cardiac – Causes abnormalities in rats such as
anomalies particularly atrial and eye and ear abnormalities, cleft palate
septal defects and incomplete ossification
Paroxetine should be avoided – Males with partners who could
in women planning pregnancy become pregnant are advised to use
and fetal ECG should be condoms during sexual intercourse
considered for those with 1
st while taking this drug and for one
trimester exposure month after discontinuing therapy
– May cause neonatal behavioral - Topical retinoids
syndrome jittering, irritability, hyper- – Initially used to treat acne; popular
or hypotonia, feeding abnormalities, treatment for sun damage that they
vomiting, hypoglycemia, are called cosmeceuticals
thermoregulatory instability and – Include tretinoin and tazarotene
respiratory abnormalities; neonatal – Tretinoin was reported to cause
effects are typically mild and self- malformations following topical
limited, lasts for only about 2 days administration
– Late-pregnancy exposure to SSRI - Vitamin A
may manifest severe adaptation – Has 2 natural forms: Beta- carotene &
abnormalities, including seizures, Retinol
hyperpyrexia, excessive weight loss Retinol is a preformed vitamin
and respiratory failure; also A which has been associated
with cranial neural-crest

6
defects when >10,000IU per 19. Herbal Medicines
st
day is consumed in the 1
trimester
Avoid doses >3,000IU daily

17. Thalidomide & Lenalidomide


- Thalidomide
– most notorious human teratogen
– causes malformations in 20% of
fetuses exposed between 34-50 days
menstrual age
– characteristic malformation:
phocomelia absence of one or
more long bones which result in the
hands or feet being attached to the
trunk by a small rudimentary bone
– cardiac malformations, GI
abnormalities and other limb
reduction defects are also common
Important Teratological
Principles:
1. The placenta is not a perfect
barrier to the transfer of toxic
substance from mother to
fetus
2. There is extreme variability in
species susceptible to drugs
and chemicals. Because
thalidomide produced no
defects in experimental mice 20. Recreational Drugs
and rats, it had been - Amphetamines
assumed to be safe for – In utero exposure associated with
humans fetal-growth restriction and with
3. There is a close relationship behavioral abnormalities in both
between exposure timing and infancy and early childhood
defect type. - Cocaine
Upper-limb phocomelia – Most adverse outcomes result from its
thalidomide exposure vasoconstrictive and hypertensive
during days 27-30 effects
Lower-limb phocomelia – Can cause serious maternal
days 30-33 complications such as
Gallbladder aplasia days cerebrovascular hemorrhage,
42-43 myocardial damage and placental
Duodenal atresia days abruption
40-47 – May be associated with cleft palate,
CV abnormalities and urinary tract
18. Warfarin abnormalities
- Vitamin K antagonist and a potent – Also associated with fetal growth
anticoagulant restriction and preterm delivery
- Has low molecular weight and readily crosses – Children exposed as fetuses
the placenta causing embrotoxic and fetotoxic increased risk for behavioral
effects abnormalities and cognitive
-
th th
Exposure between 6 -9 week warfarin impairments
embryopathy (characterized by stippling of the - Opiods- Narcotics
vertebrae and femoral epiphyses and by nasal – Periconceptional exposure
hypoplasia with depression of the nasal bridge Increased risk for spina bifida,
- Affected infants may also have choanal gastroschisis and cardiac anomalies
atresia resulting to respiratory distress – Strongly associated with fetal and
- Increased risk of embryopathy in women who neonatal effects
require >5mg daily – Heroin-addicted mothers increased
- May result in hemorrhage into fetal structures risk of preterm birth, placental
st
when used beyond 1 trimester causing abruption, fetal growth restriction and
abnormal growth and deformation from fetal death (due to effects of repeated
scarring narcotic withdrawal on the fetus and
- Abnormalities may include agenesis of the placenta)
corpus callosum, cerebellar vermian agenesis Neonatal Narcotic Withdrawal
(Dandy-Walker Malformation), micropthalmia - called the neonatal
and optic atrophy abstinence syndrome;
- Affected infants at risk for blindness, deafness manifest in 90% of exposed
and developmental delays infants

7
- characterized by CNS gastroschisis, cleft lip and palate and hand
irritability leading to abnormalities
seizures if untreated, along - Dose response reduction in fetal growth
best-documented adverse reproductive
with tachypnea,
outcome from smoking
episodes of apnea, poor - Newborns of mother who smoke weigh an
feeding and failure average of >200g than newborns of
to thrive nonsmokers
- at risk neonates - Doubles the risk of low birth rate and
monitored using scoring increases risk of fetal growth restriction
system - Growth disparity detected sonographically
between 10-20weeks
- severely affected
- Women who stop smoking early in
neonates treated with pregnancy generally have neonates with
opioids normal BW
Methadone - Has also been linked to subfertility and
- synthetic opioid routinely spontaneous abortions, increase risk for
offered to pregnant women to placenta previa and placental abruption and to
obviate uncontrolled narcotic preterm delivery
withdrawal
- increases risk of preterm
birth and fetal-growth
restriction
- Miscellaneous drugs
– Marijuana not associated with
human fetal anomalies; teratogenic
to animals if given in high doses
– Phencyclidine (PCP) or angel dust
not associated with congenital
anomalies; most exposed newborns
experience withdrawal symptoms
characterized by tremors, jitterness,
and irritability
– Toluene common solvent used in
paints and glue; exposure reported to
have significant fetal risks
Toluene embryopathy
- toluene abuse in early
pregnancy
- phenotypically similar to
fetal alcohol syndrome
- Abnormalities: pre- and
postnatal growth deficiency,
microcephaly and
characteristic face and hand
findings (includes midface
hypoplasia, short palpebral
fissures, wide nasal bridge
and abnormal palmar creases

21. Tobacco
- Cigarette smoke contains a complex of
mixture of nicotine, cotinine, cyanide,
thiocyanate, carbon monoxide, cadmium, lead
and various hydrocarbons
- Fetotoxic
- Have vasoactive effects and reduce oxygen
levels
- Not considered a major teratogen, although
associated with selected birth defects
- Vasoactive properties could produce
congenital defects related to vascular
disturbances
- Increased risk for cardiac anomalies has been
reported and may be dose-related
- Maternal smoking is associated with
hydrocephaly, microcephaly, omphalocele,

8
Prenatal, tetralogy, immunizations, drugs, high risk pregnancy: EMR, JV-C & PDV

Prenatal Care
INITIAL PRENATAL EVALUATION
Prenatal Care ini ia ed a oon a here a rea onable likelihood of pregnanc .
Major goals are to:
define the health status of the mother and fetus
estimate the gestational age
initiate a plan for continuing OB care

The initial plan for subsequent care may range from relatively infrequent routine visits to prompt hospitalization
because of serious maternal or fetal disease.

Prenatal Record
Use of a standardized record within a perinatal health care system greatly aids antepartum and
intrapartum management
Standardized Documentation
o allow communication and care continuity between providers
o enable objective measures of care quality to be evaluated over time and across different clinical
settings

Definitions
1. Nulligravida - not currently nor has ever been pregnant
2. Gravida - is currently pregnant or has been in the past, irrespective of the pregnancy outcome.
Primiravida first pregnancy
Multigravida with successive pregnancies
3. Nullipara - ne er comple ed a pregnanc be ond 20 eek ge a ion.
ha en been pregnan
had a spontaneous or elective abortion(s)
an ectopic pregnancy
4. Primipara delivered only once, born alive or dead with >20wks AOG.
NB: 500g BW threshold definition for parity
- controversial: this is used to differentiate stillborn from an abortus
- <500g of BW = common
5. Multipara - completed two or more pregnancies to >20 wks AOG.

PARITY
number of pregnancies reaching 20 weeks
not increased if multiples are delivered in a given pregnancy (twins, tripltes, quadruplets = P 1)
stillbirth does not lower this number

OB SCORE: GP (T-P-A-A)
Term: >37wks (37 - 42)
Preterm: >20wks but <37wks
Post term: >42 weeks
Abortus: <20wks
o Ectopic (tubal pregnancy) | H-mole
Ali e: c rren l UNTIL NOW!

1
Prenatal, tetralogy, immunizations, drugs, high risk pregnancy: EMR, JV-C & PDV

OB Scoring Sample Cases


E.R. S b ec JV-C a d PDV S b ec

1. 25yo CC: missed menses Case 1:


LMP: July 3-7, 2014 Patient: A.P.
(+) pregnancy test AOG= 19weeks
OB Hx: OB History:
- 2009: pu 37-38wks NSD male 2.7Kg - 2013: 38 weeks delivered by NSD
- 2010: pu 8-9wks abortion
- 2012: pu 36-37wks NSD female 2.6Kg Answer: G2 P1
G4P2 (1-1-1-2) Case 2:
2. 25yo CC: missed menses Patient: L.T., 40y/o,
LMP: July 3-7, 2014 AOG= 28 weeks
(+) pregnancy test OB History:
OB Hx: - 2004: 37 weeks delivered to a live
- 2009: pu 37-38wks NSD male 2.7Kg baby
- 2010: pu 8-9wks abortion - 2006: abortion 15 weeks
- 2012: twin preg 26-27wks, del. By 1° LT - 2007: delivered 32 weeks via NSD
o A 600g; B 700g which died
o Died after 2 days
G4P2 (1-2-1-1) Answer: G4 P2 (T1 P1 A1 L1)
3. 25yo CC: missed menses Case 3:
LMP: July 3-7, 2014 Patient: L.C., 35y/o
(+) pregnancy test AOG= 9weeks
OB Hx: OB History:
- 2009: pu 37-38wks NSD male 2.7Kg - 2009: twin pregnancy 32weeks, 1 died
- 2010: pu 8-9wks abortion - 2010: right salphingectomy with
- 2012: twin preg 39-40wks, del by 1° LT ectopic pregnancy
o A 2600g; B 2700g - 2012: 32 weeks delivered by NSD
G4P2 (3-0-1-3)
4. 25yo CC: missed menses Answer: G4 P2 (T0 P3 A1 L2)
LMP: July 3-7, 2014 Case 4:
(+) pregnancy test Patient: A.T., 30y/o
OB Hx: AOG= 28 weeks
- 2009: pu 37-38wks NSD male 2.7Kg OB History:
- 2010: pu 8-9wks abortion - 2005: H Mole, suction curettage done
- 2012: pu 39-40wks IUFD 3.5kg - 2006: twin pregnancy via cesarean at
o IntraUterine Fetal Death 34 weeks
G4P2 (2-0-1-1) - 2007: ectopic pregnancy, right
5. 25 yo G1P0 10-11wks AOG PRIMI salphingectomy
- (+) Fetal Heart Tone Doppler - 2008: completion curettage
As clinicians, we must know the ffing: - 2009: IUFD at 23 weeks
LMP with AOG & EDD
Examine total health of Pt. Answer: G6 P2 (T0 P3 A3 L2)
Head to toe Case 5:
Look for an o her problem Patient: 39y/o
that may affect pregnancy AOG= 36weeks
Diagnosis of Pregnancy OB History:
Lab test are usually last to - 2009: Twin pregnancy, 8 weeks,
perform completion curettage done
To verify/confirm reliability of - 2010: NSD at 36 weeks
OB history. - 2011: Twin pregnancy, 37 weeks, 1
died
- 2013: ectopic pregnancy

Answer: G5 P2 (T2 P1 A3 L2)

2
Prenatal, tetralogy, immunizations, drugs, high risk pregnancy: EMR, JV-C & PDV

According to Dra.: Fundic height in cm is equal to gestational


Good prenatal care= good delivery age at 16 32 weeks

3 major objectives: Ultrasound


1. Determine AOG Useful in detecting common obstetrical
2. Assess fetal well-being problems that cluster in each trimester
3. Initiate a plan of continuing care 1st term: spontaneous abortions
3rd term: pregnancy-induced hypertension
Minimum of 4 prenatal care is recommended
Twin Pregnancy count as G1; P1 if >20
weeks; but is counted separately for TPAL Previous and Current Health Status
H mole count as G1 Menstrual History
Curretage, ectopic pregnancy count as Clinically important
abortus (<20weeks) Gestational or Menstrual age number of
weeks since the onset of LMP
Normal Pregnancy Duration Ovulatory cycles without a Hx of regular,
st
280days / 40weeks mean duration from 1 day of predictable, cyclic, spontaneous menses
normal LMP o Difficulty in accurate dating of
Mean Pregnancy Duration: 281 days with SD of pregnancy
13days o by Hx and PE
Divided into 3 equal epochs (approx. 3mos /
14wks) Normal Cycle: 28days, ovulation takes place on the
th
First Trimester 14 day
o Period of Organogenesis Irregular Cycle: >28-30days, ovulation is beyond day
o Spontaneous abortion takes place 14
Second Trimester Chronic anovulation menstrual period with
o Period of organ maturation a very much longer and irregular intervals
Third Trimester
o Dx of HPN disorders for mothers NOTES:
I important to ascertain whether or not
EDD (Expected Delivery Date): Naegele Rule steroidal contraceptives were used before
st
1 day of LMP + 7days 3mos + 1year = the pregnancy
EDD o ovulation may not have resumed 2
weeks after the onset of the last
Gestational Age / Menstrual Age withdrawal bleeding. Instead, may
Assumes pregnancy begun 2 weeks before have occurred at an appreciably
ovulation later and highly variable date
Used by clinicians to mark temporal events o Use of sonography in early
during pregnancy pregnancy
Clarify gestational age
Ovulatory Age / Fertilization Age
Used by embryologists and other Obstetrical History Gravidity, Parity, and OB
reproductive biologists score
Typically two weeks earlier
Medical and Surgical History DM, Thyroid,
Precise knowledge of fetal age is imperative for ideal HPN, Asthma, Seizure disorder
OB managemen , eek of ge a ion comple ed i If patient is positive for any of the medical and
a clinically appropriate unit for use. surgical history, patient is considered as a high- risk
For 33 completed weeks and for days px.
4/7
o Either 33 weeks or 33+4
Other pertinent history:
Timing from ovulation add 267 days to last - Includes complete past and family history & a
ovulation date to determine EDC good obstetric resume
- Obstetric resume should emphasize the ff:
Timing from quickening movement perceived Menstrual history
from 16th to 18th week (multipara); 18th 20th Evidence of infertility
week (primipara) Careful inquiry into previous pregnancy
- this method is more useful as a confirmation of Time in gestation when labor occurred
other parameters Duration, type of delivery and complications
W eight and sex of baby
Height of the Fundus
Woman reac ion o c rren pregnanc
Fundus above pubic symphysis: 12 wks
- Fundus halfway between Symptoms during pregnancy
symphysis and umbilicus: 16 wks Dietary History nutritional adequacy
Fundus at level of umbilicus: 20 wks Post-partum course
Fundus just below the ensiform cartilage: 36 Infan wellbeing
wks PHYSICAL EXAM
o The fundus will remain here until A. MATERNAL EVALUATION
onset of labor (esp in multipara) - Do a complete PE from head to foot
o Fundal height drops at time of - Obtain the ff:
lightening (in primipara) o BP actual and extent of change

3
Prenatal, tetralogy, immunizations, drugs, high risk pregnancy: EMR, JV-C & PDV

o Maternal weight actual and extent of Considered negative if


change fetal head is engaged
o Fundic height distance over 2. Cephalic prominence
abdominal wall from the top of the symphysis If on the same side as fetal
pubis to the top of the fundus small parts = head is
- At 20-34 wks AOG is equal flexed and vertex is
the presenting part
to the fundic height in cm) If on same side as fetal
- Therefore if baby is 28 back = head is extended
wks AOG and is only 25cm fundic height, consider and presenting part is the
SGA or consider wrong LMP date given face
B. VAGINAL EXAMINATION
- Vaginal exam during 1st visit
o Fetal heart tones (subsequent only if indicated)
- 110-160bpm o However at term: done weekly to
- First heard in most women bet 16- 19 weeks determine consistency, effacement,
with a standard nonamplified stethoscope and dilatation of the cervix;
- Doppler ultrasound: 10 weeks presenting part, station of PP,
- Ultrasound: 5 weeks clinical mensuration of pelvis
o C/I if with hx of vaginal bleeding
o Leopold s maneuver - Diagnosis of presentation may be obtained
accurately during labor when cervix is dilated
- Presentations identified as follows:
Leopold s Maneuver o Vertex sutures and fontanels
- Provides information on o Face portions of fetal face
o Position o Breech sacrum & ischial
tuberosities
o Presenting part o Shoulder acromion
o Extent to w/c presenting part has descended into
pelvis C. PELVIC EXAMINATION
- Performed during latter m o n t hs o f - Insert speculum into vagina and is able
to visualize abnormalities in the vagina and the
pregnanc y (3rd trimester) and during intervals cervix
between uterine contractions of labor - Usually done during initial prenatal care
- First 3 maneuvers are done with the doctor - Do systematically: Inspection -> Palpation
on the right side of the bed and facing the upper part o Inspect vulva for lesions
of the patient o Observe whether perineum is
anatomically
- 4th maneuver done facing the lower part of the intact or lacerated
mother o Vaginal discharge/bleeding
1st maneuver: Fundal Grip o Polyps
- Screening for cervico-vaginal infections
o Determines what fetal part and
occupies fundus cervical cancer is now considered routine
o Sensation of large nodular body = in prenatal care
buttocks or lower extremities at o Obtain cervical scrape
fundus in cephalic presentation o Inspect cervix for Chadwick s
sign and local esions (ex.
o Hard, freely movable, ballotable part Nabothian cysts occlusion cyst
fetal head at fundus in breech position of endocervical glands bulging
o Use fingertips of both hands beneath exocervical mucosa)
nd
2 maneuver: Umbilical Grip
o Determine location of fetal back Subsequent Assessments
o Palms are placed on each side of 1. Cervical culture for N gonorrhoeae
mother s abdomen and gentle but deep 2. Hgb electrophoresis
pressure is exerted 3. HIV titer by ELISA Western blot if HIV + by
o Hard resistant convex structure fetal ELISA
back 4. Glucose screening for DM; all pregnant
o Numerous nodulations - fetal small women should be tested
parts a. OGTT Oral Glucose Tolerance Test
3rd maneuver: Pawlic/Pawlik s Grip
st
- performed during the 1 trimester if the patient is at
o Determine what is occupying the high risk for DM
pelvic inlet Family history
o Use of thumb and fingers of one hand Age > or = to 35 y/o
o Movable, round and hard body fetal Previous birth of large baby
head not engaged
o Considered negative if lower pole of (+) sugar in urine
the fetus is fixed in the pelvis or
engaged b. GCT Glucose Challenge Test
- performed if patient is not a high risk patient
4th maneuver: Pelvic Grip - done at 24 28 weeks AOG
o Determine whether head is - Threshold value: 130 mg/dL if Glucose is greater
extended or flexed that this, do OGTT
1. Engagement occurs if both 5. MSAF at 15 -18 weeks (Usually with
hands converge from each other since presenting HCG, estriol)
part has entered the pelvis
If head is not engaged,
the cephalic prominence is
palpated

4
Prenatal, tetralogy, immunizations, drugs, high risk pregnancy: EMR, JV-C & PDV

Psychosocial Screening o I have quit smoking before


Psychosocial Issues pregnancy, and I do not currently
These are nonbiomedical factors that affect smoke
one men al and ph ical ell-being. o If smoking abstinence has already begun,
Women should be screened regardless of then reinforce her decision to quit,
social status, education level, race, or congratulate on her success, and
ethnicity. encourage her continued abstinence.
o seek barriers to care, For persistent smokers, proceed to the ffing steps:
communication obstacles, nutritional 2. ADVISE with clear, strong statements that
status, unstable housing, desire for explain the risks of continued smoking to the
pregnancy woman, fetus, and newborn
o safety concerns that include intimate 3. ASSESS he pa ien illingne o a emp
partner violence, depression, stress cessation
o use of substances such as tobacco, 4. ASSIST with pregnancy-specific, self-help
alcohol, and illicit drugs smoking cessation materials
performed on a regular basis (at least 5. ARRANGE to track smoking abstinence
once/trimester) progress at subsequent visits
o to identify important issues and
reduce adverse pregnancy Alcohol - Ethyl alcohol or ethanol
outcomes potent teratogen causes fetal syndrome
o Results to a less likely preterm or o characterized by growth restriction,
low- birthweight newborns delivery, facial abnormalities, and CNS
and decreased rate of gestational dysfunction
DM, premature rupture of Pregnant women, aged 35-44 y/o, white,
membranes, and vaginal bleeding college graduates or employed, should
Cigarette Smoking abstain from using any alcoholic beverages.
Potential teratogen
twofold risk of placenta previa, placental Illicit Drugs
abruption, and premature membrane rupture Agents may include heroin and other
likely to be preterm, have lower BW, and die opiates, cocaine, amphetamines,
of SIDS (sudden infant death syndrome) barbiturates, and marijuana
ri k for pon aneo abor ion, fe al dea h, Chronic use of large quantities is harmful to
and fetal digital anomalies the fetus
e po re in ero: ri k of a hma, infan ile o fetal-growth restriction, low
colic, childhood obesity birthweight, and drug withdrawal
soon after birth.
Pathophysiological Mechanisms Methadone maintenance for heroin abuse
fetal hypoxia o 10-30mg OD with titration PRN
o increased carboxyhemoglobin Buprenorphine
o reduced uteroplacental blood flow o Monotherapy or in combination with
direct toxic effects of nicotine and other Naloxone
compounds in smoke o Less commonly used
o Nicotine transfer is so efficient that
fetal nicotine exposure is greater Intimate Partner Violence
than that of the mother Major public health problem
o E po ed fe e : HR ariabili pattern of assaultive and coercive behaviors
due to impaired autonomic o include physical injury,
regulation psychological abuse, sexual assault,
progressive isolation, stalking,
Smoking Cessation deprivation, intimidation, and
First and subsequent prenatal visits reproductive coercion
counseling and effective intervention options Domestic Violence prevalent during
are recommended routine prenatal screening aside from
Cessation at preconception = GREATEST preeclampsia
BENEFIT Could lead to adverse perinatal outcomes
Quitting at any stage of Pregnancy = such as preterm delivery, fetal-growth
improved perinatal outcomes restriction, and perinatal death.

Fi e A of Smoking Ce a ion Domestic Violence Screening


- 15min. counseling approach Use at the first prenatal visit, again at least
1. ASK about smoking at the first and 1/trimester, and at the postpartum visit
subsequent prenatal visits. Done privately and away from family
- To improve assessment accuracy, members
patients should choose the ffing statement
that best describes her smoking status: Clinical Evaluation
o I smoke regularly now; the same as - thorough, general PE should be completed at the
before pregnancy initial prenatal encounter
o I smoke regularly now, b I e c Pelvic examination is performed as part of
down with pregnancy. I smoke every the evaluation
once in a while

5
Prenatal, tetralogy, immunizations, drugs, high risk pregnancy: EMR, JV-C & PDV

Cervix is visualized employing a speculum Risk factors include unmarried status, recent
lubricated with warm water or water-based change in sexual partner or multiple
lubricant gel concurrent partners, < 25 years of age,
- Bluish-red passive hyperemia: inner-city residence, history or presence of
characteristic but not of itself diagnostic other STD , and little or no prenatal care
nd
- Nabothian cysts prominent dilated- Test of Cure 2 test ffing treatment.
occluded cervical glands (beneath o Recommended during 3-4wks after
ectocervical mucosa) Tx completion
- The cervix is not normally dilated
except at the external os Pregnancy Risk Assessment
- Pap smear - identification cytological SUBSEQUENT PRENATAL VISITS
abnormalities 4-week intervals until 28 weeks
- Bimanual examination is completed by every 2 weeks until 36 weeks
palpation weekly until delivery
- consistency, length, and dilatation
of the cervix - complicated preganancies = 1-2weeks
- to uterine and adnexal size | bony interval visits
pelvic architecture | any vaginal or o twin pregnancies
perineal anomalies - re rn i i e er 2 k = o come
Fetal presentation determined at the latter o emphasis on prenatal nutrition and
part of pregnancy education
Lesions of the cervix, vagina, or vulva
evaluated by colposcopy, biopsy, culture, or Prenatal Surveillance
dark-field examination A e men of he mo her and he fe ell-
Perianal region - visualized with DRE as being at each return visit
required for complaints of rectal pain, - Fetus: Evaluation of the heart rate, growth,
bleeding, or mass amniotic fluid volume, and activity
- Mother: BP, weight, extent of change
Gestational Age Assessment - Symptomatic headache, nausea, vomiting,
one of the most important aspects of altered vision, abdominal pain, bleeding,
prenatal care vaginal fluid leakage, and dysuria are
Treatment for the development of pregnancy sought
complica ion dependen on fe al age - Measurement of uterine size (symphysis
Based on Uterine Size: to fundus).
o 6wks gestation small orange - In late pregnancy, vaginal examination
o 8wks pregnancy large orange provides valuable information
o 12wks - grapefruit o Confirmation of the presenting part
Crown-Rump Length (CRL) and its station
st
o Most accurate during 1 trimester o Clinical estimation of pelvic capacity
Sonographic interrogation General configuration, amniotic
o Provision of an estimated fluid volume adequacy, cervical
gestational age during the latter consistency, effacement, and
pregnancy dilatation
o With declining accuracy Fundal Height
Between 20 and 34 weeks
Laboratory Tests uterine fundus height (cm) = gestational age
Initial Blood Tests in weeks
complete blood count o used to monitor fetal growth and
blood type with Rh status determination amnionic fluid volume.
an antibody screen o distance along the abdominal wall
top of the symphysis pubis
Universal HIV testing recommended part for op of he f nd
prenatal care o bladder must be emptied before
with patient notification and right of refusal fundal measurement
refused or declined RECORDED in o if used alone, fetal growth restriction
prenatal record (FGR) can be undiagnosed
Obese patient or presence of uterine mass
All pregnant women should also be (Leiomomata) limits accuracy
screened for hepatitis B virus and syphilis o Ultrasound can be used
and with immunity to rubella at the initial visit
Fetal Heart Sounds
Routine Urine Analysis - FH Rate: 110 160bpm
Urine culture: performed for the treatment of - Doppler ultrasound 10weeks
asymptomatic bacter ria = mp oma ic - Standard nonamplified Stethoscope
UTI development 16weeks; 20weeks in majority (80%); 22wks
for all women
Cervical Infections - Because the fetus moves freely in amnionic
st
Chlamydia trachomatis screened at 1 prenatal fluid
rd
visit, with additional 3 trimester testing (high risk) site on the maternal abdomen where
Neisseria gonorrhoeae G(-) diplococci fetal heart sounds can be heard best will
vary
6
Prenatal, tetralogy, immunizations, drugs, high risk pregnancy: EMR, JV-C & PDV

Sonography - increased risks for gestational HPN


provides invaluable information regarding (preeclampsia) & DM, macrosomia,
fetal anatomy, growth, and well-being cesarean delivery
used at least once during the entire - do e rela ed o prenatal weight gain.
pregnancy - Maternal weight gain during pregnancy
only be performed when there is a valid influences birthweight
medical indication
o used with lowest possible US exposure Severe Undernutrition
setting severe nutritional deficiencies - consequence of
social, economic, or political disaster
Subsequent Laboratory Tests Dutch Hunger Winter
- Fetal aneuploidy screening 11-14wks - Intense nutritional deprivation in Netherlands
and/or 15-20wks - Rations reached 450kcal/day
- Serum screening for neural tube defects o With generalized malnutrition
15-20wks - Smith (1947), correlates the influence of
- Hct & Hgb determination + syphilis serology starvation to birth wt. during late pregnancy
(if prevalent) repeated at 28-23wks - Intense dietary deprivation = impaired brain
- ri k for HIV acquisition repeated testing development
rd
at 3 trimester
- ri k for Hep B infec ion retested at the
time of hospitalization for delivery
- D (Rh) negative and unsensitized women Barker hypothesis
Ab screening test repeated at 28-29wks + - Earl ar a ion = CNS anomalies,
anti-D Ig administration (if remained schizophrenia, and schizophrenia-spectrum
unsensitized) personality disorders
- Mid to Late pregnancy deprivation = light,
short, thin at birth
o incidence of dimini hed gl co e
Group B Streptococcal (GBS) Infection tolerance, HPN, reactive airway
- Between 35-37 AOG attain vaginal and disease, dyslipidemia & CAD
rectal GBS cultures - Concept of Fetal Programming
- Intrapartum antimicrobial prophylaxis (+) o Adult morbidity and mortality ~ fetal
cultures death

Gestational Diabetes Weight Retention after Pregnancy


- All should be screened for gestational DM - Maternal Weight Loss:
o by history, clinical factors at delivery (12lb or 5.5kg)
- routine laboratory testing (bet. 24-28wks) ensuing 2wks (9lb or 4kg)
most sensitive approach between 2wks and 6mos postpartum
(5.5lb or 2.5kg)
Selected Genetic Screening
- offered to those at increased risk based on family
history, ethnic or racial background, or age - Gain > Loss; thus, 3lb or 1.4kg of weight is
- Tay-Sachs disease Eastern European retained.
Jewish / French Canadian ancestry - gain d ring pregnanc = po par m
- -thalassemia - Mediterranean, Southeast loss
Asian, Indian, Pakistani, or African - No relationship bet. Pre-pregnancy BMI or
ancestry weight gain and weight retention
- -thalassemia - Southeast Asian or
African ancestry Recommended Dietary Allowances
- sickle-cell anemia - African, - Iron, Zinc, Selenium, Vit A, B6, C, and D =
Mediterranean, Middle Eastern, with potential toxic effects in intake >2x the
Caribbean, Latin American, or Indian RDA
descent - Excessive Vit A (>10,000 IU/day) -
- Trisomy 21 - with advanced maternal age. teratogenic

NUTRITIONAL COUNSELING Calories


Weight Gain Recommendations - Pregnancy requires 80,000kcal on the last
20wks.
st th
1 half of 20 century - Recommended increase from 100 to
- Recommended wt gain = limited to <20lb or 300kcal/day is advised during pregnancy
9kg o should not be divided equally during
- prevent gestational hypertension and fetal the course of pregnancy
macrosomia. - Used as a means of energy source and for
1970s at least 25lb or 11-12kg gain the sparing of protein metabolism, used
- prevent preterm birth and fetal-growth for fetal growth and development
restriction
prepregancy BMI basis for weight gain Protein
foc of d : lo bir h eigh ne born obe i - Used for the growth and remodeling of fetus,
placen a, er , and brea , a ell a
maternal blood volume

7
Prenatal, tetralogy, immunizations, drugs, high risk pregnancy: EMR, JV-C & PDV
nd
- 2 half of pregnancy = 1000g of CHON are larger infants with bigger head
deposited, amounting to 5-6g/day circumference
o orni hine, gl cine, a rine, and - 30mg zinc from 12-16wks AOG
proline maternal plasma level until delivery = no improvement
o gl amic acid and alanine conc. in birthweight
During pregnancy - Low-birthweight infants of mothers with zinc
- Supplied from: animal source (meat, milk, pplemen a ion = ri k of ac e diarrhea,
eggs, cheese, poultry, fish) dysentery, impetigo
- Milk and dairy products source of CHON
and calcium for pregnant or lactating women Magnesium
- Mineral deficiency during pregnancy still has
Minerals no recognized consequence.
Iron
- Iron and iodine minerals not found in diets
the supply adequate calories for weight gain. Trace Metals
- 300mg of Fe transferred to the fetus and - Copper, Selenium, Chromium and
placenta Manganese (Cu, Se, Cr, Mn) (+) roles in
- 500mg of Fe added into the maternal Hgb certain enzyme functions
mass - Selenium deficiency fatal cardiomyopathy
Nearly all is used after mid-pregnancy in young children and reproductive-aged
- At least 27mg of elemental iron (FDA: women
30mg) supplement per day given to - Oversupplementation Selenium Toxicity
pregnant women
Amount contained in most prenatal Potassium - decreased by 0.5mEq/L during
vitamins (as Fe fumarate, sulfate, midpregnancy on the maternal plasma
gluconate)
If taken daily throughout the latter half Fluoride
of pregnancy = adequate requirement - no beneficial evidence on fluoride
+ protection of preexisting Fe stores supplementation during pregnancy
Also provide for Fe requirements of - no alteration on fluoride metabolism
lactation - ingestion does not increase concentration in
- 60-100mg of elemental Fe/day for large breast milk for lactating women
women, has twin fetus, late pregnancy
supplementation, irregular Fe intake, or with Vitamins
depressed Hgb levels Folic Acid
- IDA woman responds well to oral iron salts - 400mcg = ne ral be defec a
- Iron supplementation during the first four preconceptional period
months of pregnancy is not required - Level A recommendation
Avoids risk of aggravating nausea and o 0.4 to 0.8 mg of folic acid per day
st
vomiting - 4mg/day 1 mon h before concep ion 1
- Iron intake at bedtime or empty stomach = trimester
ab opr ion & ad er e GI reac ion o recurrence risk of NTD
o Consumed as a separate
Iodine supplement, not as multivitamins.
- RDA: 220mcg
- Iodized salt and bread products Vitamin A
recommended to offset the increased fetal -
req. and maternal renal loss of iodine malformations
- Maternal hypothyroidism = cretinism with - Accutane Vit A derivative isotretinoin
neurodevelopmental defect in children o Most potent teratogen
- -carotene found in fruits and vegetables
Calcium o incidence of toxicity
- pregnant woman retains approximately 30g - Deficiency
of calcium o <
rd
- Most is deposited in the fetus late in o night blindness 3 trimester
pregnancy o ri k of ma ernal anemia and
- Represents only 2.5% of total maternal spontaneous preterm birth
calcium Vitamin B12
most of which is in bone and can - pla ma transcobalamins (carrier) =
readily be mobilized for fetal growth maternal plasma B12
- in e inal ab orp ion and progre i e - naturally only in foods of animal origin
retention throughout pregnancy o strict vege arian = B12 infan
o Profound in breast-fed infant
Zinc - Vi C inge ion = f nc ional deficienc of
- Severe zinc deficiency - poor appetite, B12
suboptimal growth, and impaired wound - Deficienc : ri k of ne ral be defec
healing o Still controversial
- RDA = 12mg during pregnancy
- 25mg zinc supplementation at Vitamin B6 Pyridoxine
midpregnancy = higher zinc plasma level =
- 2mg supplement for high risk inadequate
nutrition
8
Prenatal, tetralogy, immunizations, drugs, high risk pregnancy: EMR, JV-C & PDV

o substance abusers, adolescents, - Avoidance of high risk of falling / abdominal


and those with multifetal gestations ra ma & c ba di ing ( ri k of
- Vi B6 + do lamine = na ea and decompression sickness)
vomiting - e erci e or ph ical ac i i = re = (+)
Vitamin C outcome
- RDA = 80 to 85 mg/day o For pregnant women who are
- Most water soluble vitamins decline during having:
pregnancy but higher in the cord=blood level pregnancy-associated
Vitamin D hypertensive disorder
- Fat soluble vitamin with active metabolic preterm labor, placenta
form previa
- in e inal calci m ab orp ion & promo e severe cardiac or
bone mineralization and growth pulmonary disease
- Source: endogenous synthesis from sunlight multiple or suspected
& dietary intake growth-restricted fetuses
- Deficiency: disordered skeletal homeostasis,
congenital rickets, and fractures in the Seafood Consumption
newborn - Fi h = pro ein, a . fa , (+) omega-3 fatty
- Recommended intake = 15mcg/day (600 IU acids
per day) - Fish and shellfish = (+) trace amount of Hg
- 25-hydroxyvitamin D serum level lab test (methylmercury)
for suggestive deficiency o Shark, swordfish, king mackerel,
Pragmatic Nutritional Surveillance and tile fish
Advise the pregnant woman to eat what she o Not given to pregnant and lactating
wants in amounts she desires and salted to women
taste - Recommendation: nmt 12oz or 2 servings of
Ensure that food is amply available for canned tuna / week & nmt 6oz of albacore
socioeconomically deprived women or white tuna
Monitor weight gain - goal of approx. 25-35 - Limitation: if Hg content is unknown = nmt
lbs in women with a normal BMI 6oz / week overall fish consumption
Explore food intake by dietary recall
periodically to discover the occasional Lead Screening
nutritionally errant diet - Maternal exposure = several adverse
Give tablets of simple iron salts that provide maternal and fetal outcomes
at least 27 mg of elemental iron daily o Mother: (+) gestational HPN,
o Give folate supplementation before spontaneous abortion
and in the early weeks of pregnancy o Fetus: low birthweight, and
o Provide iodine supplementation in neurodevelopmental impairments
areas of known dietary insufficiency -
Recheck Hct or Hgb concentration at 28- - =
32wks AOG to detect significant decreases undergo chelation therapy

COMMON CONCERNS Automobile and Air Travel


Employment - Encouraged to wear properly positioned
- (+) pregnancy = 12 workweeks of unpaid three-point restraints
leave o Throughout pregnancy while on
o For birth and care of newborn child automobiles
- pregnanc complica ion ri k : o Lap portion placed under
o Ph icall demanding = ra e of abdomen and across upper thighs
preterm birth, fetal-growth o Belt snug comfortably
restriction, or gestational HPN o Shoulder belt between the breasts
o 5x risk of preeclampsia - (+) availability of airbags
o Occ pa ional fa ig e = ri k of - Air travel = properly pressurized aircraft = no
premature membrane rupture harmful effect
# of hours standing - Can safely fly up to 36wks of AOG
Intensity of physical and - Significant risk: infectious disease
mental demands acquisition, lower venous thromboembolism
Environmental stressors
- Management: adequate period of rest, Coitus
limitation of work or play, minimization of - A oidance if ri k of abor ion, placen a
physical work previa, or preterm labor
- ge a ion = coi freq enc
Exercise - Intercourse late in pregnancy = not harmful
- Not limited to pregnant women provided that - Oral-vaginal intercourse is occasionally
they do not become excessively fatigued or hazardous
risk injury - (+) fetal air embolism result of air blown
- encouraged to engage in regular, moderate- during cunnilingis at late pregnancy
intensity physical activity for >30mins daily.
- Activity = reviewed for potential risks Dental Care
- Par of prena al e amina ion enco rage
good dental hygiene

9
Prenatal, tetralogy, immunizations, drugs, high risk pregnancy: EMR, JV-C & PDV

o Periodontal disease = preterm labor - Morning sickness due to be worse in the


- Dental carries not aggravate by pregnancy morning though can continue throughout the
- Pregnancy not contraindicated to dental day.
treatment and dental radiographs - Treatment: minimization of symptoms but
seldom provides complete relief
Immunization o Frequent small meals
- Exposure to thimerosal vaccine preservative o Herbal remedy ginger
= neuropsychological disorders o Vit B6 + doxylamine
o Still under controversy o Phenothiazine or H1-receptor
- three-agent Tdap vaccine - tetanus toxoid, blocking antiemetics
reduced diphtheria toxoid, and acellular - Hyperemesis gravidarum severe
pertussis vomiting = dehydration, electrolyte and acid-
o d e o per i infec ion base disturbance, starvation ketosis
o dea h ri k for infants still
dependent on passive immunization Backache
from maternal Ab - Lower back pain excessive strain or
Vaccine is initiated at significant bending, lifting, or walking
2months of age o Reduced by squatting rather than
o 27-36 wks AOG - optimal passive bending over; using a pillow for
antibody transfer of Ab to the fetus support while sitting; high-heeled
- Influenza vaccine given at any gestation shoes
age during flu season - Progre ing ge a ion = complain
o prenatal maternal accina ion = o Prevalent on obese women or (+)
infant influenza incidence/ febrile history of low back pain\
respiratory illness (first 6 months of - Severe pain = require thorough orthopedic
life) examination
- postpartum MMR vaccine - susceptible to o pregnancy-associated osteoporosis,
rubella disc disease, vertebral
o will not cause congenital rubella osteoarthritis, or septic arthritis
syndrome even if used during - muscular spasm = acute strain or fibrositis
pregnancy - usually responds to analgesics, heat, and
o no contraindication while breast rest
feeding o Tylenol for chronic pain
o NSAID ed in hor co r e =
Biological Warfare and Vaccines fetal effects
- smallpox vaccine - live attenuated vaccinia o Muscle relaxants (cyclobenzaprine
virus (Flexeril) or baclofen) added as
o may result to abortion, stillbirth, or needed
neonatal death - ac e pain (+) ph ical herap o
o contraindicated during pregnancy or stability and strength of spine and hip
to pre-pregnant women within o (+) sacroiliac joint stabilizing support
28days of vaccination belt
o if administered inadvertently at early o Essential for the increase load of
pregnancy = not an indication for pregnancy
termination - Multimodal therapy - combination of manual
o (+) infection direct contact or chiropractic therapy, exercises, and patient
victim of bioterrorist = benefit > risk education
of vaccination o Re l o pain and disability
- Anthrax vaccine - no live bacteria
o Only for occupationally exposed Varicosities and Hemorrhoids
citizens (Veterinarians, Lab workers, - Venous leg varicosities = congenital
military personnel) predi po i ion & i h ad ancing age
o No potential fetal risk - Fac or: lo er e remi ie eno pre re,
prolonged standing
Caffeine - Symptoms: cosmetic blemishes, mild to
- heavy intake (5cups or 500mg of caffeine e ere di comfor req ire rest + feet
per da ) = abor ion ri k elevation
- moderate intake (<200mg daily) no - Vulvar varicosities co-exist with leg
reported risk varicosities
- no a ocia ion i h pre erm bir h b ih o Become massive and almost
risk for fetal growth impairment incapacitating
- limited intake = <300mg or three 5-oz cups o (+) rupture = severe blood loss
of percolated coffee o Treatment: fitted pantyhose -
lower extremity varicosities
Nausea and Vomiting foam rubber pad suspended
st
- Common complaints during 1 half of across the vulva by a belt
pregnancy (+) pressure on dilated
st nd
- Usually commence between 1 and 2 veins
mi ed men r al period 14-16wks AOG for bothersome
vulvar varicosities

10
Prenatal, tetralogy, immunizations, drugs, high risk pregnancy: EMR, JV-C & PDV

- Hemorrhoids - rectal vein varicosities due to


pel ic eno pre re Cord Blood Banking

o Pain and swelling relieved by umbilical cord blood transplantations = Tx
topical anesthetics, warm soaks and for hemopoietic cancers and various genetic
stool softening agents conditions
o External hemorrhoid thrombosis 2 types of cord blood banks
relieved by incision and clot removal o Public banks promotion of
under local analgesia allogenic donation, for use by
related or unrelated recepients
Heartburn Similar to blood product
- One of the most common complaints donation
- Gastric reflux into the lower esophagus o Private banks for storage of stem
- Result from upward displacement and cell f re a ologo e ih
compression of stomach by uterus + LES fees for initial processing and
relaxation annual storage
- Relie ed b hort meal frequency,
bending over / lying flat, antacids (Al(OH)3, Identification of High Risk Preganncy
Mg(OH)2, Mg trisilicate) A. Medical History
Asthma, cardiac disease, DM, drug and alcohol use,
Pica and Ptyalism epilepsy (on medication), family hx of genetic
- Pica craving for strange food problems (Down Syndrome, PKU),
o Pagophagia - ice hemoglobinopathy, hypertension, prior DVT or
o Amylophagia - starch pulmonary embolism, psychiatric illness, COPD,
o Geophagia - clay chronic renal disease
- Can be riggered by severe iron deficiency
- Ptyalism distressed by profuse salivation
o Stimulation after starch ingestion
B. Obstetrical History
Sleeping and Fatigue Age >35 y/o, caesarean delivery, incompetent
- earl pregnanc = (+) fa ig e, leep req iremen cervix, prior fetal structural/chromosomal
- soporific effect of progesterone abnormality, prior
st
- 1 trimester: nausea and vomiting neonatal death, prior fetal death, prior
rd
- lesser extent than 3 trimestral pregnancy preterm delivery or preterm ruptured membranes,
rd
- 3 trimester: general discomfort, urinary frequency, prior LBW (<2500g), uterine leiomyomas or
dyspnea malformation
- poor leep efficienc , a akening, age
4 (deep) and rapid-eye movement sleep C. Initial Lab Tests
- short sleep duration, (+) snore, HIV
restless leg syndrome, (+) sleep disturbance Rh positive
- Diphenhydramine (Benadryl) can be helpful Initial examination condylomata

Leukorrhea
- aginal di charge d ring pregnanc
- Most instances are not pathologic
- e rogen = m c ecre ion b cer ical gland
- Contributing factor
Speculu
Dischar
m Dx Tx
ge
Finding
Foamy/f
Hanging Metroni
Tricho rothy,
Strawbe drop dazole
monas yellow/g st
rry NSS (after 1
vaginal reen,
cervix Trichom trimeste
is leucorrh
onas r)
ea
Reddish Flagellat
Candid
vaginal e
a Cheesy Nystatin
- Thayer
albican white, /
pruritus/ Martin
s/ curd- Micona
burning Agar
Chlamy like zole
sensatio Pseudo
dia
n hyphae
Clue
Grayish, cells
Gardin rotten Hanging
ella fishy drop Metroni
vaginal odor w/ 10% dazole
is pH < KOH (+
4.5 amine
test)

11
Drugs, Immunizations and High Risk

Drugs & Immunizations

Pharmacokinetics in Pregnancy
o Ideal drug for treatment
Efficacious for maternal indication
Minimally transported through the placenta & in event of passage, exerts minimal effect
on the fetus
Drug characteristics that influence tolerability & adherence to dosage & dosing schedule
Must not exacerbate conditions common in pregnancy: emesis, GIT changes, glucose
intolerance, hypertension
o Therapeutic consideration: Effects on fetus > Maternal factors
o Physiologic changes in pregnancy significantly affect pharmacokinetics in pregnant patient

Physiologic Change Effect on PK Examples

GIT Variable bioavailability of oral Most oral preparations


GIT motility, gastric pH, nausea preparations
& vomiting
Respiratory Increased alveolar uptake of inhaled Inhaled anesthetics
Hyperventilation, TV Pulmonary drugs
blood volume
Cardiovascular Increased volume of distribution
: Cardiac output (40%), plasma
volume (50%) & total blood volume
(40%)
Body space Further increase in volume of
: Maternal aqueous, fatty tissue distribution
space (peak at 10-30wks), body
water (ECF: 40% maternal, 60%
feto-placental)
Metabolic Increased free-drug fractions of Diazepam, phenytoin, valproic acid
rd
: Albumin (25%) protein-bound drugs specially in 3
: Steroid hormones competing for trimester
protein binding sites
Serum Progesterone Increased cytochrome metabolism Phenytoin, carbamazepine, valproic
of drugs acid
Serum Estrogen Decreased cytochrome oxidase Theophylline & caffeine
metabolism of some drugs
Renal Increased renal excretion & Lithium, digoxin, ampicillin
Renal blood flow (50%) leading to creatinine clearance (doubled)
GFR (50%)
Net effect expected decrease in steady state concentration if a normal dose is
administered, higher dose will be needed
Important in theory, but often not clinically significant, exceptions
Drugs exclusively eliminated via renal route
o Lithium & Digoxin
o Increase in renal clearance decreased serum concentration
o Narrow therapeutic range serum concentration monitoring
Anti-epileptics
o Carbamazepine, valproic acid & phenytoin
o Increase in: hepatic metabolism, renal clearance & plasma volume +
Decreased protein binding decrease in effective serum concentration
Feto-placental Pharmacokinetics
o Drugs reaching the fetus are almost always administered to the mother
o Maternal administration Placenta Fetal serum Placenta Maternal excretion
o Cotyledons (Maternal lobes)
Raised areas on the basal surface of the placenta intimately related to several fetal
cotyledons
Allows maternal circulation to be superimposed onto the fetus
Passage of solutes (require passing through the syncytiotrophoblast either
Directly through its cytoplasm more common
Via a network of specific transporters less common
o 3 Compartments to cross: Maternal blood trophoblastic cytoplasm fetal circulation
Simple diffusion major transport mechanism
Ra e of ran por i dic a ed b ol e
Molecular weight
Degree of ionization

1
Drugs, Immunizations and High Risk

Protein-binding
Relative concentration across the compartments
Factors involved in drug transfer across the placenta
Physiochemical properties of drug: highly lipid soluble, non-ionized, low
molecular weight (<200daltons) & low protein-binding easily cross the placental
barrier
Transfer of flow-limited drugs is affected by maternal serum drug concentration &
placental blood flow: Higher maternal drug concentration create a greater
concentration gradient causing rapid placental transport
Compounds that alter blood flow also alter maternal drug disposition &
consequently placental transfer
Placental metabolism (dealkylation, hydroxylation, demethylation) influence drug
ran por o a le er degree han ma ernal me aboli m infl ence
Fetal metabolism plays a part but may change during pregnancy

Fetal Pharmacokinetics also play a role in determining the net effect of drugs on the fetus
o Maternal blood flow to the placenta
Increases gradually during gestation
10 weeks 50mL/min
38 weeks 600mL/min (peak)
Increases the fetal drug exposure as pregnancy progresses
o Protein-binding capacity
Fetal plasma (vs maternal)
Albumin where acidic drugs bind to, higher by 15%
a1-Acid Glycoprotein where basic drugs bind to, lower by 37%
Protein-binding capacity lower
Leads to a greater free drug fraction that enter fetal circulation
Propanolol & lidocane basic drugs
Ampicillin & benzylpenicillin drugs with low affinity for fetal proteins
Less protein-binding, less a1GP More free drug in circulation Greater risk for
toxicity
o Ionization
pH: Fetal plasma & amniotic fluid (more acidic) vs maternal blood
Ionization is favored ion-trapping of basic drugs in fetal compartment greater risk for
toxicity
o Hepatic metabolism
st
Fetal liver has cytochrome oxidase responsible for 1 pass effect in the fetus, less
metabolizing capacity vs maternal
Modulated by ductus venosus shunt & may vary by 30-70%
Maternal estrogen Decrease activity of cytochrome oxidase Decrease hepatic
metabolism of drugs Increase circulating drug concentration greater risk for toxicity
o Renal clearance
Fetal kidney is immature low GFR
GFR increases with AOG but peaks only at 2-4mL/min at term because it receives only
3% of CO (vs 25% in adults)
Cation secretion high, efficient clearance of cimetidine
Anionic drug secretion very low, inefficient clearance of penicillin
o Reabsorption
Fetal urine Amniotic fluid May be swallowed by the fetus
Renally excreted drugs & metabolites may be reabsorbed

Generalities on Pharmacokinetics (Maternal, placental, fetal)


o Drugs that are large, highly protein-bound, ionized in maternal plasma & poorly absorbed into the
maternal bloodstream
Rarely cross the placenta
Usually confined in the maternal compartment
Exhibit minimal effect on the fetus
o Drugs that cross the placenta exhibit greater toxicity for the fetus than for the mother due to
Decreased protein binding
Ion-trapping
Poor metabolism
Poor excretion

2
Drugs, Immunizations and High Risk

Guidelines for Prescribing & Counseling in Pregnancy


o Generalities
Rule of thumb:
st
Drug intake in the pregnant patient is generally avoided especially in the 1
trimester when fetus exhibits highest risk for congenital malformations
Consider non-pharmacologic treatments
If absolutely necessary,
Preference must be given to drugs with proven safety: Classes A & B
Monotherapy is preferred
Drug combinations agents must be introduced one at a time
Exposure of fetus to agent must be kept to a minimum: Lowest possible dose &
shortest possible effective treatment duration
Systemic administration should be avoided (inhalation & topical vs oral & IV)
Benefits >>> Risk to fetus & mother
o Bacterial infections
Penicillins (B) preferred agents & probably the safest antimicrobial in pregnancy
Erythromycin (B) for penicillin-allergic patients
Use with caution: Cephalosporins (B), Chloramphenicol (B) & Metronidazole (C)
o Tuberculosis
Rifampicin, Isoniazid, Ethambutol no increased risk for congenital malformation
Use drugs with caution
o Colds, cough
Always aim for symptomatic relief with fluids & rest
Antihistamines use with relative safety
Chlorpheniramine (B)
Tripolidine (D)
Diphenhydramine (C) more rapid-onset of rebound congestion in pregnancy;
use for no longer than 2-4days
Pseudoephedrine (C) generally safe
Cough suppressants
Guaifenesin (C)
Dextromethorphan (C)
Avoid preparations with alcohol & iodine
o Nausea & vomiting
Treated conservatively when effective: Rest, small frequent meals & acupressure
st
Emetrol 1 line
Vitamin B6 (A) + Doxylamine (B)
Use with caution: Promethazine (C) & Metaclopramide (C)
o Pain Relief
Paracetamol (B) pain reliever & antipyretic of choice
Ibuprofen (B)
Local anesthetics (Xylocaine) useful but do not combine with Epinephrine
Narcotics for sever pain Codeine, Demerol & Morphine
Mother addictive potential
Fetus respiratory depression
o Diarrhea
Kaolin & Pectin (B) anti-diarrheal of choice, not absorbed systematically
Loperamide (B) most probably safe
o Asthma
Treatment is same with non-pregnant patient
Theophylline (B)
Salbutamol (B) via aerosol
Steroids (B) & Leukotrienes (B) safe for pregnancy
o Epilepsy
Emphasize risk for malformations (5% with epipleptics, 3% in others)
Avoid drug combinations & use therapeutic drug monitoring
Therapy is maximized before pregnancy & may consider slowly withdrawing if patient has
been seizure-free for 2-3years
No justification for shifting from a teratogen to one with no available data
Phenobarbital (B) no increased frequency in minor/major defects
Carbamazepine (D) teratogenic potential is influenced by genetics (epoxide
hydroxylase levels)
Valproic Acid (D) 1-2% risk of spina bifida, associated with minor craniofacial
deformities
o HPN
Methyldopa (C) most widely used, unparalleled safety record in pregnancy
b-Blockers, except Atenolol (D) non-teratogenic but may cause growth restriction
nd rd
Hydralazine no adverse fetal effects; used for the 2 & 3 trimester

3
Drugs, Immunizations and High Risk

NaNitroprusside readily crosses the placenta & may cause accumulation of cyanide, no
adequate data
o Hyperthyroidism
Propylthiouracil used more often than carbamazepine, less lipid-soluble & more protein-
bound, this transported less well to fetus & breast milk

Immunization During Pregnancy


o Generalities
Vaccines that generate maternal immune response may also become protective to the
fetus when maternal antibodies pass through the placenta
Detrimental when sufficient amount of maternal antibodies are transmitted as to interfere
with the development of natural antibodies in the fetus
Maternal reactions to vaccination may adversely affect fetal support & development
Fever
Hypersensitivity
Risks are theoretical no available clinical evidence; Benefits >>>> Risk
Live vaccines carry greater theoretical risk of vertical transmission & are
contraindicated
Measles
Mumps
Rubella
o Tetanus-Diphtheria Vaccine (Td)
Safe in pregnancy - toxoid
Booster shots for previously immunized: okay
Unimmunized must be given the complete 3-dose series
nd
Preferred schedule: after 2 trimester with each dose given 4 weeks apart
ONLY VACCINE routinely indicated for all susceptible pregnant patients
o Hepatitis A Vaccine
Not established if safe/not
Killed vaccine - theoretical risk is low but safety is not determined
May be indicated for high-risk patients
o Hepatitis B Vaccine
Safe in pregnancy, no known detrimental effects on the fetus
Indicated in patients: >1 sexual partners in the last 6months, concurrent STD, IV drug
use, HBsAg-positive partner
o Human Papillomavirus Vaccine
Not recommended in pregnancy
Has been related to some adverse outcomes in pregnancy but data is limited
o Influenza Vaccine (Inactivated)
Safe in pregnancy, no adverse fetal outcomes reported
Recommended during influenza season, pregnant who become infected are at increased
risk for severe complications
o Measles, Mumps & Rubella Vaccine
Live vaccines - should not be administered in pregnant patients
Vaccinated women should not be advised to get pregnant for 28 days after administration
Congenital Rubella Syndrome in fetus (no documented case yet)
o Pneumococcal Vaccine
Not established if safe/not
No adverse effects reported
o Polio Vaccine
Safe but avoided
No adverse effects reported but avoided due to theoretical risks
High-risk patients may be considered for vaccination
o Tetanus-Diphtheria-Pertussis Vaccine
Safe in pregnancy
Maternal pertussis Abs may be protective to the infant in early life (preferred over tetanus
toxoid alone)
o Varicella Vaccine
Not safe in pregnancy - unknown effects in fetus
Avoided entirely in pregnancy
Lower virulence than the wild type
o BCG - Not recommended; No Harmful effects reported
o Typhoid Vaccine - Not established; No available data
o Rabies Vaccine
Safe as post-exposure prophylaxis
Consequences of rabies >>> Risk of vaccination

4
Drugs, Immunizations and High Risk

5
Drugs, Immunizations and High Risk

Identification of High-Risk Pregnancy

Maternal Age
Less than 18
More than 30 (Nullipara) or 35 (Multipara)
Advanced age is inimical to the fetus
Nearly 2-fold greater risk of fetal death
Maternal Height
Small women
o Birth trauma
o Cesarean section
o Congenital anomalies
Less than 5ft contracted pelvis
Weight: Obesity increased fetal growth, hypertensive disorders, gestational diabetes, increased need for
cesarean section

Social Factors
Smoking IUGR, mean weight non-smokers is 3148g vs 2982g for smokers (165g deficit)
Potential teratogen
2-fold risk of placenta previa, placental abruption, and premature membrane rupture
Likely to be preterm, have lower BW, and die of SIDS (sudden infant death syndrome)
Ri k for pon aneo abor ion, fe al dea h, and fe al digi al anomalie
E po re in ero: ri k of a hma, infan ile colic, childhood obe i
o Low birthweight infants, premature labor, abruptio placenta, bleeding, premature rupture of
membranes
o Effects of smoking
CO inactivates fetal and maternal Hgb
Vasoconstrictor effect of nicotine = induce placental abrution
Reduced appetite = reduced caloric intake
Decreased maternal plasma volume
Unexplained predisposition to ill effects of nicotine that persists even after quitting smoking
o Fetal hypoxia
increased carboxyhemoglobin
reduced uteroplacental blood flow
o Direct toxic effects of nicotine and other compounds in smoke
Nicotine transfer is so efficient that fetal nicotine exposure is greater than that of the mother
o E po ed fe e : HR ariabili d e o impaired a onomic reg la ion
Cessation
o First and subsequent prenatal visits
counseling and effective intervention options are recommended
Cessation at preconception = GREATEST BENEFIT
Quitting at any stage of Pregnancy = improved perinatal outcomes
o Fi e A of Smoking Ce a ion: 15min. co n eling approach
ASK about smoking at the first and subsequent prenatal visits.
To improve assessment accuracy, patients should choose the ffing statement that
best describes her smoking status:
o I smoke regularly now; the same as before pregnancy
o I moke reg larl no , b I e c do n i h pregnanc . I moke e er once
in a while
o I have quit smoking before pregnancy, and I do not currently smoke
If smoking abstinence has already begun, then reinforce her decision to quit,
congratulate on her success, and encourage her continued abstinence.
o (For persistent smokers, proceed to the ffing steps)
ADVISE with clear, strong statements that explain the risks of continued smoking to the
woman, fetus, and newborn
ASSESS he pa ien illingne o a emp ce a ion
ASSIST with pregnancy-specific, self-help smoking cessation materials
ARRANGE to track smoking abstinence progress at subsequent visits
Drugs
Illicit drug use - intrauterine & fetal distress (opium, barbiturates, amphetamines, methadone, heroin and other
opiates, cocaine, amphetamines, barbiturates, and marijuana)
Chronic use of large quantities is harmful to the fetus: fetal-growth restriction, low birthweight,
and drug withdrawal soon after birth.
Methadone maintenance for heroin abuse; 10-30mg OD with titration PRN
Buprenorphine - monotherapy or in combination with Naloxone; Less commonly used
Antibiotics little harmful effect
o Tetracycline brownish discoloration of deciduous teeth & premature cessation of long bone growth
6
Drugs, Immunizations and High Risk

o Chloramphenicol cardiac fail re in neona e (Gra ndrome) if gi en la e in pregnanc


o Streptomycin congenital deafness, effects auditory nerve

Alcohol (Ethanol)
Potent teratogen - causes fetal syndrome
o characterized by growth restriction, facial abnormalities, and CNS dysfunction
Pregnant women, aged 35-44 y/o, white, college graduates or employed, should abstain from using any
alcoholic beverages.
Readily crosses the placenta
Moderate use no proven pathologic changes
Delirium tremens
o Seen in affected newborns
o Newborn is depressed at birth soon becomes extremely hyperactive with sweating tremors &
episodes of twitching of the face & extremities
Chronic use fetal craniofacial, limb & CV defects, prenatal & postnatal growth restriction
Heavy drinking (5 or 6 drinks of wine, beer or distilled spirits a day) during pregnancy can cause fetal alcohol
syndrome
o Undersized infants
o Mentally deficient
o Multiple deformities affecting head, face, limbs, heart, CNS
Even moderate consumption is related to a variety of problems

Intimate Partner Violence


Major public health problem
Pattern of assaultive and coercive behaviors - include physical injury, psychological abuse, sexual assault,
progressive isolation, stalking, deprivation, intimidation, and reproductive coercion
Domestic Violence prevalent during routine prenatal screening aside from preeclampsia. SCREENING;
o Use at the first prenatal visit, again at least 1/trimester, and at the postpartum visit
o Done privately and away from family members
Could lead to adverse perinatal outcomes such as preterm delivery, fetal-growth restriction, and perinatal
death.

Obstetrical History (Multiparity)


Incidence of Abruptio placenta, placenta previa, postpartum hemorrhage, uterine rupture, twinning,
dysfunctional labor & congenital anomalies increase with parity
Caesarean delivery, incompetent cervix, prior fetal structural/chromosomal abnormality, prior neonatal death,
prior fetal death, prior preterm delivery or preterm ruptured membranes, prior LBW (<2500g), uterine
leiomyomas or malformation
Premature Rupture of Membranes (PROM)
Rupture of fetal membranes prior to onset of labor
3 clinical significance
o Presenting part is not fixed in the pelvis possibility of cord prolapse & subsequent compression is
increased
o Labor is quite likely to occur
o Intrauterine infection
Management:
o Immediate delivery if there is evidence of
Chorioamnionitis
Vaginal bleeding
Fetal distress
o Induce labor with IV oxytocin/prostaglandin
Intrauterine Fetal Growth Restriction (IUGR)
th
Fetus is less than the 10 percentile if weight for AOG curve
Maternal Risk Factors - Diagnosis begins with identification of risk factors
Social: Poor weight in pregnancy, smoking, poor socio-economic status
Obstetrical: Previous IUGR infant, repeated abortions, stillbirths & neonatal death
Medical: Hypertension, renal disease, multiple UTIs, liver disease, cardiac disease,
hemoglobinopathies, thrombophilias
Screening methods: Abdominal palpation, fundic height, amniotic fluid volume estimate
Diagnosis
Confirmed by Ultrasound (BPD, femur length, HC, abdominal diameter & circumference)
Reduced abdominal circumference most sensitive biometric measure
Doppler velocity wave form analysis of fetal vessels best method in evaluating IUGR

Reduction of IUGR Incidence


st nd
1. Prophylaxis with acetylsalicylic acid started on 1 /2 trimester
2. Heparin before conception
Post-term pregnancy
7
Drugs, Immunizations and High Risk

Dysmaturity
Completed >42 weeks AOG
Impairment of nutritional supply
Parchment-like skin
Reduced weight - subcutaneous tissue scaling
Complications
Sharp rise in both fetal & neonatal mortality after 42 weeks AOG
Fetal injury from fetopelvic disproportion
Asphyxial damage from fetal distress with increased risk of mental subnormality &
neurologic deficit
Pre-term birth
Delivery before 37weeks AOG
Accounts for 2/3 of infant death
Administration of corticosteroid
To mothers in preterm labor
Between 24-34 weeks
Reduces neonatal mortality by reducing RDS, Intraventricular Hemorrhage (IVH) & Necrotizing
Enterocolitis (NEC)
Fetal Macrosomnia
Excessive-sized infants are born to multiparas (Gestational/overt diabetes, obesity)
Complications with vaginal delivery
o Asphyxia
o Fetal stress & trauma
Mental subnormality
Clavicular fracture
Brachial plexus injury
o Vaginal lacerations/hematoma & pelvic relaxation
Multiple pregnancies
Twinning occurs nearly 2% of gestations
o Contributes to 15% of infants weighing less than 2.5kg (half of which is preterm)
o Account for undergrowth neonates
Maternal complications
o Pre-eclampsia, eclampsia 3x more frequent
o Premature labor PROM
st
o Premature separation of placenta after delivery of 1 of the twins
o Hemorrhage & trauma
Fetal complications
o Fetal death due to abnormal fetal/placental development
o Circulatory competition
o Umbilical cord accidents
o Abruptio placenta/cord prolapse asphyxia CNS injury: Cerebral palsy, mental retardation
o Twin-twin transfusion anemia in one, plethora in the other
o Double ovum twins FARE BETTER than single ovum twins
Hydramnios & Oligohydramnios
o AFI by ultrasound is critical in antenatal surveillance
o Hydramnios AFI > 25cm LGA, Congenital abnormalities
o Oligohydramnios AFI < 5cm poor perinatal outcome

UTI
Major cause of premature delivery, fetal death & maternal mortality
Multiparas > Primigravidas
Bacteriuria Acute pyelonephritis
Outlook is good if treated

DM - prematurity is a problem because of


Spontaneous premature labor
Tendency to deliver early delivery prior to term is induced in poorly controlled diabetic gravidas to avoid fetal
death in utero
Thyroid Disorders
Hypothyroidism: Stillbirths
Hyperthyroidism: LBW infants
Medical History
Asthma, cardiac disease, DM, drug and alcohol use, epilepsy (on medication), family hx of genetic problems
(Down Syndrome, PKU), hemoglobinopathy, hypertension, prior DVT or pulmonary embolism, psychiatric
illness, COPD, chronic renal disease
Initial Lab Tests
HIV, Rh positive, Initial examination condylomata

8
Drugs, Immunizations and High Risk

Indications for Antepartum Fetal Monitoring Postnatal motor & intellectual impairment
Premature delivery
Tests Notes

CBC depends on the AOG/trimester - to determine hemtologic status


st
1 trimester 11g as normal - to r/o anemia
nd
2 trimester 10.5g as normal
rd
3 trimester 11g
* anything below is Abnormal

Urinalysis, Urine culture & sensitivity - to evaluate for UTI and renal function
- to check for asymptomatic bacteremia

Blood group, Rh - to determine blood type, Rh status and risk of isoimmunization

Serologic test for syphilis (RPR, - to detect previous or currents infections


VDRL) - if positive, do specific treponemal tests (FTA-ABS and MHA-TP)

Hep B surface Ag Do at 3rd trimester for cost- effectiveness


If positive:
- Use double glove delivery
- Give baby immunoglobulin
and Hep B vaccine immediately after delivery

Rubella Titer Approx 85% of mothers have evidence of prior infection


If px is seronegative, special precautions needed Vaccination is then
required postpartum

Cervical cytology (Pap Smear) To screen for cervical dysplasia/cancer


Indication: hx of sexual contact of at least 3 years (because you want to
detect the pre-cancer stage)
- Repeat every year
Patients at risk for uteroplacental insufficiency: Congenital abnormalities
Prolonged Pregnancy Need for specific therapy
DM
HPN
Previous stillbirth
Suspected IUGR
Advanced maternal age
Multiple gestations with discordant growth
Anti-phospholipid syndrome
Tests suggest fetal compromise
Suspected IUGR
Decreased fetal movement
Oligohydramnios
Routine antepartum surveillance

Obstetric conditions & management that may be


influenced by antepartum testing

Preterm delivery
Route of delivery
Bed rest
Observation
Drug therapy
Operative intervention in labor
Neonatal intensive care
Termination of pregnancy for a congenital
anomaly
Aspects of fetal condition that might be reduced
by antepartum testing

Perinatal death
IUGR
Non-reassuring fetal status (Intrapartum)
Neonatal asphyxia

9
Ultrasound

Technology

- Real-time image on the ultrasound screen is produced by sound waves reflected back from fluid and
tissue interfaces of the fetus, amnionic fluid, and placenta.
- Sector array transducers (w/ groups of piezoelectric crystals) working simultaneously in arrays
electrical energy into sound waves (synchronized pulses)
- Dense tissue such as bone produces high-velocity reflected waves = as bright echoes - hyperechoic
- Fluid generates few reflected waves and appears dark or anechoic.
- Sound waves traveling at a frequency above 20,000 hertz (cycles per second).
- Higher-frequency transducers = better image resolution
- Lower frequencies = penetrate tissue more effectively
nd
- 2 trimester, a 4- to 6-megahertz abdominal transducer is often in close enough proximity to the fetus
rd
- 3 trimester - lower frequency 2- to 5-megahertz transducer may be needed for penetration, but can lead
to compromised resolution. This explains when imaging obese patients and why low-frequency
transducers are needed to reach the fetus through maternal tissues.
- Early pregnancy, a 5- to 10-megahertz vaginal transducer may provide excellent resolution, because the
early fetus is close to the transducer

Fetal Safety

Sonography

valid medical indication, using the lowest possible exposure setting to gain necessary information the
ALARA principle as low as reasonably achievable.
no causal relationship has been demonstrated between diagnostic ultrasound and recognized adverse
effects in human pregnancy
required to display two indices: the thermal index and the mechanical index.
o Thermal index
measure of the relative probability that the examination may raise the temperature,
potentially enough to induce injury.
The potential for temperature elevation is higher with longer examination time and is
greater near bone than in soft tissue.
higher with pulsed Doppler applications than with routine B-mode scanning.
First trimester, if pulsed Doppler is needed for a clinical indication, the thermal index
ho ld be 1.0, and e po re ime ho ld be kep a hor a po ible, all no
longer than 5 to 10 minutes
To document the embryonic or fetal heart rate, M-mode imaging should be used instead
of pulsed Doppler imaging

*Also, theoretical risks are greater during organogenesis than later in gestation.

o Mechanical index
measure of likelihood of adverse effects related to rarefractional pressure, such as
cavitation which is relevant only in tissues that contain air.
Microbubble ultrasound contrast agents are not used in pregnancy for this reason.
No adverse effects have been reported because fetuses cannot contain gas bodies
U e of onograph for an nonmedical p rpo e, ch a keep ake fe al imaging, i
considered contrary to responsible medical practice and is not condoned by the Food and
Drug Administration

Operator Safety

Musculoskeletal discomfort
Main risk factors for injury during transabdominal ultrasound are
o awkward posture,
o sustained static forces
o various pinch grips while maneuvering the transducer
o maternal habitus
Guidelines to avert injury
1. Position the patient on the table close to you elbow is close to your body (less than 30 degrees
shoulder abduction, keeping your thumb facing up)
2. Adjust the table or chair height so that your forearm is parallel to the floor
3. If seated, use a chair with back support, support your feet, and keep ankles in neutral position. Do not
lean toward the patient or monitor.
4. Face the monitor squarely and position it so that it is viewed at a neutral angle, such as 15 degrees
downward.
5. Avoid reaching, bending, or twisting while scanning.
6. Frequent breaks may avoid muscle strain. Stretching and strengthening exercises can be helpful.

10
First Trimester Sonography

Early pregnancy can be evaluated using transabdominal or transvaginal sonography, or both


The components listed in Table 10-2 should be assessed
The crown-rump length (CRL) is the most accurate biometric predictor of gestational age
o The CRL should be obtained in a sagittal plane and include neither the yolk sac nor a limb bud
o If carefully performed, it has a variance of only 3 to 5 days
o Correlation with real age at 1st trimester: 
+/- 3 5 days
At 2nd trimester: +/- 7 days
At 3rd trimester: +/- 21 days
First-trimester sonography can reliably diagnose
o anembryonic gestation
o embryonic demise
o ectopic pregnancy
o gestational trophoblastic disease
Multifetal gestation can be identified early, and this is the optimal time to determine chorionicity
Ideal time to evaluate the uterus, adnexa, and cul-de-sac (pouch of peritoneum behind the uterus which
becomes filled with blood when an ectopic pregnancy ruptures)
Cervical length and the relationship of the placenta to the cervical os are best evaluated in the second
trimester
An intrauterine gestational sac is reliably visualized with transvaginal sonography by 5 weeks, and an
embryo with cardiac activity by 6 weeks
The embryo should be visible transvaginally once the mean sac diameter has reached 20 mm otherwise
the gestation is anembryonic
Cardiac motion is usually visible with transvaginal imaging when the embryo length has reached 5 mm
If an embryo less than 7 mm is not identified to have cardiac activity, a subsequent examination is
recommended in 1 week
Demise is diagnosed if the embryo has reached 10 mm without cardiac motion, taking into consideration
the standard error of the ultrasound measurements

Nuchal Translucency (NT)


o a component of first-trimester
aneuploidy screening, has had a major
impact on the number of pregnancies
receiving late first-trimester ultrasound
examination
o represents the maximum thickness of
the subcutaneous translucent area
between the skin and soft tissue
overlying the fetal spine at the back of
the neck
o measured in the sagittal plane
between 11 and 14 weeks using
precise criteria > or = to 3mm is
abnormal
o Screening for trisomy 18&21
o Check for fetal breathing, movement,
and tone -Tone is measured as closing
of fetal hands
o When the nuchal translucency is
increased, the risk for fetal aneuploidy
and various structural anomalies
including heart defects is significantly
elevated
o Aneuploidy screening - nuchal
11
translucency measurement in
conjunction with assessment of
maternal serum human chorionic
gonadotropin and pregnancy-
associated plasma protein A levels

First-Trimester Fetal Anomaly Detection


Assessment for selected fetal abnormalities in an at-risk pregnancy is another indication for first-trimester
sonography
Anatomy visible at 11 to 14 weeks, to coincide with sonography performed as part of aneuploidy
screening
Wi h c rren echnolog , i i no reali ic o e pec ha all major abnormali ie de ec able in he econd
trimester may be visualized in the first trimester
Identification varies considerably according to the specific abnormality. For example, reported detection
rates are extremely high for anencephaly, alobar holoprosencephaly, and ventral wall defects. However,
only one-third of major cardiac anomalies have been identified, with no detected cases of microcephaly,
agenesis of the corpus callosum, cerebellar abnormalities, congenital pulmonary airway malformations, or
bowel obstruction
Thus, as first-trimester sonography is unreliable for detection of many major abnormalities, it should not
replace second-trimester anatomical evaluation

Second- and Third-Trimester Sonography

There are three types of examinations: standard, specialized, and limited


1. Standard sonographic
The fetal anatomical structures that should be evaluated during the examination may be adequately
assessed after approximately 18 weeks
When examining twins or other multiples, documentation also includes the number of chorions and
amnions, comparison of fetal sizes, estimation of amnionic fluid volume within each sac, and fetal sex
determination
2. There are several types of specialized examinations.
The targeted examination is a detailed anatomical survey performed when an abnormality is suspected
on the basis of history, screening test result, or abnormal findings from a standard examination
A targeted examination is performed and interpreted by an experienced operator. It includes the
anatomical structures listed in Table 10-5 (OPA! Kalma! Nandito yan. See chart below), along with
additional views of the brain and cranium, neck, profile, lungs and diaphragm, cardiac anatomy, liver,
shape and curvature of the spine, hands and feet, and any placental abnormalities
The physician performing the examination further determines whether other examination components will
be needed on a case-by-case basis
Other specialized examinations include fetal echocardiography and Doppler evaluation, biophysical
profile and additional biometric measurements.

12
3. A limited examination
performed to address a specific clinical
question (Examples include amnionic fluid
volume assessment, placental location, or
evaluation of fetal presentation or viability)
In most cases, a limited examination is appropriate only
when a prior standard or targeted examination has
previously been performed

Fetal Biometry
Equipment software derives the estimated gestational age from the crown-rump length.
Formulas are similarly used to calculate estimated gestational age and fetal weight from
measurements of the biparietal diameter, head and abdominal circumference, and femur length
The estimates are most accurate when multiple parameters are used and when nomograms derived
from fetuses of similar ethnic or racial background living at similar altitude are selected
Even the best models may over- or underestimate fetal weight by as much as 15 percent
Various nomograms for other fetal structures, including the cerebellum diameter, ear length,
interocular and binocular distances, thoracic circumference, and kidney, long bones, and feet lengths,
may be used to address specific questions regarding organ system abnormalities or syndromes

In the second trimester


Use of ultrasound: Check for anatomic disorders
Enough amniotic fluid can be found to serve as contrast which will allow easier visualization
Best time to see congenital anomalies
Biparietal diameter (BPD) most accurately reflects the gestational age, with a variation of 7 to 10
days.
measured in the transthalamic view, at the level of the thalami and cavum septum pellucidum (CSP),
from the outer edge of the skull in the near field to the inner edge of the skull in the far field
The head circumference (HC) is also measured in the transthalamic view, either by placing an ellipse
around the outer edge of the skull, or by measuring the occipital-frontal diameter (OFD) and
calculating the circumference from the BPD and OFD.
The cephalic index, which is the BPD divided by the OFD, is normally approximately 70 to 86 percent.
If the head shape is flattened dolichocephaly, or rounded brachycephaly, the HC is more reliable
than the BPD. Dolichocephaly and brachycephaly may be normal variants or may be secondary to
positional changes or oligohydramnios. However, dolichocephaly can occur with neural-tube defects,
and brachycephaly may be seen in fetuses with Down syndrome
Whenever the skull shape is abnormal, craniosynostosis and other craniofacial abnormalities are a
consideration
The femur length (FL) correlates well with both BPD and gestational age. It is measured with the
beam perpendicular to the long axis of the shaft, excluding the epiphysis. For gestational age
estimation, it has a variation of 7 to 11 days in the second trimester
A mildly foreshortened femur one that is 90 percent or less than that expected for gestational age
has been used as a minor marker for Down syndrome.
A femur length that is dramatically foreshortened prompts an evaluation for a skeletal dysplasia
In general, the normal range for the FL to abdominal circumference (AC) ratio is 20 to 24 percent. A
FL/AC < 16 percent suggests a lethal skeletal dysplasia, particularly if other skeletal abnormalities are
present
The abdominal circumference has the greatest variation, up to 2 to 3 weeks, for gestational age
estimation.
The AC is measured around the outer border of the skin. This is a transverse image at the level of the
stomach and the confluence of the umbilical vein with the portal sinus
Of the biometric parameters, AC is most affected by fetal growth.
a small abdominal circumference has been used as an early indicator of fetal-growth restriction
Variability of the estimated gestational age and of fetal weight increases with advancing gestation.
Individual measurements are least accurate in the third trimester. Although estimates are improved by
averaging multiple parameters, if one parameter differs significantly from the others, consideration is
given to excluding it from the calculation. The outlier could result from poor visibility, but it could also
indicate a fetal abnormality or growth problem.
Menstrual dates are generally considered confirmed if an estimated gestational age (EGA) based on
a sonographic first-trimester crown-rump length is within 1 week or if the EGA from biometry at 14 to
20 weeks is within 10 days

13
In the third trimester
the accuracy of sonography is only within 3 to 4 weeks. Sonographic evaluation performed to monitor
fetal growth should typically be performed at least 2 to 4 weeks after a prior examination

Use of ultrasound: For establishing the functions of the fetus
Growth result of function (SGA, LGA, etc)
Surveillance high-risk pregnancies Have to make a decision whether to keep baby in utero or
deliver him/her 

Can also do biometery, biophysical scoring, Doppler velocimetry
Reflects oxygenation of fetus

Manifest if well oxygenated

Brain sparing in the fetus: Blood is well distributed during critical times to the heart, brain, etc. if baby
manifests with a score of ZERO tone, breathing or movement = baby is very hypoxic!
Remember that you only do surveillance at a time when the baby can survive ex-utero. Do not perform
antenatal surveillance during the first trimester because baby is not yet able to survive ex utero

Amnionic Fluid
volume evaluation is a component of every second- or third-trimester sonogram.
Oligohydramnios
- volume is below normal range, and
- subjective crowding of the fetus is often noted
Hydramnios also called polyhydramnios is defined as
- volume above normal
Measurements include either the single deepest vertical fluid pocket or the sum of the deepest
vertical pockets from each of four equal uterine quadrants the amnionic fluid index
Reference range ha e been e abli hed for bo h mea remen from 16 eek ge a ion on ard
The single deepest vertical pocket is normally between 2 and 8 cm, and the amnionic fluid index
normally ranges between 8 and 24 cm

Fetal Anatomical Evaluation


Important goal of second- and third-trimester sonography is to systematically evaluate fetal anatomy
and determine whether specific anatomical components appear normal or abnormal
If a single ultrasound examination is planned for the purpose of evaluating fetal anatomy, the
American College of Obstetricians and Gynecologists (2011) recommends that it be performed at 18
to 20 weeks.
o At this gestational age range, complex organs such as the fetal brain and heart can be
imaged clearly enough to visualize many major malformations.
Technical factors such as maternal habitus, abdominal wall scarring, fetal size, and fetal position may
limit adequate visualization, and these limitations should be noted in the report.
If imaging is suboptimal, a follow-up examination may be helpful. If an abnormality is identified or
suspected during a standard examination of fetal anatomy, specialized sonography is indicated.

Second-Trimester Fetal Anomaly Detection.


The sensitivity of sonography for detecting fetal anomalies varies according to factors such as
gestational age, maternal habitus, fetal position, equipment features, examination type, operator skill,
and the specific abnormality in question
This reflects inclusion of anomalies with minimal or no sonographic detection, such as microcephaly,
anotia, choanal atresia, cleft palate, bile duct atresia, Hirschsprung disease, anal atresia, and
congenital skin disorders. These are mentioned because clinicians tend to focus on abnormalities
amenable to sonographic detection, whereas families may find those not readily detectable no less
devastating.
Most anomalous infants approximately 75 percent occur in pregnancies that are otherwise low-
risk, that is, without an indication for specialized sonography.

DOPPLER
When sound waves strike a moving target, the frequency of the waves reflected back is shifted proportionate
to the velocity and direction of that moving target a phenomenon known as the Doppler shift
- Because the magnitude and direction of the frequency shift depend on the relative motion of the moving
target, Doppler can be used to evaluate flow within blood vessels

* I did not include the Doppler equation. If you are soooo into this Doppler topic, feel free to use your book
and see how t is being computed. Turo mo sa akin tapos magpre-pretend akong nakikinig. Promise! You
on e en no ice he difference.

Continuous wave Doppler equipment


has two separate types of crystals one transmits high-frequency sound waves, and another
continuously captures signals

14
In M-mode imaging, continuous wave Doppler is used to evaluate motion through time, however, it
cannot image individual vessels

Pulsed-wave Doppler
uses only one crystal, which transmits the signal and then waits until the returning signal is received
before transmitting another one
It allows precise targeting and visualization of the vessel of interest
Configured to allow color-flow mapping such that blood flowing toward the transducer is displayed in
red and that flowing away from the transducer appears in blue.

Umbilical Artery
subjected to more rigorous assessment than has any previous test of fetal health
differs from other vessels in that it normally has forward flow throughout the cardiac cycle
the amount of flow during diastole increases as gestation advances a function of decreasing
placental impedance.
(the computation I was encouraging you to explain to me)
The S/D ratio normally decreases from approximately 4.0 at 20 weeks to 2.0 at term, and it is
generally less than 3.0 after 30 weeks - Because of downstream impedance to flow, more end-
diastolic flow is observed at the placental cord insertion than at the fetal ventral wall abnormalities
such as absent or reversed end-diastolic flow will appear first at the fetal cord insertion site. The
International Society of Ultrasound in Obstetrics and Gynecology recommends that umbilical artery
Doppler measurements be made in a free loop of cord
Recommended that assessment be performed close to the ventral wall insertion to optimize
reproducibility
The waveform is considered abnormal if the S/D ratio is above the 95th percentile for gestational age.
In extreme cases of growth restriction, end-diastolic flow may become absent or even reversed Such
reversal of end-diastolic flow has been associated with greater than 70-percent obliteration of the
small muscular arteries in placental tertiary stem villi (oh e inaral mo na ba equation?)
useful adjunct in the management of pregnancies complicated by fetal-growth restriction, and it has
been associated with improved outcome in such cases
not recommended for complications other than growth restriction and as a screening tool for
identifying pregnancies that will subsequently be complicated by growth restriction
Abnormal umbilical artery Doppler findings should prompt a complete fetal evaluation if not already
done, as such findings are associated with major fetal anomalies and aneuploidy
The Society for Maternal Fetal Medicine has recommended that so long as fetal surveillance remains
reassuring, pregnancies with fetal-growth restriction and absent end-diastolic flow in the umbilical
artery may be managed expectantly until delivery at 34 weeks, and those with reversed end-diastolic
flow managed expectantly until delivery at 32 weeks

Ductus Arteriosus
used primarily to monitor fetuses exposed to indomethacin and other nonsteroidal antiinflammatory
agents (NSAIDs)
Indomethacin, which is used by some for tocolysis, may cause ductal constriction or closure,
particularly when used in the third trimester
The resulting increased pulmonary flow may cause reactive hypertrophy of the pulmonary arterioles
and eventual development of pulmonary hypertension
ductal constriction is often reversible after NSAID discontinuation. Because ductal constriction is a
potentially serious complication that should be avoided, the duration of NSAID administration is
typically limited to less than 72 hours, and women taking NSAIDs are closely monitored so that these
can be discontinued if ductal constriction is identified

Uterine Artery
estimated to increase from 50 mL/min early in gestation to 500 to 750 mL/min by term
Doppler waveform is characterized by high diastolic flow velocities and by highly turbulent flow
Increased resistance to flow and development of a diastolic notch are associated with later
development of gestational hypertension, preeclampsia, and fetal-growth restriction
women with chronic hypertension who had increased uterine artery impedance at 16 to 20 weeks
were at increased risk to develop superimposed preeclampsia.
the predictive value of uterine artery Doppler testing is low, and screening is not recommended in
either high-risk or low-risk pregnancies
technique has not been standardized, frequency of assessment has not been established, and criteria
for an abnormal test have not been determined

perinatal benefits of uterine artery Doppler screening have not yet been demonstrated. (BOOM
Lakas! Yung akala mo yun na pero paasa lang pala. Minsan sayang [tulad ng oras na ginugol mo sa
pagbasa nitong anti sleepiness opinion ko] pero malay natin sa katagalan malalaman nilang,
importante rin pala)

15
Middle Cerebral Artery

fetal anemia detection and fetal-growth restriction evaluation.


Ana omicall , he pa h of he MCA i ch ha flo of en approache he ran d cer head-on,
allowing for accurate determination of flow velocity
imaged in an axial view of the head at the base of the skull, ideally within 2 mm of the internal carotid
artery origin
Velocity measurement is optimal when the insonating angle is close to zero, and no more than 30
degrees of angle correction should be used. In general, velocity assessment is not performed in other
fetal vessels, because a larger insonating angle is needed and confers significant measurement error
When fetal anemia is present, the peak systolic velocity is increased due to increased cardiac output
and decreased blood viscosity
This has permitted the reliable, noninvasive detection of fetal anemia in cases of blood-group
alloimmunization.
In most referral centers, MCA peak systolic velocity has replaced invasive testing with amniocentesis
for fetal anemia detection
MCA Doppler has also been studied as an adjunct in the evaluation of fetal-growth restriction
Fetal hypoxemia is believed to result in increased blood flow to the brain, heart, and adrenal glands,
leading to increased end-dia olic flo in he MCA. Thi phenomenon, brain- paring, i ac ally a
misnomer, as it is not protective for the fetus but rather is associated with perinatal morbidity and
mortality

Ductus Venosus
imaged as it branches from the umbilical vein at approximately the level of the diaphragm
Fetal position poses more of a challenge in imaging the ductus venosus than it does with either the
umbilical artery or the middle cerebral artery.
The waveform is biphasic and normally has forward flow throughout the cardiac cycle
The first peak reflects ventricular systole, and the second is diastolic filling. These are followed by a
nadir during atrial contraction termed the a-wave.
progression of Doppler findings in preterm fetuses with growth restriction, such that umbilical artery
Doppler abnormalities occur first, followed by those in the middle cerebral artery and then the ductus
venosus
wide variability in manifestation of these abnormalities.
When severe fetal-growth restriction is present, cardiac dysfunction may lead to flow in the a-wave
that is decreased, absent, and eventually reversed, along with pulsatile flow in the umbilical vein
abnormalities have potential to identify preterm growth-restricted fetuses that are at greatest risk for
adverse outcomes
recently concluded that Doppler assessment of vessels other than the umbilical artery has not been
shown to improve perinatal outcome and that its role in clinical practice remains uncertain

Additional/summary from Lalala-lala:

NORMAL AND ABNORMAL FETAL ANATOMY

1. Brain and Spine


Standard sonographic evaluation of the fetal brain includes 3 transverse (axial) views:
a. Transthalamic view
- Used to measure the biparietal diameter (BPD) and head circumference (HC) and includes
the midline falx , cavum septum pellucidum (CSP), and thalami.
CSP space between the 2 laminae that separate the frontal horns
Inability to visualize a normal CSP may indicate midline brain abnormality such as
corpus callosum, labor holoprosencephaly, or septo-optic dysplasia (de Morsier
syndrome)
b. Transventricular view
16
- Includes the lateral ventricles, which contain echogenic choroid plexus
- The ventricles are measured in their atrium, which is the confluence of their temporal and
occipital horns
c. Transcerebellar view
- Obtained by angling the transducer back through the posterior fossa
- The cerebellum and cisterna magna are measured
- Between 15 to 20 weeks, the nuchal skinfold thickness may be measured
- From between 15 until 22 weeks, cerebellar diameter in millimeters is roughly equivalent to
the AOG
Cisterna magna normally measures 2-10mm; its effacement is present in the Chiari
II malformation
Imaging of the spine includes evaluation of the cervical, thoracic, lumbar and sacral regions
Representative spinal images for record keeping
- often obtained in sagittal and coronal plane
Real-time imaging
- should include evaluation of each spinal segment in the transverse plane more sensitive for
abnormality detection
Transverse images
— demonstrate 3 ossification centers
Anterior ossification center
— represent the vertebral body
Posterior ossification center
— junction of vertebral laminae and pedicles
Ossification of the spine proceeds in a cranial-caudal fashion
Ossification of upper sacrum (S1-S2) not visible sonographically <16 weeks
Ossification of entire sacrum not visible until 21 weeks
nd
Difficult to detect some spinal anomalies during the early 2 trimester
Examples of spinal abnormalities: spina bifida, caudal regression sequence and
sacrococcygeal teratoma
- More subtle spinal abnormalities may be visible which include:
Diastematomyelia
— longitudinal cleft or splitting of the spinal cord itself
Hemivertebrae
— a component of the vertebral, anal, cardiac, trachea-esophageal fistula, renal,
limb (VACTERL) association
If brain or spinal abnormality is identified, specialized sonography is indicated
Fetal magnetic resonance (MR) imaging
— useful in further characterizing CNS abnormalities

17
Neural Tube Defects

- result from incomplete closure of the neural tube by the embryonic age of 26-28 days
nd
- 2 most common class of malformations after cardiac anomalies
- Can be prevented with folic acid supplementation

a. Anencephaly
- Characterized by absence of the cranium and telencephalic structures, with the skullbase and
orbits covered only by angiomatous stroma

b. Acrania
- Absence of the cranium, with protrusion of disorganized brain tissue
Both anencephaly and acrania are generally grouped together and ancencephaly is
considered to be the final stage of acrania
Lethal anomalies diagnosed in late first trimester
Second trimester all cases may be diagnosed
Third trimester hydramnios from impaired swallowing is common

c. Cephalocele
- Herniation of the meninges through a cranial defect, typically located in the midline occipital
region
Encephalocele brain tissue herniates through the skull defect
- Associated hydrocephalus and microcephaly are common
- Surviving infants have high incidence of neurological deficits and developmental impairment
- Important feature of the autosomal recessive Meckel-Gruber syndrome
Meckel-Gruber syndrome includes cystic renal dysplasia and polydactyly

18
Amnionic band sequence cephalocele not located in the occipital midline

d. Spina Bifida
- Defect in the vertebrae, typically the dorsal arch, with exposure of the meninges and spinal
cord
- Most cases are open spina bifida defect includes skin and soft tissues
Myelomeningocele herniated of the meningeal sac containing neural elements
Meningocele only a meningeal sac is present
nd
- May be reliably diagnosed in 2 trimester sonography due to two characteristic findings:
Lemon sign scalloping of the frontal bones
Banana sign anterior curvature of the cerebellum with effacement of the cisterna
magna
These findings are manifestations of the Chiari II malformation
Chiari II malformation (Arnold-Chiari malformation)
— Occurs when downward displacement of the spinal cord pulls a portion of the
cerebellum through the foramen magnum and into the upper cervical canal
- Children with this defect require multidisciplinary care to address problems related to the
defect, shunt, swallowing, bladder and bowel function and ambulation; they are also at
increased risk for latex allergy
Ventriculocytomegaly
— Frequent sonographic finding particularly after midgestation
— More than 80% of infants with spina bifida require ventriculoperitoneal shunt
placement
— Distention of the cerebral ventricles with CSF as a nonspecific marker of
abnormal brain development
— Atrial width of 10-15mm mild ventriculomegy
— Atrial width of >15mm overt or severe ventriculomegaly
(normal atrium: 5-10mm from 15 weeks to term)
o CSF Produced by the choroid plexus, which is an epithelium-lined
capillary core and loose connective tissue found in the ventricles
o Dangling choroid plexus found with severe ventriculomegaly
— The larger the atrium, the greater the likelihood of abnormal outcome
o Enlarged ventricle may be due to another CNS abnormality such as
Dandy-Walker malformation or holopreoscencephaly
o Dandy-Walker malformation or holopreoscencephaly may be due to an
obstructive process such as aqueductal stenosis or secondary to a
destructive process such as porencephaly or an intracranial teratoma

19
— Initial evaluation: specialized exam of fetal anatomy, fetal karyotyping, and
testing for congenital infections

Agenesis of the Corpus Callosum


— Corpus callosum major fiber bundle connecting reciprocal regions of the
cerebral hemispheres
— Normal cavum septum cannot be visualized sonographically and frontal horns
are displaced laterally
— There is mild enlargement of the atria posteriorly ventricles has a
charac eri ic eardrop appearance
— Associated with other CNS and non-CNS anomalies, aneuploidy and many
genetic sundromes
Holoproscencephaly
— Prosencephalon fails to divide completely into 2 separate cerebral
hemispheres and into underlying diencephalic structures
— Main forms:
a. Alobar
- most severe form
- a single monoventricle, with or without a covering mantle of cortex,
surrounds the fused central thalami
b. Semilobar
- partial separation of the hemisphere occurs
c. Lobar
- characterized by a variable degree of fusion of frontal structures

20
- should be considered when a normal cavum septum pellucidum cannot
be visualized
— May be associated with anomalies of the orbits and eyes (hypotelorism,
cyclopia, or micropthalmia), lips (median cleft), or nose (ethmocephaly,
cebocephaly and arhinia with probiscus)
o This is because the differentiation into two hemispheres is induced by
prechordal mesenchyme, which is also responsible for the
differentiation of the midline face

Dandy-Walker Malformation Vermian Agenesis


— posterior fossa abnormality characterized by agenesis of the cerebellar
vermis, posterior fossa enlargement and elevation of thee tentorium
— Sonographically, fluid in the enlarged cisterna magna visibly communicates
with the fourth ventricle through the cerebellar vermis defect, with visible
separation of the cerebellar hemisphere
— Associated with numerous genetic and sporadic syndromes, congenital viral
infections and teratogen exposure
o inferior vermian agenesis also called Dandy-Walker variant; term used
when only the inferior portion of the vermis is absent
Sacrococcygeal Teratoma
— Germ cell tumor; one of the most common tumors in neonates
— Believed to arise from totipotent cells along Hensen node, anterior to the
coccyx
— 4 types:
a. Type 1 predominantly external with a minimal presacral component
b. Type 2 predominantly external but with a significant intrapelvic
component
c. Type 3 predominantly internal but with abdominal extension
d. Type 4 is entirely internal with no external component
— Sonographically, appears as a solid and/or cystic mass that arises from the
anterior sacrum and usually extends inferiorly and externally as it grows
— Solid components often have varying echogenicity, appear disorganized
and may grow rapidly with advancing gestation
— Internal pelvic components more challenging to visualize, consider fetal
MR imaging
— Hydramnios is frequent
— Hydrops may develop from high-output cardiac failure (either due to tumor
vascularity or secondary to bleeding within the tumor and resultant anemia)
— >5cm tumor cesarean delivery and classical hysterectomy may be needed

Caudal Regression Sequence Sacral Agenesis


— Rare; characterized by absence of sacral spine and often portions of lumbar
spine
— Sonographically, spine appears abnormally short, lacks normal lumbosacral
curvature and terminates abruptly above the level of the iliac wings
o Because sacrum does not lie between iliac wings, they are abnormally
clo e oge her and ma appear hield-like
— There may be abnormal positioning of lower extremities and lack of normal
tissue development
— Should be differentiated from sirenomelia
o Sirenomelia single fused lower extremity in the midline
2. Face and Neck
- Micrognathia
— abnormally small jaw; should be considered in the evaluation of hydramnios
— identified by fetal profile

21
- Facial clefts
— 3 main types:
a. Cleft lip and palate
- involves the lip, may also involve hard palate
- may be unilateral or bilateral
o If cleft in upper lip is visible, a transverse image at the level of the
alveolar ridge may demonstrate that the defect also involves primary
palate
b. Isolated cleft palate
- begins at the uvula; may involve soft palate and occasionally involves the hard
palate
- does not involve the lip
- described using specialized 2- and 3-D sonography
- not visualized during standard ultrasound exam
c. Median cleft lip
- found in association with several conditions which include agenesis of the
primary palate, hypotelorism, and holoprosencephaly
- may be associated with hypertelorism and frontonasal hyperplasia, formerly
called median cleft face syndrome

- Cystic Hygroma
— Venolymphatic malformation in which fluid filled sacs extend from the posterior neck
st
— May be diagnosed in the 1 trimester
— Believed to develop when lymph from head fails to drain into the jugular veins and
accumulates instead in the jugular lymphatic sacs
— 70% are associated with aneuploidy
nd
— If diagnosed on 2 trimester, 75% of aneuploidy cases are 45,X Turner syndrome
st
— If diagnosed on 1 trimester, trisomy 21 and 45,X are common, followed by trisomy
18
— In the absence of aneuploidy, there is increased risk for cardiac anomalies that are
flow-related (hypoplastic left heart and coarctation of the aorta)
— May be part of a genetic syndrome
o Noonan syndrome an autosomal dominant disorder that shares
several features with Turner syndrome, including short stature,
lymphedema, high-arched palate and often pulmonary valve stenosis
— Large cystic hygromas found in hydrops fetalis, rarely resolve and carry poor
prognosis
— Small hygromas may undergo spontaneous resolution and if fetal karyotype and
ECG results are normal, the prognosis may be good

22
3. Thorax
- Lungs appear as homogeneous structures surrounding the heart and best visualized 20-25
weeks AOG
- In a four-chamber view of the chest: (lungs: 2/3, heart 1/3)
- Thoracic circumference measured at the skin line in a transverse plane at the level of the
four-chamber view
- Congenital Diaphragmatic Hernia
— Defect in the diaphragm through which abdominal organs herniate into the thorax
o 75% of cases left sided
o 20%-- right sided
o 5%-- bilateral
— Targeted sonography and fetal ECG should be performed
— Reduction in survival rate if with associated abnormalities
— If without associated abnormalities, major cause of death are pulmonary hypoplasia
and pulmonary hypertension
— Sonographically, most frequent finding with left sided defect is repositioning of the
heart tp the mid or right thorax
— Associated findings: stomach bubble or bowel peristalsis in the chest and wedge
shaped mass which is the liver located anteriorly in left hemithorax
— Large lesions impaired swallowing and mediastinal shift resulting in hydramnios
and hydrops

- Congenital Cystic Adenomatoid Malformation (CCAM)


— Abnormality presents with hamartomatous overgrowth of terminal bronchioles that
communicated with the tracheobronchial tree
— A.k.a CPAM (congenital pulmonary airway malformations)
— Sonographically, there is a well-circumscribed mass that may appear solid and
echogenic or may have one or multiple variably sized cysts
— Usually involves one lobe and has blood supply from the pulmonary artery and drains
into pulmonary veins
— Large mass hydrops fetalis and hypoplasia may result
— Macrocystic lesions with cyst >5mm
— Microcystic <5mm
- Extralobular Pulmonary Sequestration
— A.k.a. bronchopulmonary sequestration
— An acce or l ng b d eq e ered from he racheobronchial ree a mass of
nonfunctioning lung tissue
— Extralobar most cases diagnosed prenatally; developed in their own pleura
23
— Interlobar most sequestrations present in adulthood; within the pleura of another
lobe
— Sonographically, Extralobar Pulmonary Sequestration (ELS) presents as
homogeneous, echogenic thoracic mass; it may resemble microcystic CCAM,
however,blood supple in from systemic circulation
- Congenital High Airway Obstruction Sequence (CHAOS)
— Rare; results from laryngeal or tracheal atresia
— Normal egress of lung fluid is obstructed and the tracheobronchial tree and lungs
become massively distended
— Sonographically, lungs appear brightly echogenic and the bronchi are dilated with
fluid
— Flattening and eversion of diaphragm and compression of the heart is common
— Venous return impaired and ascites develops
— A feature of the autosomal recessive Fraser syndrome
4. Heart
- Cardiac malformations most common class of congenital anomalies
- 90% of cardiac defects multifactorial or polygenic in origin
nd
- Routine 2 trimester sonography identified approximately 40% of major cardiac anomalies
before 22 weeks
- Specialized sonography identified 80%
- Basic Cardiac Exam
— Standard cardiac assessment includes 4 chamber view
— Evaluation of rate and rhythm and evaluation of left and right ventricular outflow tracts
— Evaluation of outflow tracts aid in detection of anomalies such as tetralogy of fallot,
transposition of great bessels and truncus arteriosus, which are not initially
appreciated in 4 chamber view
- Four-Chamber view
— A transverse image of the fetal thorax at the level immediately above the diaphragm
— Allows evaluation of cardiac size, position in the thorax, cardiac axis, atria and
ventricles, foramen ovale, atrial septum primum, interventricular septum and
atrioventricular valves
o Atria and ventricles should be same size
o Apex of the heart should form 45-degree angle with the left anterior chest
wall

- Fetal Echocardiography
— Specialized examination of fetal cardiac structure and function designed to identify
and characterize abnormalities
— Indications: suspected fetal cardiac anomaly, extracardiac anomaly, chromosomal
abnormality, fetal arrhythmia, hydrops, increased nuchal translucency, monochorionic
twin gestation, first-degree relative to a fetus with congenital cardiac defect, in vitro
fertilization, maternal anti-Ro or anti-La antibodies, exposure to medications
associated with increased cardiac malformation risks, and maternal metabolic
disease associated with cardiac defects

24
- Ventricular Septal Defect
— Single most common cardiac congenital anomaly
— A defect may be appreciated in the membranous or muscular portion of the
interventricular septum in the four-chamber view
— Color Doppler demonstrates flow through the defect
— Imaging of the left ventricular outflow tract may demonstrate discontinuity of the
interventricular septum as it becomes the wall of the aorta

- Echocardial Cushion Defect


— A.k.a. atrioventricular (AV) septal defect or AV canal defect
— Echocardial cushions are the crux of the heart and defects jointly involve the atrial
septum primum, interventricular septum, and medial leaflets of the mitral and
tricuspid valve
— May develop with heterotaxy syndrome more likely to have conduction system
abnormalities resulting in third-degree AV block
o Atrial isomerism heat and/ or abdominal organs are on the incorrect side
- Hypoplastic Left Heart Syndrome
— Postnatal treatment consists of a three stage palliative repair or a cardiac
transplantation
— Morbidity remains high and developmental delays are common
— Sonographycally, the left side of the heart may appear so small that it is difficult to
appreciate a ventricular chamber
— There may be no visible inflow or outflow and may be reversal pf flow in the ductus
arteriosus
- Tetralogy of Fallot
— Characterized by four components:
o Ventricular septal defect (VSD)
o Overriding aorta
o Pulmonary valve abnormality
o Right ventricular hypertrophy
— Due to the location of the VSD, it is often not visible in the four-chamber view, which
may appear normal
- Cardiac Rhabdomyoma
25
— Most common cardiac tumor
— Appear as well circumscribed echogenic masses, usually within ventricles and
outflow tracts
— May be one or multiple; may increase in size during gestation; inflow or outflow
obstruction may result
— No obstruction good prognosis
— Largest in the neonatal period and tends to regress as children grow
o If fetal rhabdomyoma is identified, in the absence of a family history,
evaluation of parents for clinical manifestations of neurofibromatosis should
be considered
- M-Mode (Motion-mode imaging)
— A linear display of cardiac cycle events, with time on x-axis and motion on y-axis
— Used frequently to measure fetal heart rate
— If with abnormal heart rate and rhythm, it permits separate evaluation of atrial and
septal waveforms
— Useful in characterizing arrhythmias and their response to treatment

-Premature Atrial Contractions


— A.k.a. atrial extrasystoles
— Most common fetal arrhythmia and frequent finding
— Represent cardiac conduction system immaturity
— Typically resolve later in gestation or in the neonatal period
— May be conducted and sound like an extra beat
— With handheld doppler or fetoscope, they sound like a dropped beat
o The dropped beat may be demonstration with M-mode evaluation to be the
compensatory pause that follows the premature contraction
5. Abdominal Wall
- Standard examination: assess integrity of the abdominal wall at the level of the cord insertion
- Gastroschisis and omphalocele collectively termed ventral wall defects; relatively common
fetal anomalies; both associated with elevated maternal serum alpha-fetoprotein
- Gastroschisis
— Full-thickness abdominal wall defect typically located right of the umbilical cord
insertion
— Bowel herniates through defect into the amniotic cavity
— One major common anomaly in fetuses of younger mothers and average maternal
age of 20 years

26
- Omphalocele
— Forms when the lateral ectomesodermal folds fail to meet in the midline, leaving the
abdominal contents covered onlu by a two-layered sac of amnion and peritoneum
into which the umbilical cord inserts
— Is a component of syndromes such as Beckwith-Wiedemann, cloacal extrophy, and
pentalogy of Cantrell
- Body Stalk Anomaly
— A.k.a. limb-body-wall complex or cyllosoma
— Rare, lethal anomaly characterized by abnormal formation of the body wall
— No abdominal wall is visible and there is extrusion of the abdominal organs into the
extraamniotic coelom
— There is close approximation or fusion of the body to the placenta and an extremely
short umbilical cord
— Acute-angle scoliosis is another feature
— Amniotic bands are often identified

6. Gastrointestinal Tract
- 14 weeks Stomach is visible
nd rd
- 2 and 3 trimester liver, spleen, gallbladder and bowel can be identified
- Nonvisualization of stomach may be due to impaired swallowing (underlying causes
include: esophageal atresia, craniofacial abnormality or CNS or musculoskeletal abnormality)
- Bowel may appear bright or echogenic, which may indicated small amounts of swallowed
intraamnionic blood (particularly in elevated maternal serum alpha-fetoprotein)
o Bowel that appears as bright as fetal bone confers a slightly increased risk
for underlying GI malformations, cystic fibrosis, trisomy 21 and congenital
infection
- Gastrointestinal Atresia
— Obstruction and proximal bowel dilatation
— The more proximal the obstruction, the more likely there is hydramnios
o Hydramnios associated with proximal small bowel obstruction may be severe
enough to results in maternal respiratory compromise or preterm labor
o This may necessitate amniocentesis, a.k.a. amnioreduction
— Esophageal atresia
- may be suspected if stomach is not visualized and hydramnios is present
- 90% of cases present with tracheoesophageal fistula which allows fluid to enter the
stomach; this results to a problem in prenatal detection
— Duodenal atresia
- characterized by sonographic double-bubble sign, which represents distention of the stomach
and the first part of the duodenum; this finding is usually not present before 22-24 weeks

7. Kidneys and Urinary Tract


- Fetal kidneys
st
— visible adjacent to spine by 1 trimester and routinely by 18 weeks AOG
— become les echogenic and a rim of perinephric fat aids visualization of their margin
— produce most of amniotic fluid after 18weeks AOG
— urine production increases from 5mL/ hr at 20weeks to 50mL/hr at term
- 20mm length of kidney at 20 weeks; increasing by 1.1mm each week
- Unexplained oligohydramnions suggests a placental or urinary tract abnormality

27
nd
- Normal amniotic fluid volume in 2 half of pregnancy urinary tract patency with at least 1
functioning kidney

- Renal Pelvis dilatation


— May be transient or physiological
— May represent urinary tract abnormality which is confirmed in the neonatal
period
— Either uteropelvic function junction (UPJ) obstruction or vesicoureteral reflex
(VUR)
o Renal pelvis is measured anterior-posterior in the transverse plane
nd rd
o Pelis dilated > 4mm in 2 trimester or 7mm in 3 trimester
- Uteropelvic Junction Obstruction
— Most common anomaly associated with renal pelvis dilatation
— Affects males more than females
— Obstruction is functional rather than anatomical
- Duplicated Renal Collecting System
— Occurs when the upper and lower poles of the kidney (known as moieties) are each
drained by separate ureter
— More common among females
— Sonographically, an intervening tissue band separates 2 distinct renal pelves
o Hydronephrosis and/or ureteral dilatation develops due to abnormal dilatation
of one or both ureters within the bladder a relationship descried by Weigert-
Meyer rule
o Upper pole of ureter develops obstruction from a uterocele within the
bladder
o Lower pole has a shortened intravesical segment that predisposes to
vesicoureteral reflux

- Renal Agenesis
— When kidney is absent, the ipisilateral adrenal gland typically enlarges to fill the renal
fossa termed as lying down adrenal sign

28
— Absence of renal artery Demonstrated by color Doppler imaging of the descending
aorta
— If bilateral no urine if formed resulting to hydramnios which leads to pulmonary
hypoplasia, limb contractures and a distinctively compressed face Potter Syndrome
o If these abnormalities resulting from decreased AF is not caused by renal
agenesis Potter Sequence
- Multicystic Dysplastic Kidney
— Severe form of renal dysplasia which results in a nonfunctioning kidney
— Nephrons and collecting duct do nor form normally
— Primitive ducts are surrounded by fibromuscular tissue
— Ureter is atretic
— Sonographically, kidneys contain numerous smooth walled cysts of varying sizes that
do not communicated with the renal pelvis and are surrounded by echogenic cortex
— If isolated and unilateral prognosis is good
— If bilateral associated with decreased AF volume leading to Potter sequence poor
prognosis

- Bladder Outlet Obstruction


— Distal obstruction of urinary tract
— More common in males
— Common etiology: posterior urethral valves
— There is dilatation of the bladder and proximal urethra ke hole ign
— Bladder wall is thick
— If with oligohydramnios poor prognosis due to pulmonary hypoplasia
— Outcome may be poor even with normal AF volume
- Polycystic Kidney Disease
— Only the infantile form of autosomal recessive polycystic kidney disease (ARPKD) is
diagnosed prenatally
o ARPKD
chronic, progressive disease that involves kidney and liver
results in cystic dilatation of the renal collecting ducts and congenital
hepatic fibrosis
has wide phenotypic variability from lethal pulmonary hypoplasia at birth
to hepatic manifestations in the late childhood and adulthood
— Infantile characterized by abnormally large kidneys that fill and distend fetal
abdomen and have a solid glass texture
— Severe oligohydramnios poor prognosis
o Autosomal Dominant Polycystic Kidney Disease (ADPKD)
more common; does not manifest until adulthood
may have mild renal enlargement, increased renal echogenicity, and
normal AF volume
differential diagnosis: several genetic syndromes, aneuploidy or normal
variant
8. Skeletal Abnormalities
- 2 types of skeletal dysplasias:
Osteochondrodysplasias generalized abnormal development of the bone and/or
cartilage
Dystoses abnormalities of individual bones (ex. Polydactyly)

29
o In addition to these malformations, skeletal abnormalities include
deformations (ex. clubfoot) and disruptions (ex. Limb-reduction defects)
- Skeletal Dysplasias
— 2 groups account for most cases:
o Fibroblast growth factor 3 (FGFR 3) chondrodysplasia group
Include achondroplasia and thanatophoric dysplasia
Achondroplasia
- a.k.a. heterozygous achondroplasia
- most common nonlethal skeletal dysplasia
- inherited in an autosomal dominant fashion
- characterized by long bone shortening that is predominantly
rhizomelic, an enlarged head with frontal bossing, depressed nasal
bridge, exaggerated lumbar lordosis and a trident configuration of the
hands
- sonographically, femur and humerus measurements <5% until early
rd
3 trimester
Thanatophoric dysplasia
- most common lethal skeletal disorder
- characterized by severe micromelia and affected fetuses may
develop a characteristic cloverleaf skull deformity due to
craniosyntosis
o Osteogenesis imperfect and decreased bone density group
— Evaluation include survey of every long bone, hands and feet, skull size and shape,
clavicles, scapulae, thorax and spine
o Micromelia involvement of all long bones
o Rhizomelia Involvement of proximal long bones
o Mesomelia involvement of intermediate long bones
o Acromelia involvement of distal long bones
— Note for degree of ossification and presence of bowing and fractures narrows
differential diagnosis and suggests specific skeletal dysplasia
o Lethal dysplasias frequently characterized by profound long bone
shortening (<5%) and by femur length-to-abdominal circumference ratios
(<16%)
o Indicative of pulmonary hypoplasia:
- thoracic circumference is 80% of the abdominal circumference
- thoracic circumference <25%
- cardiothoracic circumference ratio >50%

— Osteogenesis imperfecta represents a group of skeletal dysplasias characterized


by hypomineralization
o There are multiple types but type IIa (a.k.a. perinatal form) is lethal and
characterized by profound lack of skull ossification such that gentle pressure
on maternal abdomen from the ultrasound transducer results in visible skull
deformation
o Other features: multiple in- ero frac re and rib ha appear beaded
— Hypophosphatasia skeletal dysplasia that results in severe hypomineralization;
inherited in autosomal recessive fashion
- Clubfoot Talipes Equinovarus
— Deformed talus and shortened Achilles tendon

30
— Affected food abnormally fixed and positioned with equinus (downward pointing),
varus (inward rotation) and forefoot adduction
— Sonographically, footprint is visible in the same plane as the tibia and fibula

- Limb-Reduction Defects
— Absence of all or part of one or more extremity
o Amelia absence of an entire extremity
o Phocomelia absence of one or more leg bones with hands or feet attached
to the trunk; associated with thalidomide exposure
— Associated with the following:
o Roberts syndrome an autosomal recessive condition characterized by
tetraphocomelia
o Clubhand deformity associated with trisomy 18 and also a component of
the thrombocytopenia-absent radius syndrome
o Chorionic villi sampling performed <10 weeks AOG

THREE- AND FOUR- DIMENSIONAL SONOGRAPHY

3-D sonography

- Not routinely used during standard exam


- Not a required modality but may be a component of various specialized evaluations
- Uses a special transducer
o After a region of interest is identified, a 3-D volume is acquired that renders to diplay
images of any plane (axial, sagittal, coronal or even oblique) within that volume
o Sequential slices can be generated (similar to CT scan and MRI)
- Can evaluate intracranial anatomy in the sagittal plane (ex. corpus callosum) and imaging of
the palate and skeletal system
- Static and obtained by processing of a volume of stored images

4-D Sonography

- A.k.a. real time 3-D sonography


- Allows rapid reconstruction of the rendered images to convey the impression that the
scanning is in real time
- Can improve visualization of cardiac anatomy
- Useful in postprocessing algorithms and techniques (ex. spatiotemporal image correlation/
STIC which us used to evaluate complex heart anatomy and function)
o addition of an inversion-mode algorithm may aid in imaging of blood flow within the
heart and great vessels and may permit measurement of ventricular blood volume

31
PRENATAL DIAGNOSIS - Exposed to excessive medication or to
Dr. Gonzalez environmental hazards
- History of down syndrome or some other
Prenatal Diagnosis chromosomal abnormality is present
- Science of identifying structural or functional - The fetus is detected to be at increased risk
abnormalities birth defects in the fetus for NTD
- Appropriate counseling and optimize
outcome
- Fetal therapy
o Blood product transfusion Neural Tube Defects
nd
o Administration of medication - 2 most common congenital malformation
o Amioreduction/amnioinfusion - Family history
o Fetal thoracentesis - Aneuploidy-trisomy 13 and 18, triploidy
o Fetal surgery - Hyperglycemia
o Laser or radiofrequency ablation of - Hyperthermia
vascular anastomoses - Coumadin
- Aminopterin
Benefits of Prenatal Diagnosis - Thalidomide
- Determines the outcome of pregnancy and - Anticonvulsants: valproic and
identifies possible complications that can carbamazepine
arise during birth
- It can be helpful in improving the outcome of Folic Acid
pregnancy using fetal treatment - Reduce risk of NTD by 70%
- Screening can help couples prepare for the - Low risk: 0.4 mg/day
birth of a child with an abnormality - High risk: 4 mg/day prior to pregnancy up to
st
the 1 trimester
Etiology of Birth Defects
- Malformation PRENATAL DIAGNOSIS OF NTD
o Most common type of structural fetal Alpha Fetoprotein
abnormality - Glycoprotein synthesized by the fetal yolk
o An intrinsic abnormality sac and later by the GIT and liver
programmed in development - Major serum protein of the embryo and early
regardless of whether precise fetus
genetic etiology is known - Concentration increases steadily in both
- Deformation fetal serum and AF until 13 weeks gestation
o A genetically normal fetus develops - Major source of AFP in AF is fetal urine
abnormally because of mechanical - Some proteins cross the fetal membrane to
forces imposed by the uterine enter maternal circulation
environment - After 13 weeks gestation, both the fetal
- Disruption serum and AF levels normally decreases
o More severe change in form or while those in maternal serum continue to
function that occurs when rise until late in pregnancy
genetically normal tissue is modified
as a result of specific insult Maternal Serum AFP
- Performed at 15-20 weeks AOG
Techniques - Upper limit of normal 2.0 or 2.5 MoM
- Non-invasive o Detection rate of 90% for
o Ultrasound anencephaly
o X ray o Detection rate of 80% for spina
o Magnetic resonance bifida
- Invasive - Abnormal test
o Fetoscopy o Counseling
o Amniocentesis o Targeted sonography
o Chorionic villus sampling o Amniocentesis
o Percutaneous umbilical cord
sampling Amniocentesis
- Amniotic fluid AFP
Prenatal Diagnosis - Acetylcholinesterase
- The pregnant woman >/35 at the time of o Most sensitive and specific in
delivery determining NTD
- She or her parents have had a previous o Leaks directly from the exposed
child with a chromosomal abnormality neural tissue
- History of recurrent abortions, or her - Sensitivity: 90%
h band pre io ife e perienced
several miscarriages Elevated MSAFP
- The couple is known to be carriers of a - NTD
chromosomal translocation - Intestinal obstruction
- The pregnant woman is affected with type I - Cyctic hygroma
DM, epilepsy, etc. - Sacrococcygeal teratoma
- Exposed to viral infections (rubella or - Omphalocoele, gastroschisis
cytomegalovirus) - Urinary obstruction

1
- Renal anomalies polycystic, absent PRENATAL DIAGNOSIS OF DOWN SYNDROME
kidneys - 11-14 weeks
- Sonographic evaluation
*Ultrasound 99% accuracy with NTD o Nuchal translucency
- Maternal serum anylate screening
NTDs o Increase serum hCG levels
- Anencephaly o Pregnancy associated plasma
- Spina bifida protein A (PAPP-A): lower
- Cephalocoele - Detection rate: 79-87%
- Encephalocoele
Second Trimester Screening
Anencephaly - Triple creen
- Failure of closure at the cranial end of the o Low MSAFP
neural tube (26-28 weeks) o Increased hCG
- Absence of the cranium and telencephalon o Low levels of unconjugated estriol
- Hydramnios o Detection rate: 61-70%
- Lethal during neonatal period - Q ad creen
o Low MSAFP
*Acrania: only cranium is absent; with brain o Low unconjugated estriol
o Increased hCG
Spina Bifida o Increased inhibin A
- Failure of distal closure during early o Detection rate: 80%
embryogenesis
- Defect in the vertebrae, typically the dorsal Integrated Screen
arch with exposure of the meninges and - First trimester screen and Quad test
spinal cord - Detection rate: 94-96%
- Associate with various degree of motor
impairment, lower limb paralysis or INVASIVE TECHNIQUES
dysfunction and incontinence - Embryoscopy
- Associated with hydrocephalus >> poor - Fetoscopy
prognostic factor for intellectual - Amniocentesis
development - Chorionic villus sampling
- Spina bifida occulta - Cordocentesis
o Vertebral shisis covered by normal
soft tissues Embryoscopy
st
- Spina bifida aperta - Performed in the 1 trimester (up to 12
o Full thickness defect of the skin, weeks AOG)
underlying soft tissues and vertebral - A rigid endoscope is inserted via the cervix
arches and exposing the neural in the space between the amnion and the
canal chorion, under sterile conditions and U/S
o Meningocoele guidance, to visualize the embryo for the
Defect is covered by thin diagnosis of structural malformations
meningeal membrane
o Myelomeningocoele Fetoscopy
nd
Presence of neural tissue - Performed during the 2 trimester (after 16
inside the sac weeks AOG)
- A fine-caliber endoscope is inserted into the
*Spina bifida: compatible with life amniotic cavity and U/S guidance, for the
- Common problem: urinary/fecal retention visualization of the embryo to detect the
o Management: catheterization every presence of structural abnormalities
6 hours - Also used for fetal blood and tissue
sampling
Cephalocoele - Associated with a 3-5% risk of miscarriage;
- Herniation of meninges through a cranial therefore, it is suprseded by detailed U/S
defect scanning
- Most common: midline occipital region
nd
- Meckel-Gruber syndrome 2 Trimester Amniocentesis
o Cephalocoele - 15-20 weeks
o Cystic renal dysplasia - Fetal karyotyping
o Polydactyly - Cytogenetic studies
- Enzyme and DNA
Encephalocoele - Alpha fetoprotein
- Incomplete skull formation with extrusion of - Acetylcholinesterase
varying amount of brain tissue
- Most common: occipital area Early Amniocentesis
- Associate with hydrocephalus and - 11-14 weeks
microcephaly - Less fluid can be withdrawn
- Neonatal mortality rate: 40% - Increased culture failure rate
- Intellectual impairment and neurologic - Higher rates of abortion (2.5% vs 0.7%)
sequelae: 80% - Higher rates of talipes equinovarus
*Compatible with life

2
o Oligohydramnios, damage to the - Fetal anemia (alloimmunization)
vascular supply of the developing o Blood transfusion into the umbilical
leg vein
- Higher rates of fluid leakage - Fetal pleural effusion
- Not recommended by ACOG o Fetal thoracentesis
- Gastroschisis
Amniocentesis o Amnioexchange
- Lung maturity Replace AF with NSS
o L/S ratio o Intestines exposed to AF >> harmful
o Phosphatidylglycerol - Fetal thyrotoxicosis
o Lamellar body count o Propylthiouracil
**Use 10 cc of AF; Coulter - Fetal arrhythmia
counter (machine used?) o Antiarrhythmic agents digoxin,
- Chorioamnionitis verapamil, propranolol
o C l re and en i i i , Gram ain o May be given to the mother
- Meconium staining o Through the umbilical cord or
- Complications intramuscular
o Fetal and maternal trauma
o AF leakage **Karyotyping result
o Vaginal spotting - Cordocentesis: 2 weeks
o Chorioamnionitis - CVS: 4 weeks
o Abortion
o Preterm labor

Chorionic Villus Sampling


- Performed at 10-13 weeks
- Transcervically or transabdominally
- Cytogenetic studies
- Fetal karyotype
- Enzyme and DNA analysis
- Results are available earlier
- Early CVS (7 weeks) increased incidence
of limb reduction and oromandibular defects
- Complications
o Vaginal bleeding
o Abortion
o Infection
o Limb abnormalities

Corodocentesis
- Prenatal diagnosis of blood disorder
- Isoimmunization
- Fetal viral and bacterial infections
- Rapid fetal karyotyping
- Metabolic and biochemical analysis
- DNA analysis
- Acid-base analysis
- Immunologic studies
- Complications
o Fetal loss
o Chorioamnionitis
o Ruptured membranes
o Bleeding
o Severe bradycardia

Fetal Tissue Biopsy


- Muscle biopsy
o Muscular dystrophy
o Mitochondrial myopathy
- Skin biopsy

Fetal Therapy
- Twin-to-twin transfusion syndrome
o Unbalance vascular anastomoses
that connect the 2 fetal circulations
Twins: 1 LGA
(polyhydramnios), 1 SGA
(oligohydramnios)
o Amnioreduction
o Septostomy
o Laser occlusion of placental
vascular anastomoses
3
OB1: Passages
Lectures per Facilitator
YELLOW - Dr. Lee

BLUE Dr. Gonzalez

RED Dr. Mariano

Pelvic Bones
composed of four bones:
o sacrum
o coccyx
o two innominate bones
o Each is formed by the fusion of
three bones
the ilium, ischium, and
TRUE PELVIS important portion especially
pubis (Fig. 2-16)
during childbearing
o Both are joined to the sacrum @
o Described as: in er ed J haped
sacroiliac synchondroses and to
an obliquely truncated
one another @ symphysis pubis
bent cylinder
with greatest height posteriorly
o Linea Terminalis superior border
o Iliopectineal line
o Imaginary line w/c separates
true from false pelvis
o Pelvic Outlet inferior margin
o sidewalls of the true pelvis of an adult
woman somewhat converge
o Ischial Spines extends from the
middle of the posterior margin of each
ischium
These are of great obstetrical
importance
distance between them - represents
the shortest diameter of the true
pelvis
also serve as valuable landmarks
o assessment of level to
which the presenting part
of the fetus has descended
into the true pelvis
conceptually divided into false and true
these aid pudendal nerve block
components
placement
True Pelvis False Pelvis o sacrum - posterior wall of the true pelvis
Below the Above the upper anterior margin
LOCATION linea linea corresponds to the
terminalis terminalis
promontory that may
P: anterior P: Lumbar be felt during
surface of Vertebra bimanual pelvic
sacrum L: Iliac examination in women
L: inner Fossa with a small pelvis
surface of A: Lower provide a landmark for clinical
ischial bones portion of pelvimetry
& sacrosciatic anterior Normally, it has a marked
notches & abdominal vertical & a less pronounced
BOUNDARIES ligaments wall horizontal concavity
A: pubic
ABNORMAL: undergo
bones,
important variations
ascending
straight lines
superior rami
from the promontory
of the ischial
to the tip of the
bones, &
sacrum = 10cm
obturator
foramina distance along the
concavity = 12 cm

Pelvic Joints
ANTERIORLY: pelvic bones - joined together by the
symphysis pubis
consists of fibrocartilage and the superior &
inferior pubic ligaments
o latter ligament - frequently designated
the arcuate ligament of the pubis
POSTERIORLY: pelvic bones - joined by articulations
between the sacrum and the iliac portion of the innominate
bones to form the sacroiliac joints

1
These joints in general have a limited degree (Diagonal conjugate - 1.5 to 2cm)
of mobility = <11.5 cm means inadequate
during pregnancy
o there is remarkable relaxation of
these joints at term
o caused by upward gliding of the
sacroiliac joint
The displacement may increase the diameter
of the outlet by 1.5 to 2.0 cm.
o greatest in the dorsal lithotomy
position
o main justification for a vaginal
delivery
increase in pelvic outlet diameter occurs only
if the sacrum is allowed to rotate posteriorly 2. Transverse diameter
o will not occur if the sacrum is forced o constructed at right angles to the
anteriorly by the weight of the obstetrical conjugate
maternal pelvis against the delivery o represents the greatest distance
table or bed between the linea terminalis on either
side
Sacroiliac joint mobility = successful McRoberts o can be mea red
maneuver o usually intersects the obstetrical
o Release of an obstructed shoulder conjugate at a point approx. 5 cm in
cases of shoulder dystocia front of the promontory and measures
(+) labor difficulty approx. 13 cm
These changes have also been attributed to the
success of the modified squatting position to 3. Two oblique diameters
hasten second-stage labor o extends from 1 sacroiliac synchondrosis
o squatting position -
diameter & pelvic outlet diameter o Each eminence - minor
NOTE: These latter observations are unconfirmed, but this elevation
position is assumed for birth in many societies. marks the union site
of the ilium & pubis
Planes and Diameters of the Pelvis o These oblique diameters average < 13
The pelvis is described as having 4 imaginary planes: cm
1. The plane of the pelvic inlet the superior strait
2. The plane of the pelvic outlet the inferior strait
3. The plane of the midpelvis the least pelvic Midpelvis and Pelvic Outlet
dimensions Midpelvis
4. The plane of greatest pelvic dimension no o measured at the level of the ischial
obstetrical spines
significance. o lower margin of the anterior symphysis
Pelvic Inlet o also called the midplane or plane of
called he perior rai & i al o he perior least pelvic dimensions (Fig. 2-19)
plane of the true pelvis
bounded by the promontory & alae of the sacrum
(P), by the linea terminalis (L), and by the
horizontal pubic rami & the symphysis pubis (A)
During labor, fetal head engagement is defined
b he fe al head biparie al diame er pa ing
through this plane
o To aid this passage, the inlet of the
female pelvis more nearly round
(gynecoid) than ovoid
compared with the male pelvis

Four diameters of the pelvic inlet are usually described:


1. AnteroPosterior described using specific
landmarks
o Most cephalad
o AP diameter true /anatomic conjugate
Not measurable o During labor, the degree of fetal head
o Extends from the upper most margin of descent into the true pelvis may be
the symphysi described by station
promontory midpelvis and ischial spines
serve to mark zero station.
Obstetrical Conjugate o The interspinous diameter = 10 cm or
o Clinically important (not measurable) slightly greater
o Shortest distance between sacral usually the smallest pelvic
promontory and symphysis pubis = diameter
smallest diameter important in cases of
o Normally measure > 10cm obstructed labor
o Can be mea red direc l i h o AP diameter thru the level of the ischial
examining fingers spines normally measures at least 11.5
Diagonal conjugate distance from cm.
lowest margin (inferior border) of the o Can only be estimated thru an Internal
Exam
Clinical purpose measured with Feel for the sacrum
estimation
2
✓curvature =
adequate midpelvis From viewing the four basic types in Figure 2-20, the
(12cm) configuration of the gynecoid pelvis would intuitively seem
Feel the ischial spines suited for delivery of most fetuses. Indeed, Caldwell (1939)
+ prominent = small reported that the gynecoid pelvis was found in almost half
interspinous/bispinous of women.
diameter o Gynecoid Pelvis most suited for fetal delivery
Converge of sidewalls o Extreme android pelvis poor Dx for VSD
✗ convergent walls o Anthropoid type 33% of women

Pelvic Outlet Tests for assessment of pelvic adequacy:


o consists of two approximately triangular Diagonal conjugate determination (11.5-12cm)
areas Assess engagement
boundaries mirror the perineal Müller-Hillis Manuever
triangle o Pushes fundus downward then if the
o have a common base head of the fetus reaches the ischial
a line drawn between the two spine = (+) adequate
ischial tuberosities o Head is at ischial spine = station 0 =
o apex of the posterior triangle - tip of the head is engaged
sacrum o Engagement when the bipareital
o lateral boundaries - sacrotuberous diameter of the flexed head is at the
ligaments and the ischial tuberosities level of the pelvic inlet
o anterior triangle - formed by the
descending inferior rami of the pubic Indications for Xray Pelvimetry
bones Fetal delivery by breech or vaginally
These rami unite at an angle of o If head is flexed or if the head will fit
90-100 degrees to form a (+) previous injury to pelvis (fracture/trauma/hip
rounded arch under which the replacement) / (+) polio infection
fetal head must pass o changes in pelvic configuration
o Clinically, three diameters of the pelvic previous CS
outlet usually are described:
AnteroPosterior diameter
Transverse diameter All PRIMIs w/ breech presentation are delivered
Posterior Sagittal diameter adbominally
NOTE: Unless there is significant pelvic bony disease, the Generally, clinical pelvimetry is done for PRIMIs &
pelvic outlet seldom obstructs vaginal delivery. mothers who did not deliver vaginally during
o Thing to assess are: previous pregnancies
o Subpubic Arch Patient must be in dorsal lithotomy position
✗ measure the angle o When legs are elevated, AP diameter
increases
determine either wide
(>90) or not
o Bituberous diameter
Clinical Obstetrics
At least 8cm (nlt 8cm)
(+) pelvic contractions = difficulty in vaginal
Try to see if clenched
delivery
fist can fit
o indication for CS
Can be mea red
o Types of contraction
during IE
Inlet contraction
Pelvic Shapes
Inlet & Midpelvis contraction
The Caldwell-Moloy ( 33, 34) ana omical cla ifica ion of
Midpelvis contraction
the pelvis
Midpelvis & outlet contraction
based on shape, and its concepts aid an
Outlet contraction lowest
understanding of labor mechanisms
incidence
greatest transverse diameter of the inlet
Trial of Labor
its division into anterior and posterior segments
o Except for those with:
o used to classify the pelvis as gynecoid,
Placenta previa
anthropoid, android, or platypelloid.
Breech in a PRIMI
posterior segment determines the
Malpresentation
type of pelvis
All are indicated for
anterior segment determines the
CS
tendency
G2P
These are both determined st
o 1 Pregnancy: CS
because many pelves are not nd
o 2 Pregnancy: vaginally
pure but are mixed types
request for pelvic scan before
E.g. a gynecoid pelvis w/ an proceeding.
android tendency
o P pelvis = gynecoid
o A pelvis = android
shaped

3
PASSENGER 1. Cephalic Presentation
— Full flexion is ordinarily achieved
because occipital condyles are
located near the posterior aspect of
3P the skull
As head enters the inlet, the
1. Passages longer segment of the lever
2. Passenger yields to the pressure and
3. Power flexion results
— Classified according to the
relationship between the head and
Fetal orientation relative to the maternal pelvis is body of the fetus:
described in terms: a. Vertex or occiput
presentation
1. Fetal lie head is fully flexed sharply
2. Fetal presentation so that the chin is in contact
3. Fetal attitude with the thorax
4. Fetal position presenting anteroposterior
diameter is the
suboccipitobregmatic
FETAL LIE (9.5cm at term)
occipital/ posterior
- Relation of the fetal long axis to that of the (triangular) fontanel is the
mother presenting part
- It can be: longitudinal, transverse, or oblique
b. Face presentation
fetal neck is hyperextended
1. Longitudinal Lie so that the occiput and back
— present in >99% of labors at term come in contact, and the face
— Long axis of the fetus parallel to the is foremost in the birth canal
long axis of uterus presenting AP diameter is
As term approaches, the uterine cavity submentobregmatic or
most often accommodates the fetus in tracheobregmatic (9.5cm at
longitudinal lie with either fetal head term)
(cephalic) or breech presenting allows advancement through
pelvis but vaginal delivery
2. Transverse Lie may result to cervical spinal
— Perpendicular to the long axis of the cord injury
mother chin/ mentum presenting
— Predisposing factors: multiparity, part
placenta previa, hydramnios, uterine
anomalies c. Sinciput presentation
— The shoulder is usually over the pelvic Partially flexed fetal head
inlet with the fetal head lying in one iliac Anterior (large diamond
fossa and the breech in the other shaped) fontanel or bregma
— Less common is the presenting part
— Potentially serious; when membranes Military attitude
rupture, cord prolapse commonly follows AP diameter is
— Indication for cesarean section because occipitofrontal (12.5cm at
labor will cause a ruptured uterus term)

3. Oblique Lie d. Brow presentation


— Fetal and maternal axes may cross at a Partially extended fetal
45-degree angle head
— Unstable and becomes longitudinal and AP diameter is
transverse during labor occipitomental plane
(13.5cm at term; longest AP
diameter)
FETAL PRESENTATION Frontum/ Frontal bone
and orbits presenting part
- Presenting part portion of the fetal body
that is either foremost within the birth canal The sinciput and brow
or in close proximity to it presentations are usually
- Typically can be felt thought the cervix on transient. As labor progresses,
vaginal examination both presentations almost
In longitudinal lie: presenting part is always convert into vertex and
either fetal head (cephalic) or breech face presentations by neck
presentation flexion and extension,
In transverse lie: shoulder is the respectively.
presenting part o Vertex neck flexion
o Face neck extension

1
o Failure to do so can Peculiarity in fetal attitude,
lead to dystocia particularly extension of the
vertebral column as seen in
frank breeches may also
prevent fetus from turning
Implantation of placental in
lower uterine segment which
may distort normal
intrauterine anatomy

a. Frank breech presentation


Lower extremities are flexed
at the hips and extended at
the knees
Fetal feet lie in close
proximity to the head

Term fetus usually presents


with vertex because uterus is
piriform or pear shaped

Cephalic pole
composed of fetal head only
slightly larger than the
breech b. Complete breech
presentation
Podalic Pole One or both knees are
Breech and flexed flexed
extremities Thighs are flexed over the
abdomen, legs are flexed
bulkier and more mobile
upon the thighs and the feet
than the cephalic pole present at the level of the
buttocks

At 32 weeks, amniotic cavity is


large compared with the fetal
mass and the fetus is not
crowded by the uterine walls.

As fetal mass increases, the


ratio of amniotic fluid volume
decreases results to the
uterine walls more apposed
closely to the fetal parts
c. Incomplete breech
presentation
If presenting part is breech, One or both hips are not
fetus most often changes flexed, and one or both feet
polarity to make use of the or knees lie below the
roomier fundus for its bulkier breech, such that a foot or
and more mobile podalic knee is lowermost in the
pole Incidence of breech birth canal
presentation decreases with Possible complication:
gestational age compression of a prolapsed
cord or a cord entangled
2. Breech Presentation around the extremities as
— Considered when fetus presents the breech fills the pelvis
with buttocks towards the pelvis and
the bitrochanteric diameter
(9.5cm) presents
— Presented by the sacral prominence
— 3 general presentations: frank,
complete and footling presentations
— May result from circumstances that
prevent normal version from taking
place such as:
Septum that protrudes into
the uterine cavity

2
Footling breech an FETAL POSITION
incomplete breech with one or
both feet below the breech - Relationship of an arbitrarily chosen portion
Single footling one leg is of the fetal presenting part to the right or left
completely extended and the side of the birth canal (According to
other is flexed Williams)
Double footling both legs According to APMC, fetal
are extended below the level of position is the relationship of
the buttocks the chosen portion of the fetal
presenting part in reference to
3. Shoulder Presentation one of the four quadrants or to
— Shoulder or acromion is presenting the transverse diameter of the
into the pelvic inlet in transverse lie maternal birth canal
— Bisacromial diameter (11.0cm) - For each presentation there may be 2
presents positions right or left
- Determining points:
Vertex presentation occiput
Face presentation chin (mentum)
Breech presentation sacrum
Presenting part may either be
right or left position
Left occipital (LO) and right
occipital (RO)
Left mental (LM) and right
mental (RM)
4. Compound Presentation Left Sacral (LS) and right
— Fetal hand or foot prolapses sacral (RS)
alongside the presenting vertex or
breech
— Causes: conditions that prevent VARIETIES OF PRESENTATIONS AND
complete occlusion of the pelvic POSITIONS
inlet by the presenting part
Combination of hand with - Relationship of a given position of the
vertex or breech tends to presenting part to the anterior, transverse, or
resolve spontaneously as posterior portion of the maternal pelvis for
labor advances more accurate orientation
Combination of foot and - Presenting part in right or left positions may
fetal head tends to be be directed anteriorly (A), transversely (T),
complicated with cord or posteriorly (P)
prolapse - There are 6 varieties for each of the 3
presentations
In an occiput presentation, the
FETAL ATTITUDE or POSTURE presentation, position and variety
may be abbreviated in clockwise
- Relation of fetal parts to one another fashion as:
- The fetus forms an ovoid mass that
corresponds roughly to the shape of the
uterine cavity
- The fetus becomes folded or bent; the back
becomes markedly convex; the head is
flexed with the chin in contact with the chest;
the thighs are flexed over the abdomen; legs
are bent at the knees; arches of feet rest
upon the anterior surface of the legs this
characteristic feature results partly from the Acromion (scapula)
mode of growth of the fetus and partly from portion of the fetus arbitrarily
a process of accommodation to the uterine chosen for orientation with the
cavity maternal pelvis in shoulder
- In cephalic presentations, arms are usually presentations
crossed over the thorax or become parallel may be directed wither posteriorly
to the sides. or anteriorly and superiorly and
- The umbilical cord lies in the space between
inferiorly
the arms and lower extremities.
Abnormal exceptions to this All transverse lies are referred to as
attitude: fetal head becomes shoulder presentations
more extended from vertex to Transverse lie, with back up or back
face presentation resulting down, is clinically important when
from change in fetal attitude deciding incision type for cesarean
from convex (flexed) to delivery
concave (extended) contour of
the vertebral column.

3
1. Abdominal Palpation Le d
Maneuver
— Conducted systematically employing
4 maneuvers
— Mother lies supine and comfortable
positioned with her abdomen bared
— May be difficult to perform in obese
patient, excessive amniotic fluid or if
placenta is implanted anteriorly
— Can be performed throughout the
latter months of pregnancy and
during and between contractions of
labor
— Can be used to estimate size of
fetus

a. LM 1 (Fundic Grip)
Identification of which fetal
pole (either cephalic or
podalic) occupies uterine
fundus
Breech felt as large,
nodular mass
Head felt as hard and
round and is more mobile
and ballotable

b. LM 2 (Umbilical Grip)
Palms are placed on either
side of the maternal
abdomen, and gentle but
deep pressure is exerted
Fetal back felt as hard
and resistant structure
Fetal extremities felt as
numerous small, irregular
mobile parts
Identify if fetal back is on the
right or left side and where
the fetal heart sound is best
appreciated
Fetal orientation
can be determined
by noting whether
the back is directed
anteriorly,
transversely or
posteriorly

c. LM 3 (Pa c/ Pa G )
Performed by grasping with
the thumb and fingers on
one hand the lower portion
of the maternal abdomen
just above the symphysis
pubis
To determine what is
occupying the pelvic inlet
If presenting part is not
engaged movable mass is
felt (usually the head
DIAGNOSIS OF FETAL PRESENTATION AND
POSITION d. LM 4 (Pelvic Grip)
Examiner faces the
- Several methods may be used: abdominal mo her fee
palpation, vaginal examination, auscultation Tips of the first 3 fingers of
and sonography each hand are used in
- Rarely used: radiographs, computed exerting deep pressure in
tomography or MRI the direction of the axis in
the pelvic inlet

4
Determine whether head is — Determines vertex presentations
extended or flexed and positions through palpation of
If cephalic various fetal sutures and fontanels
prominence is on — Face and breech presentations
the same side as identified by palpation of facial
the fetal small features and fetal sacrum
parts head is — Comprises 4 movements:
st
flexed and is in 1 : Examiner inserts 2
vertex presentation fingers into the vagina and
If cephalic presenting part is found
prominence is on Differentiation of
the same side as vertex, face and
the fetal back breech is readily
head is extended accomplished
nd
and face is the 2 : If vertex presentation,
presenting part fingers are directed
Tells us if there is posteriorly and then swept
engagement forward over the fetal head
Difficult to assess toward the maternal
engagement if symphysis
breech presentation During this
If presenting part is movement, the
not engaged can fingers necessarily
be determined if cross the sagittal
both hands meet, suture and its linear
hands should have course is delineated
rd
a meeting point or 3 : The positions of the 2
they should fontanels are ascertained;
intersect at one Fingers are passed to the
point; cephalic most anterior extension of
prominence is on the sagittal suture and the
the same side as fontanel encountered there
that of the fetal back is examined and identified.
Deeply engaged Then with a sweeping
indicative that the motion, the fingers pass
lower fetal pole is in along the suture to the other
the pelvis; cephalic end of the head until the
prominence is no other fontanel is felt and
longer palpable on differentiated
th
the ipsilateral side 4 : Established the station
as that of the fetal or extent to which the
back presenting part has
descended into the pelvis
Various sutures and
fontanels are
located readily
using these
maneuvers.

2. Vaginal Examination
— Inconclusive in diagnosing fetal
presentation and position when
done before labor because
palpation is done through closed
cervix and lower uterine segment
— Usually done during onset of labor
and after cervical dilatation

5
Examples:

1. Right Occiput Transverse (ROT)


LM 1: Breech
3. Auscultation LM 2: fetal back right (FBR)
- does not provide very reliable information LM 3: Cephalic
concerning fetal presentation and position LM 4: Cephalic prominence (CP) is
- fetal heart sounds best heard through fetal on the contralateral side
back
- The point of maximal intensity of fetal heart
sounds in cephalic presentation is usually Unless the examiner tells you that her
midway between the maternal umbilicus and fingers did not meet, there is (-) engagement
the anterior superior spine of the ilium
- Breech presentation above the level of the
umbilicus

4. Sonography and Radiography


- Can aid fetal position identification,
especially in obese women or in women with
rigid abdominal walls

DETERMINING FETAL POSITION:

2. Left Mentum Posterior (LMP)


LM 1: Breech
LM 2: FBR
LM 3: Cephalic
LM 4: CP is ipsilateral to fetal back;
(-) engagement

*T e a e e e a
facing the examiner

A Anterior

P Posterior

R Right (per ain o he mo her right, not


he e aminer )

L Left (per ain o he mo her lef , no


3. Left Sacral Anterior (LSA)
he e aminer ) LM 1: Cephalic
LM 2: Fetal back left
T Transverse
LM 3: Breech
LM 4:

6
If heart tones is heard above the umbilicus
breech presentation

If heart tones is heard below the umbilicus


cephalic presentation

Asynclitism

- Lateral deflection of the sagittal suture to a


more anterior or posterior position in the
4. Left Acromio-dorso posterior pelvis
LM 1: (-) - Anterior asynclitism if sagittal suture
LM 2: Head left; buttocks right approaches the sacral promontory, more of
LM 3: (-) the anterior parietal bone presents itself to
LM 4: (-) the examining fingers
- Posterior asynclitism sagittal suture lies
close to the symphysis, more of the
posterior parietal bone will present

5. Right Acromio-dorso anterior (RADA)


LM 1: (-)
LM 2: Head right; buttocks left
LM 3: (-)
LM 4: (-) Passenger factors that affect vaginal delivery:

1. Presentation: cephalic vaginal; breech


On Internal Exam: abdominal delivery
2. Varieties of cephalic presentation: face
Breech presentation usually abdominal delivery
- If there is difficult in reaching for the part 3. Multiple gestation
- Soft area 4. Size of the baby
- If soles of the feet is felt 5. Maturity of the baby
- If straight line is felt between the ischial Preterm disproportion of the size
tuberosity and anal opening of the head and chest; better to
- If finger is put is put inside anal opening, deliver abdominally
there is meconium

Face or Cephalic Presentation


- If one side of the triangle is formed when
palpating for the malar eminence and the Sources: Williams, APMC, Group discussion
mouth
- When finger is put inside the mouth, the
baby sucks the fingers

For compound presentations wherein the


hand goes over the head
- Longer fingers are felt
- There is grasp reflex

Compound presentation is differentiated


from breech by:
- Shape of the soles are different clumpier
compared to palm of the hands
- There i fanning (babin ki ign) hen ole
of the feet is stroked

7
Fetal Contributions to Initiation of Parturition o considered a critical component of
normal parturition.
mature human fetus provides the signal to initiate o
parturition preterm labor
o signals transmitted in several ways to As in the sheep, steroid products of the human
suspend uterine quiescence. fetal adrenal gland
fetus may provide a signal thru a blood-borne o (+) effects on the placenta and
agent that acts on the placenta membranes
o Although signals may arise from the o transform myometrium from a quiescent
fetus to contractile state.
the uterus and cervix likely first A key component in the human - unique ability of
must be prepared for labor
before production & release of
uterotonin
Placental Corticotropin-Releasing Hormone (CRH)
Production.
Uterine Stretch and Parturition
A CRH hormone identical to maternal and fetal
Fetal Growth important component in uterine hypothalamic CRH
activation (Phase 1 of Parturition) o synthesized by the placenta in relatively
o Significant myometrial tensile stress large amounts
& AF pressure flow One important difference is that, unlike
o (+) uterine activation hypothalamic CRH, which is under GC (-
stretch is required for induction )feedback
of specific contraction- o cortisol - stimulate placental CRH
associated proteins (CAPs) production
stretch - increases expression o thru activation of the transcription factor,
of the gap junction protein nuclear factor kappa B (NF- B)
connexin 43 and of o This ability create a feed-forward
oxytocin receptors endocrine cascade that does not end
Gastrin-releasing peptide until delivery
st
a stimulatory agonist
for smooth muscle (from midgestation to term)
increased by stretch in o In the last 12 weeks, CRH plasma levels
the myometrium rise exponentially
Other Hypothesis: stretch plays an integrated o peaking during labor and then falling
role with fetal- maternal endocrine cascades precipitously after delivery
of uterine activation
Clinical support for a role of stretch CRH the only trophic hormone-releasing factor
o multifetal pregnancies - greater risk for to have a specific serum binding protein.
preterm labor than singletons o During most of pregnancy, CRH-binding
preterm labor - common in protein (CRH-BP) binds most maternal
pregnancies complicated by
hydramnios o -BP
o (+) role for uterine stretch is considered levels in both maternal plasma and
Mechano-Transduction cell signaling systems
used by stretch to regulate the myometrial cell levels of bioavailable CRH
that includes: In pregnancie in hich he fe (+) re ed
activation of cell-surface receptors or ion from various complications
channels o
transmission of signals through maternal plasma
extracellular matrix o as compared with those seen in normal
release of autocrine molecules that act gestation
directly on myometrium o placenta -
For example, Fibronectin (EC matrix protein) + - concentration
5 integrin receptor (its cell-surface receptor) For example, women with
o induced in the rodent by stretch preeclampsia = 4x placental
o aid force transduction during labor CRH content
contraction by anchoring hypertrophied than in those from normal
myocytes to the uterine EC matrix. pregnancies

ge a ion & ecre ion of placen al CRH in


complicated pregnancies
Fetal Endocrine Cascades Leading to Parturition
o may play a role in fetal adrenal cortisol
synthesis
(+) ability of the fetus to provide endocrine o may result in the supranormal levels of
signals initiate parturition umbilical cord blood cortisol
Liggins and associates (1967, 1973) used as noted in stressed neonates
sheep Corticotropin-Releasing Hormones and Parturition Timing
o signal comes from the fetal
hypothalamic pituitary adrenal Placental CRH (+) play several roles in
(HPO) axis parturition regulation
o exact mechanisms regulating human o
parturition - more difficult to prove
o all evidence humans are not regulated Late in pregnancy (phase 2 or 3 of parturition)
in the exact manner seen in the sheep o
Even so, activation of the human fetal
hypothalamic pituitary adrenal placental myometrial cell Ca levels via PKC
axis (protein kinase C) activation
1
Oxytocin - acts to attenuate CRH-stimulated Thus, a fetal signal through the paracrine arm of
accumulation of cAMP in myometrial tissue the fetal-maternal communication system does
CRH augments the contraction-inducing potency not appear to be mandated for parturition
of a given dose of oxytocin in human myometrial initiation
strips Some brain anomalies of the fetal calf, fetal
o CRH - lamb, and sometimes the human fetus
response to PGF2 o delay the normal timing of parturition.
o (+) fetal anencephaly = prolonged
o 19- human gestation (374 days or 53
steroid synthesis weeks)
attributable to anomalous fetal
brain-pituitary-adrenal function.
production
Very small adrenal
(+) shifting of the glands and only 5-
estrogen-to- 10% as large of those
progesterone ratio of a normal fetus at
promote the term
expression of a series caused by developmental
of myometrial failure of the fetal zone
contractile proteins. accounts for most of
o rising level of CRH at the end of gestation reflects fetal adrenal mass
a fetal-placental clock and production of
o CRH levels vary greatly among women C19-steroid hormones
o -
more accurate predictor of pregnancy Such pregnancies are associated with delayed
outcome than is a single measurement labor and suggest that the fetal adrenal glands
o In this regard, the placenta and fetus, through are important for the timely onset of parturition
endocrinological events, influence the timing of
parturition at the end of normal gestation.
Phase 3: Uterine Stimulation

Fetal Lung Surfactant and Parturition synonymous with uterine contractions


o that bring about progressive cervical
Surfactant protein A (SP-A) - produced by the dilatation and delivery.
fetal lung uterotonin theory of labor initiation
o required for lung maturation o Phase 1 suspension & Phase 2
o im
- production
A concentrations in AF = (+) o # of uterotonins (important) = success of
phase 3 (Active labor)
migration into the myometrium Uterotonins - candidates for labor induction
and induce NF- B includes:
NF- B - activates inflammatory o oxytocin, prostaglandins, serotonin,
response genes in the histamine, PAF, angiotensin II, and
myometrium many others.
promote uterine o (+) stimulate smooth muscle contraction
contractility through G-protein coupling
This model supports the
supposition that fetal signals
play a role in parturition Oxytocin and Phase 3 of Parturition
initiation
SP-A is expressed by the human amnion and Late in pregnancy, during phase 2 of parturition
decidua o 50-
o Present in AF (amniotic fluid) oxytocin receptors
o prompts signaling pathways in human o
myometrial cells responsiveness to oxytocin.
The exact mechanisms by which SP-A o Moreover, prolonged gestation = delay
activates myometrial contractility in in the increase of these receptors
women, however, remains to be clarified Oxytocin q ick bir h
st
SP-A selectively inhibits PGF2 in the term o 1 uterotonin to be implicated in
decidua & AF concentration of SP- parturition initiation
Fetal Anomalies and Delayed Parturition o nanopeptide, synthesized as a
prohormone in the magnocellular
neurons of the SON (supraoptic) & PVN
= prolonged gestation (paraventricular).
o The e na ral e perimen incl de o transported with its carrier protein,
fetal anencephaly with adrenal neurophysin
hypoplasia and those with inherited along the axons to the neural
placental sulfatase deficiency. lobe of the posterior pituitary
o The broad range of gestational length gland in membrane-bound
seen with these disorders calls into vesicles for storage and later
question the exact role of estrogen in release.
human parturition initiation. o converted enzymatically to oxytocin
during transport
Other fetal abnormalities do not prolong human
pregnancy
o
entry of fetal urine into AF Role of Oxytocin in Phase 3 and 4 of Parturition
o
secretions

2
logically suspected in parturition initiation. o 2 in the
First, in addition to its effectiveness in inducing decidua at term
labor at term regulatory step in PG action in
o oxytocin is a potent uterotonin and the uterus.
occurs naturally in humans. myometrium synthesizes PGHS-2 with labor
Subsequent observations provide additional onset
support for this theory: o most PG likely comes from the decidua
The fetal membranes and placenta also produce
and decidual tissues near the end of gestation PGs
(2) oxytocin acts on decidual tissue to o Primarily PGE2, but also PGF2
promote prostaglandin release o detected in AF at all gestational stages
(3) synthesized directly in decidual and
extraembryonic fetal tissues and in the placenta o
Although little evidence suggests a role for demonstrable after labor begins
oxytocin in phase 2 of parturition o results to the dilatation of the cervix &
o abundant data support its important role decidual tissue exposure
during second-stage labor & in the o
puerperium - phase 4 of parturition those in the upper compartment
Specifically, there are follow an inflammatory
levels:
(1) during second-stage labor - end of labor.
o Together, the increases in cytokines and
phase 3 of parturition
prostaglandins
(2) in the early puerperium degrade the EC matrix =
(3) during breast feeding weakening of fetal membranes.
Immediately after delivery of the fetus, placenta, o inflammatory mediators
and membranes aid cervical dilatation
o Completion of parturition phase 3 alterations to the lower uterine
o (+) firm and persistent uterine segment
contractions and myometrial retraction It can be envisioned that they, along
with the increased prostaglandin levels
hemorrhage measured in vaginal fluid during labor,
o Oxytocin - causes persistent add to the relatively rapid cervical
contractions changes that are characteristic of
o Oxytocin infusion in women promotes parturition.

encode proteins essential for Endothelin-1


uterine involution
These include interstitial family of 21-amino acid peptides
collagenase, monocyte o powerfully induce myometrial
chemoattractant protein-1, IL-8, contraction
and urokinase plasminogen endothelin A receptor - preferentially expressed
activator receptor.
Therefore, oxytocin action at Endothelin-1
the end of labor may be o produced in myometrium of term
involved in uterine involution gestations
Prostaglandins and Phase 3 of Parturition o able to induce synthesis of other
contractile mediators
(+) critical role in phase 3 of parturition such as PG and inflammatory
o mediators
noncomplicated pregnancies
First, levels of prostaglandins, or their metabolites
(+) requirement of endothelin-1 in normal
o In amnionic fluid, maternal plasma, and parturition physiology
maternal urine However, there is evidence of pathologies
Second, Tx of pregnant women with PG = associated with aberrant endothelin-1 expression
abortion or labor at all gestational stages o such as premature birth and uterine
o At any route of administration leiomyomas
(+) giving of prostaglandin H synthase type 2 Angiotensin II
(PGHS-2) inhibitors to pregnant women
o Delay onset of spontaneous labor & may 2 G-protein-linked angiotensin II receptors
arrest preterm labor expressed in the uterus: AT1 and AT2
Last, PG Tx of myometrial tissue in vitro o nonpregnant women - AT2 receptor is
o causes contraction predominant
o dependent on the prostanoid tested & o
physiological status of the tissue treated. expression
Angiotensin II binding to the plasma-membrane
receptor = contraction
Uterine Events Regulating Prostaglandin Production. During pregnancy, vascular smooth muscle (+)
expression of AT2 receptor
During labor, PG production within the o refractory to the pressor effects of
myometrium and decidua infused angiotensin II
o efficient mechanism of activating In myometrium near term, angiotensin II may be
contractions. another component of the uterotonin system of
o For example, PG synthesis is high and parturition phase 3
unchanging in the decidua
during phase 2 and 3 of
parturition

3
Contribution of Intrauterine into the maternal compartment
to promote myometrial
Tissues to Parturition contractility
progesterone - maintains chorion PGDH
the amnion, chorion laeve, and decidua parietalis expression
o have a potential role in parturition cortisol - decreases PGDH expression.
initiation o
o plays an alternative role levels
The membranes and decidua part of progesterone withdrawal
o make up an important tissue shell
around the fetus
o serves as a physical, immunological, Decidua
and metabolic shield
to protect against untimely A metabolic contribution of decidual activation to
initiation of parturition. parturi- tion initiation is an appealing possibility
Late in gestation, however, the fetal membranes for both anatomical and functional reasons.
may indeed act to prepare for labor. (+) generation of decidual uterotonins
o paracrine action on contiguous
myometrium is intuitive.
Amnion (+) steroid metabolizing enzymes expression
o 20 -HSD and eroid 5 R1
Provides virtually all of the tensile strength of the may regulate local
fetal membrane progesterone withdrawal
o (+) resistance to tearing and rupture Decidual activation is characterized by:
avascular tissue o increased proinflammatory cells
highly resistant to penetration by leukocytes, o increased expression of proinflammatory
microorganisms, and neoplastic cells cytokines, PG and uterotonins
constitutes a selective filter Uterotonins such as oxytocin
o to prevent fetal particulate-bound lung receptors and connexin 43
and skin secretions from reaching the Cytokines produced in the decidua
maternal compartment o increase uterotonin production
o maternal tissues are protected from (principally prostaglandins)
amnionic fluid constituents o act directly on myometrium to cause
contraction.
myometrial function o Examples: tumor necrosis factor-
(TNF- ) and in erle kin 1, 6, 8, and 12.
amnionic-fluid embolism also can act as chemokines
recruit to the
myometrium
Several bioactive peptides and prostaglandins neutrophils and
that cause myometrial relaxation or contraction eosinophils
are synthesized in amnion further increase
landin contractions and labor
There is uncertainty whether prostaglandin
2 and PGHS-2 concentration or output from the decidua
Accordingly, many hypothesize that PG regulate increases with term labor onset.
events leading to parturition. Olson and Ammann (2007) suggest that the
Amnion - major source for AF prostaglandins major regulation of decidual prostaglandin action
o (+) role in activation of cascades = is not their synthesis but rather increased PGF2
promote membrane rupture
o n-derived PG on receptor expression.
uterine quiescence & activation
due to PG transport from the
amnion thru the chorion to Summary: Regulation of Phase 3 and 4 of Parturition
access maternal tissues
limited by expression of the It is likely that multiple and possibly redundant
inactivating enzyme, processes contribute to the success of the three
prostaglandin dehydrogenase. active labor phases once phase 1 of parturition is
suspended and phase 2 is initiated.
Phase 3
Chorion Laeve o -protein-
coupled receptors
primarily protective tissue layer
(+) immunological acceptance
enriched with enzymes that inactivate (+)interaction of actin and
uterotonins: myosin = force generation.
o prostaglandin dehydrogenase (PGDH) - Simultaneously, (+) alteration of cervical
inactivates amnion-derived PG proteoglycan composition and collagen structure
o oxytocinase o promotes tissue distensibility and
o enkephalinase increased compliance.
(+) chorionic rupture = NET RESULT:
adjacent decidua and myometrium o initiation of coordinated myometrial
contractions
labor. with sufficient amplitude and
o - stimulated matrix frequency
metalloproteinase activity = cervical dilatation & pushing
associated with membrane of fetus thru birth canal
rupture Multiple regulatory ligands orchestrate these
o It would further allow prostaglandin entry processes and vary from endocrine hormones

4
such as oxytocin to locally produced
prostaglandins.
In phase 4 of parturition
o Initiation of a complicated series of
repair processes
to resolve inflammatory
responses
to remove glycosaminoglycans,
proteoglycans, and structurally
compromised collagen.
Simultaneously, (+) synthesis of matrix and
cellular components = complete uterine
involution. (+) reformation of dense connective
tissue and structural integrity of the cervix

5
OB1 Biochemical Aspects of Parturition (Part II) Fetal Endocrine Cascades Leading to Parturition
th
- from 24 Ed. OB. Williams (+) ability of the fetus to provide endocrine
signals initiate parturition
Liggins and associates (1967, 1973) used
Fetal Contributions to Initiation of Parturition sheep
o signal comes from the fetal
mature human fetus provides the signal to hypothalamic pituitary adrenal
initiate parturition (HPO) axis
o signals transmitted in several ways o exact mechanisms regulating
to suspend uterine quiescence. human parturition - more difficult to
fetus may provide a signal thru a blood- prove
borne agent that acts on the placenta o all evidence humans are not
o Although signals may arise from the regulated in the exact manner seen
fetus in the sheep
the uterus and cervix likely Even so, activation of the human fetal
first must be prepared for hypothalamic pituitary adrenal
labor before production & placental axis
release of uterotonin o considered a critical component of
normal parturition.
o
Uterine Stretch and Parturition preterm labor
As in the sheep, steroid products of the
Fetal Growth important component in human fetal adrenal gland
uterine activation (Phase 1 of Parturition) o (+) effects on the placenta and
o Significant myometrial tensile membranes
stress & AF pressure flow o transform myometrium from a
o (+) uterine activation quiescent to contractile state.
stretch is required for A key component in the human - unique
induction of specific
contraction-associated
proteins (CAPs)
stretch - increases Placental Corticotropin-Releasing Hormone
expression of the gap
(CRH) Production.
junction protein
connexin 43 and of
A CRH hormone identical to maternal and
oxytocin receptors
fetal hypothalamic CRH
Gastrin-releasing peptide
o synthesized by the placenta in
a stimulatory
relatively large amounts
agonist for smooth
One important difference is that, unlike
muscle
hypothalamic CRH, which is under GC (-
increased by stretch
)feedback
in the myometrium
o cortisol - stimulate placental CRH
Other Hypothesis: stretch plays an
production
integrated role with fetal- maternal
o thru activation of the transcription
endocrine cascades of uterine activation
factor, nuclear factor kappa B (NF-
Clinical support for a role of stretch
B)
o multifetal pregnancies - greater risk
o This ability create a feed-forward
for preterm labor than singletons
endocrine cascade that does not
preterm labor - common in
end until delivery
pregnancies complicated by st
Tri)
hydramnios
o (+) role for uterine stretch is
o In the last 12 weeks, CRH plasma
considered
levels rise exponentially
Mechano-Transduction cell signaling
o peaking during labor and then falling
systems used by stretch to regulate the
precipitously after delivery
myometrial cell that includes:
activation of cell-surface receptors
CRH the only trophic hormone-releasing
or ion channels
factor to have a specific serum binding
transmission of signals through
protein.
extracellular matrix
o During most of pregnancy, CRH-
release of autocrine molecules that
binding protein (CRH-BP) binds
act directly on myometrium
For example, Fibronectin (EC matrix protein)
inactivation
+ -5 integrin receptor (its cell-surface
o -BP
receptor)
levels in both maternal plasma and
o induced in the rodent by stretch
o aid force transduction during labor
levels of bioavailable CRH
contraction by anchoring
In pregnancies in which the fetus (+)
hypertrophied myocytes to the
re ed from ario complica ion
uterine EC matrix.
o
1
and maternal plasma o required for lung maturation
o as compared with those seen in o
normal gestation
o placenta - SP-A concentrations in AF =
concentration (+) fluid macrophages
For example, women with
preeclampsia = 4x placental the myometrium and induce
CRH content NF- B
than in those from normal NF- B - activates
pregnancies inflammatory response
genes in the myometrium
ge a ion & ecre ion of placen al CRH promote uterine
in complicated pregnancies contractility
o may play a role in fetal adrenal This model supports the
cortisol synthesis supposition that fetal signals
o may result in the supranormal levels play a role in parturition
of umbilical cord blood cortisol initiation
as noted in stressed SP-A is expressed by the human amnion
neonates and decidua
o Present in AF (amniotic fluid)
o prompts signaling pathways in
Corticotropin-Releasing Hormones and Parturition human myometrial cells
Timing The exact mechanisms by which
SP-A activates myometrial
Placental CRH (+) play several roles in contractility in women, however,
parturition regulation remains to be clarified
o may enhance fetal cortisol SP-A selectively inhibits PGF2 in the term
decidua & AF concentration of SP-
placental CRH term
Late in pregnancy (phase 2 or 3 of
parturition)
o (+) modification in the CRH receptor Fetal Anomalies and Delayed Parturition

myometrial cell Ca levels via PKC


(protein kinase C) activation production = prolonged gestation
Oxytocin - acts to attenuate CRH-stimulated o The e na ral e perimen incl de
accumulation of cAMP in myometrial tissue fetal anencephaly with adrenal
CRH augments the contraction-inducing hypoplasia and those with inherited
potency of a given dose of oxytocin in placental sulfatase deficiency.
human myometrial strips o The broad range of gestational
o CRH - length seen with these disorders
in response to PGF2 calls into question the exact role of
estrogen in human parturition
o 19-
initiation.
steroid synthesis
Other fetal abnormalities do not prolong
ns human pregnancy
production o
(+) shifting of the
AF
estrogen-to-
o
progesterone ratio
secretions
promote the
Thus, a fetal signal through the paracrine
expression of a
arm of the fetal-maternal communication
series of myometrial
system does not appear to be mandated for
contractile proteins.
parturition initiation
o rising level of CRH at the end of gestation
reflects a fetal-placental clock Some brain anomalies of the fetal calf, fetal
o CRH levels vary greatly among women lamb, and sometimes the human fetus
o the rat - o delay the normal timing of
more accurate predictor of parturition.
pregnancy outcome than is a single o (+) fetal anencephaly = prolonged
measurement human gestation (374 days or 53
o In this regard, the placenta and fetus, weeks)
through endocrinological events, influence attributable to anomalous
the timing of parturition at the end of normal fetal brain-pituitary-adrenal
gestation. function.
Very small adrenal
glands and only 5-
Fetal Lung Surfactant and Parturition 10% as large of
those of a normal
fetus at term
Surfactant protein A (SP-A) - produced by
caused by developmental
the fetal lung
2
failure of the fetal zone occurs naturally in humans.
accounts for most Subsequent observations provide additional
of fetal adrenal support for this theory:
mass and
production of C19- myometrial and decidual tissues near the
steroid hormones end of gestation
Such pregnancies are associated with (2) oxytocin acts on decidual tissue to
delayed labor and suggest that the fetal promote prostaglandin release
adrenal glands are important for the timely
(3) synthesized directly in decidual and
onset of parturition
extraembryonic fetal tissues and in the
placenta
Phase 3: Uterine Stimulation Although little evidence suggests a role for
oxytocin in phase 2 of parturition
synonymous with uterine contractions o abundant data support its important
o that bring about progressive cervical role during second-stage labor & in
dilatation and delivery. the puerperium - phase 4 of
uterotonin theory of labor initiation parturition
o Phase 1 suspension & Phase 2
oxytocin levels:
production (1) during second-stage labor - end
o # of uterotonins (important) = of phase 3 of parturition
success of phase 3 (Active labor) (2) in the early puerperium
Uterotonins - candidates for labor induction (3) during breast feeding
includes:
Immediately after delivery of the fetus,
o oxytocin, prostaglandins, serotonin,
placenta, and membranes
histamine, PAF, angiotensin II, and
o Completion of parturition phase 3
many others.
o (+) firm and persistent uterine
o (+) stimulate smooth muscle
contractions and myometrial
contraction through G-protein
retraction
coupling
hemorrhage
o Oxytocin - causes persistent
Oxytocin and Phase 3 of Parturition contractions
o Oxytocin infusion in women
Late in pregnancy, during phase 2 of
parturition myometrial genes
o 50- encode proteins essential
myometrial oxytocin receptors for uterine involution
o ne These include interstitial
contractile responsiveness to collagenase, monocyte
oxytocin. chemoattractant protein-1,
o Moreover, prolonged gestation = IL-8, and urokinase
delay in the increase of these plasminogen activator
receptors receptor.
Oxytocin q ick bir h Therefore, oxytocin action at
st
o 1 uterotonin to be implicated in the end of labor may be
parturition initiation involved in uterine involution
o nanopeptide, synthesized as a
prohormone in the magnocellular
neurons of the SON (supraoptic) & Prostaglandins and Phase 3 of Parturition
PVN (paraventricular).
o transported with its carrier protein, (+) critical role in phase 3 of parturition
neurophysin o
along the axons to the noncomplicated pregnancies
neural lobe of the posterior
First, levels of prostaglandins, or their
pituitary gland in
membrane-bound vesicles
o In amnionic fluid, maternal plasma,
for storage and later
and maternal urine
release.
Second, Tx of pregnant women with PG =
o converted enzymatically to oxytocin
abortion or labor at all gestational stages
during transport
o At any route of administration
(+) giving of prostaglandin H synthase type 2
(PGHS-2) inhibitors to pregnant women
Role of Oxytocin in Phase 3 and 4 of Parturition
o Delay onset of spontaneous labor &
may arrest preterm labor
Last, PG Tx of myometrial tissue in vitro
logically suspected in parturition initiation.
o causes contraction
First, in addition to its effectiveness in
o dependent on the prostanoid tested
inducing labor at term
& physiological status of the tissue
o oxytocin is a potent uterotonin and
treated.
3
Uterine Events Regulating Prostaglandin Angiotensin II
Production.
2 G-protein-linked angiotensin II receptors
During labor, PG production within the expressed in the uterus: AT1 and AT2
myometrium and decidua o nonpregnant women - AT2 receptor
o efficient mechanism of activating is predominant
contractions. o
o For example, PG synthesis is high expression
and unchanging in the decidua Angiotensin II binding to the plasma-
during phase 2 and 3 of membrane receptor = contraction
parturition During pregnancy, vascular smooth muscle
o 2 in the (+) expression of AT2 receptor
decidua at term o refractory to the pressor effects of
regulatory step in PG action infused angiotensin II
in the uterus. In myometrium near term, angiotensin II
myometrium synthesizes PGHS-2 with labor may be another component of the uterotonin
onset system of parturition phase 3
o most PG likely comes from the
decidua
The fetal membranes and placenta also Contribution of Intrauterine
produce PGs
o Primarily PGE2, but also PGF2 Tissues to Parturition
o detected in AF at all gestational
the amnion, chorion laeve, and decidua
stages
parietalis
o have a potential role in parturition
o
initiation
demonstrable after labor begins
o plays an alternative role
o results to the dilatation of the cervix
The membranes and decidua
& decidual tissue exposure
o make up an important tissue shell
o
around the fetus
with those in the upper compartment
o serves as a physical, immunological,
follow an inflammatory
and metabolic shield
to protect against untimely
labor.
initiation of parturition.
o Together, the increases in cytokines
and prostaglandins Late in gestation, however, the fetal
membranes may indeed act to prepare for
degrade the EC matrix =
labor.
weakening of fetal
membranes. Amnion
o inflammatory mediators
aid cervical dilatation Provides virtually all of the tensile strength of
alterations to the lower the fetal membrane
uterine segment o (+) resistance to tearing and
It can be envisioned that they, along rupture
with the increased prostaglandin avascular tissue
levels measured in vaginal fluid highly resistant to penetration by leukocytes,
during labor, add to the relatively microorganisms, and neoplastic cells
rapid cervical changes that are constitutes a selective filter
characteristic of parturition. o to prevent fetal particulate-bound
Endothelin-1 lung and skin secretions from
reaching the maternal compartment
family of 21-amino acid peptides o maternal tissues are protected from
o powerfully induce myometrial amnionic fluid constituents
contraction
endothelin A receptor - preferentially myometrial function

Endothelin-1 amnionic-fluid embolism


o produced in myometrium of term
gestations
o able to induce synthesis of other Several bioactive peptides and
contractile mediators prostaglandins that cause myometrial
such as PG and relaxation or contraction are synthesized in
inflammatory mediators amnion
(+) requirement of endothelin-1 in normal
parturition physiology prostag
However, there is evidence of pathologies PLA2 and PGHS-2
associated with aberrant endothelin-1 Accordingly, many hypothesize that PG
expression regulate events leading to parturition.
o such as premature birth and uterine Amnion - major source for AF
leiomyomas prostaglandins
o (+) role in activation of cascades =
4
promote membrane rupture also can act as chemokines
o n-derived PG recruit to the
on uterine quiescence & activation myometrium
due to PG transport from neutrophils and
the amnion thru the chorion eosinophils
to access maternal tissues further increase
limited by expression of the contractions and
inactivating enzyme, labor
prostaglandin There is uncertainty whether prostaglandin
dehydrogenase. concentration or output from the decidua
increases with term labor onset.
Olson and Ammann (2007) suggest that the
Chorion Laeve major regulation of decidual prostaglandin
action is not their synthesis but rather
primarily protective tissue layer increased PGF2 receptor expression.
(+) immunological acceptance
enriched with enzymes that inactivate
uterotonins: Summary: Regulation of Phase 3 and 4 of
o prostaglandin dehydrogenase
Parturition
(PGDH) - inactivates amnion-
derived PG
It is likely that multiple and possibly
o oxytocinase
redundant processes contribute to the
o enkephalinase
success of the three active labor phases
once phase 1 of parturition is suspended
influence adjacent decidua and myometrium
and phase 2 is initiated.
Phase 3
during labor.
o -
o - stimulated matrix
protein-coupled receptors
metalloproteinase activity
associated with membrane
rupture
stores
o It would further allow prostaglandin
(+)interaction of actin and
entry into the maternal compartment
myosin = force generation.
to promote myometrial
Simultaneously, (+) alteration of cervical
contractility
proteoglycan composition and collagen
progesterone - maintains chorion PGDH
structure
expression
o promotes tissue distensibility and
cortisol - decreases PGDH expression. increased compliance.
o
NET RESULT:
PGDH levels
o initiation of coordinated myometrial
part of progesterone
contractions
withdrawal
with sufficient amplitude and
Decidua
frequency
= cervical dilatation &
A metabolic contribution of decidual pushing of fetus thru birth
activation to parturi- tion initiation is an canal
appealing possibility for both anatomical and
Multiple regulatory ligands orchestrate these
functional reasons.
processes and vary from endocrine
(+) generation of decidual uterotonins hormones such as oxytocin to locally
o paracrine action on contiguous produced prostaglandins.
myometrium is intuitive.
In phase 4 of parturition
(+) steroid metabolizing enzymes expression o Initiation of a complicated series of
o 20 -HSD and eroid 5 R1 repair processes
may regulate local to resolve inflammatory
progesterone withdrawal responses
Decidual activation is characterized by: to remove
o increased proinflammatory cells glycosaminoglycans,
o increased expression of proteoglycans, and
proinflammatory cytokines, PG and structurally compromised
uterotonins collagen.
Uterotonins such as Simultaneously, (+) synthesis of matrix and
oxytocin receptors and cellular components = complete uterine
connexin 43 involution. (+) reformation of dense
Cytokines produced in the decidua connective tissue and structural integrity of
o increase uterotonin production the cervix
(principally prostaglandins)
o act directly on myometrium to cause
contraction.
o Examples: tumor necrosis factor-
(TNF- ) and in erle kin 1, 6, 8, and
12.

5
STAGES OF LABOR
Stages of labor
- Belong to phase 3 of parturition (processes/ stimulation)

Early phase of labor/ Phase 1 Parturition


Characterized by quiescent phase
Uterus should not contract excepts during episodes of Braxton-hicks contractions
We need the uterus to be relaxed because it still has a growing fetus inside that needs all the
space and all the time to develop until it reaches term

Phase 2 Parturition (preparedness/ preparation)


major changes happen both in uterus and cervix
Cervix needed for dilatation end effacement at the time of delivery
Uterus needed for contraction
— Formation of lower uterine segment happens
Isthmus/ isthmic portion forms lower uterine segment
Lower uterine segment passive segment
Upper portion more active segment; where contraction happen and receives all the action
from the uterus
Uterotubal portion Site/ pacemaker of contractions of the uterus; where all the actions
would have to come from
Uterine smooth muscles generate multidirectional courses that would generate
contractions, but these contractions must come from the upper segment/ fundus of the
uterus

What happens to the cervix during phase 2?


— Uterus and cervix is being prepared
Uterus
prepared for more contractions to happen
more gap junctions are being stimulated, more connexin 43 are there and more
contractile proteins are available for the release of oxytocin for more stimulation of
oxytocin receptors and therefore more uterine contractions
Cervix
must always be closed during phase 1 to prevent any form of miscarriage
At the time patient enters labor and delivery, there will be breakdown of collagen
fibers, particularly
Dermatan sulfate holds the fibers of the cervix together in such a way that it
does not give in to any Braxton-hicks contraction during the phase 1 parturition
A competent cervix is very important because we want to keep the pregnancy
intact, only to open by the time the pregnancy is to be delivered/ terminated
because the fetus is already term
Cervix becomes dilated due to more absorption of water due to the presence of
hyaluronic acid
Hyaluronic acid GAG that helps retain water making the cervix softer, flexible
and more pliable to whatever changes that would come from the uterus from the
upper segment
At the time the cervix is ready, the formation of the lower uterine segment makes
he pa ien e perience a cer ain form of dropping do n of he bab kno n a
lightening
Lightening decrease in fundic height may be felt; head may already occupy
the lower uterine segment, ready to be engaged/ enter the pelvic inlet in
preparation for labor

Phase 3 of Parturition
— Time of activation/ stimulation
— Define the different stages of labor

1
st
1 Stage of Labor: Clinical Onset of Labor
- Encompasses the onset of cervical dilatation and effacement and ends with full (10cm) cervical dilatation and
effacement
- In some women, beginning of forceful uterine contractions that effect delivery
- In others, labor initiation is marked by spontaneous release of a small amount of blood-tinged mucus from the
vagina or b d
Show/ bloody show
— extrusion of mucus plug that had previously filled the cervical canal during
pregnancy
— it indicates that labor is already in progress and will likely ensure in hours to
days

o Cervical Changes
two fundamental changes due to contraction forces: Effacement and dilatation
1. Cervical Effacement
— thinning
— obli era ion or aking p of he cer i o ard he lo er erine egmen
shortening of the cervical length
— normal cervical length: 3cm
If there is 50% cervical effacement noted cervical length of 1.5cm from
the lower uterine segment until the edge of the cervix
2
If cervix is 4cm dilated and 80% effaced means there is more
shortened cervical length, cervical length of 0.6cm
Fully dilated cervix or 100% effacement cervix is completely in
complement with that of the lower uterine segment, therefore lip of the
cervix is not felt anymore
— Length of the cervix at the time of labor
— manifest clinically by shortening of the cervical canal from a length of
approximately 2cm to a mere circular orifice with almost paper-thin edges
Internal cervical os --muscular fibers at this the level are pulled upward, or
aken p , in o he lo er erine egmen
External cervical os remains temporarily unchanged
— may be compared to a funneling process in which the whole length of a narrow
cylinder is converted into a very obtuse, flaring funnel with a small cervical
opening
effacement is sometimes accomplished before active labor begins due
to increased myometrial activity
— causes expulsion of the mucous plug as the cervical canal is shortened
2. Cervical Dilatation
— The force exerted by the presenting part makes the cervix open, increasing the
cervical diameter
Internal os diameter measured to measure the cervical dilatation
Patulous cervix external os is open, internal os is still closed; seen in
multiparus patients
During internal exam, insert fingers inside, separate the labia using the
thumb and pinky, insert one finger at a time, rotate 280 degrees and
search/ palpate for the cervix
Evaluate if arching is already in the cervix early onset of labor
Regular pains, regular contractions, ruptured membranes verify using
speculum exam if bag of water is indeed ruptured by observing for pooling
of fluid in the posterior lip of the speculum, check for cervical dilatation,
check for the bloody show
During the entire time of pregnancy, uterus is protected from the vaginal
environment by the mucus plug and once labor starts, the mucus plug is
expelled because of the contractions exposes what is inside the uterine
cavity with that of the vaginal environment
How to look for the cervix? Insert finger downward, posteriorly, reach for
the posterior fornix and then turn upwards and midline and feel for a
depression that your fingers may feel into, feel for the crater cervical os
If patient is still preterm/ not yet into labor, cervix is still pointing
downwards and you will have to go through the posterior fornix to reach
for the cervical os
If patient is in true labor, by the time that fingers are put inside, the cervix
will be meeting you right in the middle
— 10cm dilation for an average-size fetal head to pass through the cervix
— Presenting part descends somewhat as the cervix dilates
nd
During 2 stage of labor: nulliparas presenting part descends slowly
and steadily; multiparas rapid descent
— created by the centrifugal pull exerted on the cervix this is because the lower
segment and cervix have lesser resistance during a contraction
As uterine contractions cause pressure on the membranes, hydrostatic
action of the amniotic sac dilates cervical canal
Absence of intact membranes still with effective pressure of the
presenting part against the cervix and lower uterine part
Early rupture of membranes does no retard cervical dilation as long as
presenting fetal part is positioned to exert pressure against cervix and
lower uterine segment
— Divided into:
a. Latent phase
Its duration is more variable and sensitive to changes by extraneous
factors
Sedation may prolong latent phase; Myometrial stimulation may
shorten it
Has little bearing on course of labor
b. Active phase
Subdivided further into:
Acceleration phase
Phase of maximum slope
Deceleration phase
Usually predicted of labor outcome
During this phase, completion of cervical dilatation is accomplished
by cervical retraction about the presenting part

3
Cervical effacement + dilation formation of the forebag of amniotic fluid (leading portion of fluid and
amniotic sac located in front of presenting part

o Uterine Labor Contractions


— Painful uterine smooth muscle contractions
— Cause is unknown, but several possibilities have been suggested:
1. Hypoxia of the contracted myometrium (similar to angina pectoris)
2. Compression of nerve ganglia in the cervix and lower uterus by contracted interlocking
muscle bundles
3. Cervical stretching during dilatation
4. Stretching of the peritoneum overlying the fundus
Paracervical infiltration with local anesthetic usually produces pain relief with contractions
— Involuntary and independent of extrauterine control
— Its intensity and frequency cannot be diminished by neural blockade from epidural anesthesia
Ferguson reflex mechanical stretching of cervix enhances uterine activity
Manip la ion of he cer i and ripping he fe al membrane associated with increase blood
levels of prostaglandin F2a metabolite (PGFM)

Interval between contractions diminishes gradually (from 10 minutes to as little as 1 minute or


nd
less) in the 2 stage of labor
Periods of relaxation between contractions essential for fetal welfare
Unremitting contractions compromise uteroplacental blood flow causing fetal hypoxemia
Active-phase labor 30-90 seconds (average 1 min) duration of contraction
Normal labor has variable contraction intensity
40mmHg (range: 20-60mmHg) amniotic fluid pressures generated by contractions during
spontaneous labor

o Distinct Lower and Upper Uterine Segments


— anatomical uterine divisions become increasingly evident
— can be differentiated by abdominal palpation even before membranes rupture
Upper segment
— firm during contractions
— contracts, retracts and expels fetus
— its myometrium does not relax to its original length after contractions
— becomes relatively fixed at a shorter length
— contracts down on its diminishing contents, but myometrial tension remains constant
Uterine musculature is kept in firm contact with uterine contents
As a consequence of retraction, each successive contraction commences
where it predecessor left off resulting to a slightly smaller upper part of the
uterine cavity with each successive contraction shortening of the muscular
st
fibers leads to thickening of the upper active segment throughout the 1 and
nd
2 stage of labor
This process continues resulting to a thickened upper uterine segment
immediately after deliver
Upper segment retraction decrease in volume of its contents
Lower segment
— softer, distended and more passive
— dilate together with cervix and form a greatly expanded, thinned-out tube through
which fetus can pass
— Its musculature must stretch, particularly in early labor when cervical dilatations is
minimal, to facilitate the decrease in the volume of contents in the upper segment
— Its relaxation mirrors the same gradual progression of retraction
With labor, successive lengthening of the fibers of the lower segment is
accompanied by thinning

4
Lower segment thinning + upper segment thickening= physiological retraction
ring
Physiologic retraction ring boundary between the upper and lower
segments
Pathological retraction ring (ak.a. Bandl ring) abnormal condition
described as a prominent ring during extreme thinning of lower uterine
segment; seen in obstructed labor
— this mechanism is imperative because if entire myometrium (including lower segment and cervix)
were to contract simultaneously and with equal intensity decreased net expulsive force
Upper Segment Lower Segment

- active - passive

- firm during - softer


contraction
- contracts - dilates
- retracts - distends/ stretches

- expels fetus - forms a tube for


fetus to pass
- shortens - lengthens

- thickens - thinnig

o Changes is Uterine Shape during labor


— elongation of ovoid uterine shape with a decrease in horizontal diameter during each contraction
— importance in labor process:
1. Increase in fetal axis pressure
— Decreased horizontal diameter serves to straighten the fetal vertebral column
resulting in the upper pole of the fetus pressed firmly against the fundus and
lower pole is further thrusted downward
— Lengthening of ovoid shape 5-10cm
2. With the lengthening of the uterus, the longitudinal muscle fibers are drawn taut
— Lower segment and cervix are the only parts of the uterus that are flexible
pulled upward and around the lower pole of the fetus

o Ancillary Forces in Labor


produced by maternal intraabdominal pressure
Pushing contraction of the abdominal muscles simultaneously with forced respiratory efforts
with the glottis closed
Force is similar to that of defecation but with greater intensity
Importance of intraabdominal pressure shown in prolonged descent in labor in paraplegic
women and those with dense epidural block
nd
Increased in intraabdominal pressure is necessary to complete the 2 stage of labor (only
nd
instruct the patient to push during the 2 stage of labor/ if 10cm or fully dilated)
5
st
Pushing in the 1 stage of labor is unnecessary may exhaust the mother and harm the fetus
by increasing intrauterine pressures
nd
2 Stage of Labor: Fetal Descent
- from full cervical dilatation until the expulsion of the fetus
Patient may have full cervical effacement but still not fully dilated
- part of the labor where mother plays the most active role
- time when the patient is asked to push (success of the vaginal delivery largely comes from the effort of the
mother in pushing)
only ask mother to push by the time that she is fully dilated avoid exhausting the mother
exhaustion of mother during pushing may be indicative of cesarean/ abdominal delivery
- Nulliparas engagement of fetal head is accomplished before labor begins
- Head may not further descend until late in labor
- Descent pattern of labor a typical hyperbolic curve is formed when station of fetal head is plotted as a
function of labor duration
Station descent of fetal biparietal diameter in relation to a line drawn between maternal ischial
spines
- Active descent takes place after filatation has progressed
Nulliparas increased rate of descent are observed ordinarily during cervical dilatation phase of
maximum slope
At this time, speed of descent is maximal and is maintained until the presenting part reaches the
perineal floor

o Pelvic Floor Changes During Labor


— Birth Canal supported and is functionally closed by several layers of tissues that together form
the pelvic floor
— Marked changes in the biomechanical properties of levator ani muscle and fibromuscular
connective tissue covering its upper and lower surfaces and of the vaginal wall during
parturition results from altered extracellular matrix structure or composition
Levator ani
— Consists of pubovisceral, puborectalis and iliococcygeus muscles
— Close the lower end of the pelvic cavity as a diaphragm
Concave upper and a convex lower surface
— Varies in thickness from 3-5mm; thicker in margins encircling the rectum and
vagina
— Undergoes hypertrophy during pregnancy forms a thick band that extends
backwards from the pubis and encircles the vagina about 2cm above the plane
of the hymen
— Draws both the rectum and vagina forward and upward in the direction of the
symphysis pubis upon contraction and thereby acts to close the vagina
— Posterior and lateral portions of the pelvic floor occupied bilaterally by the piriformis and
coccygeus muscles

6
st
In the 1 stage of labor intact membranes and fetal presenting part serve to dilate the
upper vagina
Stretching of the levator ani muscle fiber most marked change
Accompanied by thinning of the central portion of the perineum from a wedge- shaped,
5cm thick tissue mass to a thin, almost transparent membranous structure less than 1cm
thick.
When perineum is distended maximally, anus becomes dilated and presents an opening
that varies from 2-3cm in diameter and though which the anterior wall of the rectum bulges

rd
3 Stage of Labor: Delivery of Placenta and Membranes
- Fetal deliver until the expulsion of the placenta
- Begins immediately after fetal delivery and involves separation and expulsion of the placenta and membranes
- As neonate is born, uterus spontaneously contracts
- By the time the newborn is completely delivered:
uterine cavity is nearly obliterated
Uterus consists of an almost solid mass of muscle, several centimeters thick, above the thinner
lower segment
Uterine fundus below the level of the umbilicus
- Sudden decrease in uterine size accompanied by a decrease in the area of the placental implantation site
Placenta increases in thickness, but because of limited placental elasticity, it is forced to buckle
Resulting tension pulls weakest layer (decidua spongiosa) from that placental implantation site
Placental separation due to disproportion created between relatively unchanged placental size
and the reduced size of the implantation site
- Cleavage of placenta aided greatly by the loose structure of the spongy decidua
- Hematoma formed between the separating placenta/ decidua and the decidua that remains
attached to the myometrium; usually a result rather than the cause of placental separation

o Fetal Membrane Separation and Placental Extrusion


— Great decrease in uterine cavity surface area simultaneously throws fetal membranes
(amniochorion and the parietal decidua) into innumerable folds
— Membranes remain in situ until placental separation is nearly completed
— Causes of peeling off of membranes from the uterine walls:
further contraction of myometrium
traction exerted by the separated placenta

7
Increase abdominal pressure facilitated expulsion after the placenta has separated
rd
Completion of 3 stage accomplished by alternately compressing and elevating the fundus
while exerting minimal traction on the umbilical cord
Retroplacental hematoma either follows the placenta or is found within the inverted sac formed
by the membranes

2 Mechanisms of Placental Expulsion:


1. Schultze mechanism
— Blood from the placental site pours into the membrane sac and does not escape
externally until after extrusion of the placenta
— Central mechanism of placental deliver
— Shiny side comes out (Shiny Shultze)
— Not much blood is coming out at this point because the placental will form like
an umbrella which will be filled up by blood that will come out later on (placenta
comes out first)
— Retroplacental type of hematoma will be formed
2. Duncan mechanism
— the placenta separates first at the periphery and blood collects between the
membranes and the uterine wall escapes from the vagina
— placenta descends sideways and its maternal surface appears first
— peripheral type of placental circulation
— reddish part comes out/ maternal side comes out first (DirtyDuncan)
— cotylidons will be seen
— blood will come out first before the placenta that will come out later on
CS(Central Schultze)
MD (Maternal Duncan)
th
4 Stage of Labor
- first hour after the placental delivery
- gives overview of how much bleeding the patient has had during labor and delivery
500 ml expected blood loss for vaginal deliver
1L for CS
>500ml blood loss in vaginal delivery or >1L for CS manifested in the changes in the vital signs
st
of the patient during the 1 hour of delivery
- more frequent monitoring is needed for early detection of excess bleeding and early treatment/ management

Signs that patient has excess bleeding during the first hour after delivery:
- Checheck HR increased
>90bpm indica e ha ome hing i rong i h he pa ien ; peak in ide he pa ien agina and
see if the patient is bleeding too much (is she soaking the bed linen because of too much blood
loss)
- Check for BP Hypotension
- Check urine output decreased

Initial IE tells you:


- dilatation
- effacement
- station of the presenting part
- presentation
- whether bag of water in intact or not

8
o may now last 1min and recur at an
TOPICS: interval no longer than 5.5mins
the median duration of the second stage:
2nd-4th Stage of Labor o 50 minutes in nulliparas
Episiotomy & Episiorraphy abnormal if >2hr w/o
anesthesia; >3hrs w/
Uterine Atony anesthesia
o 20 minutes in multiparas
abnormal if >1hr w/o
Second(24
BLACK Stage
TH EDofWilliams)
Labor ORANGE (LaLaLaLa Notes) anesthesia; >2hrs w/
anesthesia
begins with complete cervical dilatation Monitoring of the fetal heart rate and
(10cm) and ends with fetal delivery interpretation of second-stage electronic fetal
o median duration is approx 50min heart rate patterns should be assessed
(nulliparas) and about 20min
(multiparas)
In a woman of higher parity with a previously Expulsive Efforts
dilated vagina and perineum
o two or three expulsive efforts after full In most cases, bearing down is reflexive and
cervical dilatation may suffice to spontaneous during second-stage labor.
complete delivery. o Occasionally, a woman may not
Conversely, second stage may become employ her expulsive forces to good
abnormally long in women: advantage and coaching is desirable.
o with a contracted pelvis o legs should be half-flexed -> push with
o with a large fetus them against the mattress
o with impaired expulsive efforts from When the next uterine contraction begins
conduction analgesia or sedation o (+) instructed to exert downward
Robinson and colleagues (2011) found that pressure (straining to defecate)
increasing maternal body mass index did not A woman is not encouraged to push beyond
interfere with the second stage of labor the completion of each contraction.
o Instead, she and her fetus should be
allowed to rest and recover.
Duration of Labor During this period of actively bearing down
o the fetal HR appears to be slow (STAT
Our understanding of the normal duration of auscultation after contraction) but
labor may be clouded by the many clinical should recover to normal before the
variables that affect conduct of labor in next expulsive effort
modern obstetrical units. Several positions during the second stage have
the mean length of first- and second-stage been recommended to augment pushing
labor efforts.
o was approx 9 hours in nulliparous o supported upright position than
women w/o regional analgesia recumbent one
(Upper Limit of 18.5hr) Upright positions include
o Corresponding times for multiparous sitting, kneeling, squatting, or
women were a mean of 6 hours (UL: resting with the back at a 30-
13.5 hrs) degree elevation.
labor onset is defined as the time when a Fetal and obstetrical outcomes appear to be
woman recalled: unaffected whether pushing is coached or
o regular, painful contractions q3-5 min uncoached during second-stage labor
-> cervical change o Women coached to push during
Parity nulliparous versus multiparous and second-stage labor had decreased
cervical dilatation at admission were bladder capacity and decreased first
significant determinants of the length of urge to void compared with women
spontaneous labor. encouraged to push or rest as desired.
o The median time from admission to (+) head descends through the pelvis ->
spontaneous delivery for all perineum begins to bulge and the overlying
parturients skin becomes stretched.
Equates to 3.5 hours (UL: o (+) fetal scalp visible through the
10.1 hours) vulvar opening
These results suggest that normal human o At this time, the woman and her fetus
labor is relatively short. are prepared for delivery

Risk Factors for prolonged 2nd Stage:


o Maternal weight / weight gain
o Nulliparity Management of 2nd Stage of Labor
o Use of conduction/regional
anesthesia 1. Monitoring of Fetus and Mother
o Fetal OP (occiput posterior) position a. Low Risk Pregnancies
o Increased BW i. FHR q15min after each
contraction
ii. Fetal heart tones best heard
Management of the Second Stage of Labor after contraction or 1-2mins
before peak of contractions
With full cervical dilatation (onset of the b. High Risk Pregnancies
second stage) -> begins to bear down i. FHR q5min
c. Slowing of FHR during this stage
With descent of the presenting part -> (+)
maybe due to compression of head
develops the urge to defecate.
during descent (but not all)
Uterine contractions and the accompanying
i. Other reasons: cord
expulsive forces
compression or tightening of
1
nuchal cord, premature o May require <10min but may last as
placental separation long as 30mins
d. Monitor FHR after contractions If beyond this, consider active
i. Determines the response of intervention
FHR to temporary loss of Interference in the absence of
oxygenation hemorrhage may cause:
2. When do we ask mothers to push? hemorrhage, uterine inversion,
a. 2nd stage of labor and placental entrapment
b. during contractions with full cervical Signs of Placental Separation
dilatation o Calkin s Sign (earliest)
3. Preparation for Delivery Change in uterine shape from
a. Assume a dorsal lithotomy position discoid -> globular (contraction)
leg flexed o Blood how from the vagina
i. Asked to breathe normally Cord lengthening
until start of contraction when o Uterus rises in abdomen as placenta
they are asked to strain down descends to lower uterine segment or
ii. Push sustained as long as vagina
possible (+) upward uterine displacement
iii. Ideal position because it inc.
pelvic outlet diameter rd
b. Strict asepsis and antisepsis (sterile Active Management of 3 Stage of Labor
gown, cap, mask) o Before, they used to just wait for the
i. Perineal area scrubbed w/ placenta to go out. Now, there is
antiseptic solution active management
ii. Apply antiseptic first before Get anesthesia for uterine
sterile drapes are applied relaxation
4. Delivery of the Head and Episiotomy (+) gloving & sterile
a. Perineal opening becomes ovoid to gowning -> insert fingers
circular with the descent of the fetal inside (remove remnants
similar to tearing of pages
presenting part
in a book)
b. Perineum is stretched to almost paper
o Do gentle and downward cord traction
thin o One hand is placed on the abdomen
c. Crowning event wherein fetal head is and uterus is pushed cephalad until

encircled by the vulvar ring placenta reached intoitus
i. sign that episiotomy can be When majority of placenta is
done out, gently grasp with both
hands and pull while twisting
the placenta allowing
Active Management of Labor separation of the fetal
membranes
o Massage fundus until contraction is felt
(+) concept that a disciplined, standardized labor Palpate abdomen to confirm

management protocol reduced the number of reduction in size of uterus and
cesarean 
deliveries for dystocia. 
 firmness
Two of its components amniotomy and Ongoing blood loss and soft
oxytocin 
have been widely used 
 boggy uterus suggests uterine
With this protocol, labor is diagnosed when atony
painful 
contractions are accompanied by: 
 If uterus is firm but with
ongoing blood loss
o complete cervical effacement
(+)Consider cervical or
o blood ho 
 vaginal lacerations
o ruptured membranes o Start oxytocin drip or give ergot
Women with such findings are committed to derivatives (contraindicated to Px with
delivery within 12 hours. HPN)
o Pelvic examination -> performed q1hr
(for the next 3hr) -> 2-hour intervals
thereafter FOURTH STAGE OF LABOR
o amniotomy is performed - dilatation has
not increased by at least 1 cm/hr
1hr immediately ffing placental delivery
o Progress is again assessed at 2 hours
critical period and where lacerations are repaired
high-dose oxytocin infusion is
started unless dilatation of at (+) admini ra ion of ero onic ake place
least 1 cm/hr is documented o for tx of postpartum hemorrhage as the
result of uterine atony
Women are constantly attended by midwives.
Uterine tone and perineum should be evaluated
If membranes rupture before admission,
frequently
oxytocin is begun for no progress at the 1-hour
mark. Maternal BP and pulse should be recorded STAT
after delivery & q15min for the first 2hrs.
This type of management somewhat shortened
o Recommended by the American
labor but did not affect the cesarean delivery
Academy of Pediatrics and the
rate
American College of Obstetricians and
Gynecologists (2012)
Examination for completeness of the ffing
THIRD STAGE OF LABOR Stage of Placental
structures:
Separation and Expulsion o Placenta, membranes, and umbilical
cord
Begins after delivery of fetus until delivery of
the placenta
o Usually begins 5-10mins after fetal
delivery

2
Birth Canal Lacerations

Lower genital tract lacerations


o involves the cervix, vagina, or perineum
Perineal tears - follows any vaginal delivery &
classified as BCL
3rd and 4th-degree lacerations - higher-order
lacerations
o Short-term assoc. with éblood loss,
puerperal pain, and wound disruption or
infection risk
o Long-term (0.25-6%) linked with inc.
rates of anal incontinence and
dyspareunia.
Risk factors for these more complex lacerations
o midline episiotomy, nulliparity, longer
second- stage labor, precipitous
delivery, persistent OP position,
operative vaginal delivery, Asian race,
and inc. fetal BW
Epidural analgesia was found to be protective
Mediolateral episiotomy - most powerful predictor of
wound disruption.

Episiotomy

Greek episton pubic region plus tomy to cut.


o incision of the pudendum the external
genital organs.
Perineotomy incision of the perineum
the term episiotomy often is used synonymously
with perineotomy
o common to practice
midline incision è creating a median or midline
episiotomy
begin at midline and directed laterally and
downward away from the rectum -> mediolateral
episiotomy Episiotomy Type and Timing

Before episiotomy, analgesia may be provided thru:


Episiotomy Indications and Consequences o Labor epidural analgesia
o bilateral pudendal nerve blockade
episiotomy - common obstetrical procedure o infiltration of 1% lidocaine
o use has inc. over the past 30 years Bleeding from the episiotomy
(age-adjusted rate) o During the interval b/w incision and
Primipara common practice to cut an episiotomy delivery due to unnecessarily early
(1970s) administration
o Easier to repair due to a straight surgical (+) Lacerations - If performed too late
incision Typically, episiotomy is completed when the
Less operative pain, improved healing, prevention head is visible during a contraction to a diameter
of pelvic floor disorders with episiotomy of approximately 4 cm, that is, crowning.
o Still has no evidence When used in conjunction with forceps delivery,
Routine Episiotomy - assoc with dec. incidence of most perform an episiotomy after application of
anal sphincter & rectal tears but with inc. the blades.
incidence of anterior perineal trauma
o (+) anal sphincter incontinence inc.
risk of higher-order lacerations Technique
fecal and flatal incontinence
increased rates of posterior perineal trauma, For midline episiotomy
surgical repair, and healing complications o fingers - insinuated between the
o women managed with a restrictive use crowning head and the perineum
of episiotomy o scissors - positioned a 6 o clock on he
o routine used by the American College of vaginal opening and directed posteriorly
Obstetricians and Gynecologists o incision length - varies from 2 to 3 cm
(2013a) depending on perineal length
episiotomy should be considered for indications and degree of tissue thinning
such as: incision is customized for
o shoulder dystocia, breech delivery, specific delivery needs
macrosomic fetuses, operative vaginal should stop before reaching
deliveries, persistent OP positions external anal sphincter.
o other instances in which failure to
perform an episiotomy will result in
significant perineal rupture
FINAL RULE: no substitute for surgical judgment
and common sense

3
For mediolateral episiotomy
scissors - po i ioned a 7 o clock or a 5
o clock
incision - extended 3 to 4 cm toward the
ipsilateral ischial tuberosity
inc. in severe perineal lacerations
(+) protective effect against higher-order
lacerations when used during operative vaginal
delivery
midline episiotomy is superior - third and fourth-degree
extensions

perineal suturing
oincrease dose for those with epidural
analgesia
Hemo a i and ana omical re ora ion /o
suture
o Essential for episiorrhaphy (repair)

Repair of Episiotomy or Perineal Laceration


continuous or interrupted closure (+)similar
episiotomy repair - deferred until the placenta has postoperative pain scores
been delivered. o less suture material and faster
o permits undivided attention to the signs procedure = less pain
of placental separation and delivery Blunt needles suitable & dec. incidence of
o Advantage needlestick injuries
repair is not interrupted or o The suture material commonly used is
disrupted by the obvious 2 0 chromic catgut
necessity of delivering the o Sutures made of polyglycolic acid
placenta derivatives are also used
specially if manual o Synthetic materials advantage is dec.
removal must be postsurgical pain
performed that may though occasionally require
disrupt a newly suture removal from the repair
repaired episiotomy. site because of pain or
o major disadvantage: dyspareunia
continuing blood loss until the maybe reduced using a
repair is completed rapidly absorbed
(+) Direct pressure from an polyglactin 910 (Vicryl
applied gauze sponge Rapide).
will help to limit blood loss.
Adequate analgesia is imperative
o Local lidocaine Repair of a mediolateral episiotomy is similar to a midline
Used solely or as a supplement repair.
to bilateral pudendal nerve
blockade.
o women w/o regional analgesia
(+)high levels of pain during

4
Unfortunately, normal function is not always ensured even
with correct and complete surgical repair. Some women
may experience continuing fecal incontinence caused by
injury to the innervation of the pelvic floor musculature

Postepisiotomy Pain

Pudendal nerve blockade - aid relief of perineal


pain postoperatively
Locally applied ice packs - help reduce swelling
and allay discomfort
Topical application of 5% lidocaine ointment
o was not effective in relieving episiotomy
or perineal laceration discomfort
Analgesics such as codeine give considerable
Fourth-Degree Laceration Repair relief
o pain may be a signal of a large vulvar,
Two methods are used to repair a laceration paravaginal, or ischiorectal fossa
involving the anal sphincter and rectal mucosa. hematoma or perineal cellulitis
o first is the end-to-end technique - these sites should be examined
which we prefer carefully if pain is severe or
o second is the overlapping technique. persistent
o In addition to pain, urinary retention may
complicate episiotomy recovery

For those with second-degree or greater


lacerations
o intercourse is usually proscribed until
essential to approximate the torn edges of the after the first puerperal visit
rectal mucosa with sutures placed in the rectal usually at 4 to 6 weeks.
muscularis approximately 0.5 cm apart
o choice is to use 2 0 or 3 0 chromic gut
o next is to cover this muscular layer by CAUSES OF OBSTETRICAL HEMORRHAGE
o reapproximation of the internal anal
sphincter
The most important causes because of their
o Finally, there are isolation,
incidence or consequence of severe obstetrical
approximation and suturing of the cut
hemorrhage and their contribution to maternal
ends of the external anal sphincter
mortality rates
together end-to-end with three or four
Fatal hemorrhage is most likely in circumstances in
interrupted stitches
which blood or components are not immediately
o remainder of the repair is the same as
available.
for a midline episiotomy.
the ability to provide prompt resuscitation of
The overlapping technique - alternative method to
hypovolemia and blood administration is an
approximate the external anal sphincter
absolute requirement for acceptable obstetrical
(+) recommends perioperative antimicrobial
care.
prophylaxis
o for the reduction of infectious morbidity
assoc with higher-order perineal injury
Uterine Atony
repair
o single dose of a cephalosporin (2nd- most frequent cause of obstetrical hemorrhage
generation) is suitable o failure of the uterus to contract
clindamycin for penicillin- sufficiently after delivery and to arrest
allergic women bleeding from vessels at the placental
Postoperatively, stool softeners should be given implantation site
o prescribed for a week some bleeding is inevitable during third stage labor
o enemas and suppositories should be as the placenta begins to separate
avoided. o the Duncan mechanism of placental
separation immediate escape of the
blood from the implantation site to the

5
vagina
o the Schultze mechanism remained
concealed behind the placenta and
membranes until the placenta is
delivered
With bleeding during the third stage
o uterus should be massaged if it is not
contracted firmly
o After the signs of separation, placental
descent is indicated by the cord
becoming slack.
Placental expression should be attempted by
manual fundal pressure
If bleeding continues, then manual removal of the
placenta may be necessary

Separation and delivery of the placenta by cord traction,


especially when the uterus is atonic, may cause uterine
inversion

Third-Stage Bleeding

Manual Placental Removal


o Indicated if significant bleeding persists
after delivery of the infant and while the
placenta remains partially or totally
attached
adequate analgesia is
mandatory
aseptic surgical technique
should be used.
o the placenta is peeled off its uterine
attachment Risk Factors
similar motion to that used in
separating the pages of a book In many women, uterine atony can at least be
o After its removal, trailing membranes are anticipated well in advance of delivery
removed the ability to identify which individual woman will
by carefully teasing them from experience atony is limited
the decidua Risk factors for retained placenta are similar
(+) use of ring forceps The magnitude of risk imposed by each of these
Another method is to wipe out the uterine cavity factors shown in Table 41-2 varies considerably
with a gauze-wrapped hand between reports

Uterine Atony after Placental Delivery

fundus should always be palpated following o Primiparity has been cited as a risk factor
placental delivery o High parity is a long-known risk factor for uterine
o to confirm that the uterus is well atony.
contracted o The overdistended uterus - prone to hypotonia after
o If it is not firm, then do vigorous fundal delivery
massage o Inc. risk for women with a large fetus,
prevents postpartum multiple fetuses, or hydramnios
hemorrhage from atony o Labor abnormalities predispose to atony
o Simultaneously, 20 units of oxytocin in o Includes hyper- or hypotonic labor
1000mL of crystalloid solution o Labor induction or augmentation
IV admin of 10 mL/min (dose of o with either prostaglandins or oxytocin
200mU/min) o more likely to be followed by atony
Proven to be effective o woman who has had a prior postpartum
Higher concentrations are hemorrhage
minimally more effective o risk for recurrence with a subsequent
Never give Oxytocin (undiluted bolus) delivery
Can develop serious
hypotension or cardiac
arrhythmias Evaluation and Management

With immediate postpartum hemorrhage


o careful inspection is done to exclude
birth canal laceration
o bleeding can be caused by retained
placental fragments
do routine inspection of the
placenta after delivery
If a defect is seen, uterus
should be manually explored ->

6
fragment removal o Dinoprostone (prostaglandin E2)
o Occasionally, retention of a 20-mg suppository per rectum
succenturiate lobe may cause post- or per vaginam q2h
partum hemorrhage It typically causes diarrhea
During examination for lacerations and causes of which is problematic for the
atony rectal route whereas vigorous
o uterus is massaged and uterotonic vaginal bleeding may preclude
agents are administered its use per vaginam
o Sulprostone (PGE2 IV) - used in
Europe, but not available in US
Uterotonic Agents
o Misoprostol (Cytotec) - synthetic
prostaglandin E1 analogue
several compounds that prompt the postpartum
evaluated for both prevention
uterus to contract
and treatment of atony and
One of these is routinely selected and given to
postpartum hemorrhage
prevent postpartum bleeding
From a systematic review, oxytocin and ergot
o by ensuring uterine contractions.
preparations administered after delivery were
also used to treat uterine atony with bleeding
more effective than misoprostol for prevention of
(+) given prophylactically will prevent most cases of postpartum hemorrhage
uterine atony
When atony persists despite preventive measures
o Ergot derivatives have been used for
Bleeding Unresponsive to Uterotonic Agents
second- line treatment
includes methylergonovine
(Methergine) and ergonovine Hemorrhage that persists despite uterine massage
Only methylergonovine is and continued uterotonin administration may be
currently manufactured in USA from a yet unrecognized genital tract laceration
Given IM, (+) rapidly stimulate (uterine rupture)
tetanic uterine contractions (for Thus, if bleeding persists after initial measures for
45mins) atony have been implemented, then the following
management steps are performed immediately
0.2mg of either drug is
and simultaneously:
a common regimen
1. Begin bimanual uterine compression
Disadvantage:
which is easily done and controls
if given IV = dangerous
most cases of continuing
HPN with women w/
hemorrhage
preeclampsia
This technique is not simply fundal
severe HPN massage.
concomitant use of o The posterior uterine wall is
protease inhibitors massaged by one hand on the
(HIV infection) abdomen
o E- and F-series prostaglandins o while the other hand is made into
2nd line agents for atony in the a fist and placed into the vagina
more recent years This kneads the anterior
Carboprost tromethamine uterine wall through the
(Hemabate) anterior vaginal wall
15-methyl derivative of Concurrently, the uterus is
prostaglandin F2á also compressed between
dose of 250 ìg (0.25 the two hands
mg) IM Tx for 2. Immediately mobilize the emergent-care
uterine atony (>25yrs) obstetrical team to the delivery room
and call for whole blood or packed red
can be repeated if
cells
necessary o 15- to 90-
3. Request urgent help from the
minute intervals (max
anesthesia team.
of 8doses)
4. Secure at least two large-bore IV
Another pharmacological effect
catheters
pulmonary airway and
crystalloid with oxytocin is
vascular constriction.
continued simultaneously with
should not be used for blood products
asthmatics and those Insert an indwelling Foley
with suspected catheter
amnionic-fluid
o for continuous urine
embolism output monitoring.
Side Effects (in descending 5. Begin volume resuscitation with rapid
order of frequency): intravenous infusion of crystalloid
Diarrhea 6. With sedation, analgesia, or anesthesia
Hypertension established and now with optimal
Vomiting exposure
Fever once again manually explore the
Flushing uterine cavity
Tachycardia o for retained placental
severe hypertension given to fragments
women with preeclampsia o for uterine
can cause arterial oxygen abnormalities, including
desaturation lacerations or rupture.
7. Thoroughly inspect the cervix and
E-series prostaglandins have been used to prevent vagina again for lacerations that may
or treat atony have escaped attention.
8. If the woman is still unstable or if there is
7
persistent hemorrhage hemorrhage from atony and these includes:
(+)blood transfusions are given o uterine compression sutures
o pelvic vessel ligation
o angiographic embolization
At this juncture, after causes other than atony have o hysterectomy
been excluded and after hypovolemia is
reversed, several other measures are considered
if bleeding continues. Their use depends on
several factors such as parity, desire for
sterilization, and experience with each method.

Uterine Packing or Balloon Tamponade

Uterine packing was popular to treat refractory


uterine atony (1st half of the 20th century)
o (+) concealed bleeding and infection
subsequently fell from favor
Newer techniques have been described that
alleviate some of these concerns
o In one technique, the tip of a 24F Foley
catheter with a 30-mL balloon
guided into the uterine cavity
and filled with 60 to 80 mL of
saline
(+) open tip permits continuous
drainage of blood from the
uterus
If bleeding subsides,
the catheter is
typically removed after
12 to 24 hours
Similar devices that have been used for tamponade
include Segstaken-Blakemore and Rusch
balloons and condom catheters
Alternatively, the uterus or pelvis may be packed
directly with gauze
Enthusiasm has developed for specially
constructed intrauterine balloons to treat
hemorrhage from uterine atony and other causes
A Bakri Postpartum Balloon (Cook Medical) or BT-
Cath (Utah Medical Products)
o may be inserted and inflated to
tamponade the endometrial cavity and
stop bleeding
o Insertion requires two or three team
members
The first performs abdominal
sonography during the
procedure.
The second places the deflated
balloon into the uterus and
stabilizes it.
The third member instills fluid
to inflate the balloon
rapidly infusing at
least 150mL -> further
instillation over a few
minutes (for a total of
300 to 500 mL) to
arrest hemorrhage.

balloon tamponade + uterine compression = Good


Prognosis
Failures for all of these require various surgical
methods (hysterectomy)

Well-designed studies are needed before instituting


intrauterine balloon tamponade as a second-line treatment
for atony.

Surgical Procedures

Several invasive procedures can be used to control


8
Mentioned by lecturer: this magandang shade of 3 parameters to assess in uterine
BLUE contractions:
1. Interval
Summary of LABOR: 2. Duration
Definition: Uterine contractions that bring about 3. Intensity
demonstrable effacement and dilatation of the Has 2 phases: latent phase & active phase
cervix 1. Latent phase: period between the
Onset: onset of labor and when rate of
- commences with the time of admission cervical dilatation changes most
- Painful contractions become regular rapidly (0-4cm dilatation)
intact membranes, then a cervical dilatation of 3 Contractions are still not too
to 4 cm or greater is presumed to be a close to each other (8-10
reasonably reliable threshold for the diagnosis of mins interval)
labor Normal duration:
Cervical dilatation in the absence of uterine o 8 ½ hrs in
contractions = incompetent cervix nulliparas
Uterine contractions without cervical changes = o 5 hrs in multiparas
NOT true labor Abnormal:
o > 20 hrs in
Admission criteria: nulliparas
Uterine contractions plus any of the ff: o > 12 hrs in
1. Ruptured membranes multiparas
2. Bloody show 2. Active phase: period of increased
3. Complete cervical effacement and dilatation rapidity of cervical dilatation
Contractions with 2-3mins interval (ideal), lasting
The ideal management of labor and delivery 40-60 secs
requires two potentially opposing viewpoints on the Ends with full cervical dilatation at 10cm (also marks
part of clinicians. the end of the first stage of labor)
1. Birthing should be recognized as a normal Also coincides with descent of presenting
physiological process that most women experience part into pelvis
without complications Normal duration and rate of
2. Intrapartum complications should be anticipated. dilatation:
- clinicians must simultaneously make every o 5-7 hrs in nulliparas (1- 2cm/hr)
woman and her supporters feel comfortable and safe o 2-4 hrs in multiparas
- Call o Ac ion - provide detailed information (1.5cm/hr)
on the appropriate content of intrapartum care, o Pelvic exams done every
including both personnel and facility requirements o 2-3 hrs to evaluate labor progress
(lack of progress in
Admission Procedures dilatation/descent may indicate
dystocia)
Early admittance to the labor and delivery unit is o *Crowning encirclement of the
important, especially if during antepartum care the largest head diameter by the vulvar
woman, her fetus, or both have been identified as ring
being at risk.
Emergency Medical Treatment and Labor Act
Identification of Labor EMTALA
Differentiation between false and true labor - Ensure public access to emergency services
can be clarified by contraction frequency regardless of the ability to pay
and intensity and by cervical dilatation. Must provide an appropriate screening
o instances when a diagnosis of labor examination for any pregnant woman
cannot be established with certainty, experiencing contractions and presenting to
observation for a longer period is the emergency department for evaluation
often wise Labor i defined a he proce of childbir h
In the absence of ruptured membranes or beginning with the latent phase of labor
bleeding, uterine contractions 5 minutes continuing through delivery of the placenta.
apart for 1 hour ha i , 12 con rac ion in A woman experiencing contractions is in true
1 hour may signify labor onset labor unless a physician certifies that after a
Active labor defined a cer ical dila a ion reasonable time of observation the woman is
4 cm i h 12 con rac ion per ho r. in false labor.
Women admitted during latent-phase labor A woman in true labor is considered
had more active-phase arrest, more frequent n able for in erho pi al ran fer p rpo e
need for oxytocin labor stimulation, and until the newborn and placenta are delivered.
higher rates of chorioamnionitis Electronic Fetal Heart Rate Monitoring
- routinely used for high-risk pregnancies
Stage of cervical effacement and dilatation commencing at admission
Involves widely spaced uterine contractions - some recommend monitoring women with
(intervals diminishes from 10 mins at onset low-risk pregnancies upon admission as a test of
of stage 1 to 1 min or less in stage 2) fetal well being the so-called
Period of relaxation in between is important Fetal admission test/ Labor Admission Test.
for fetal welfare 20 min EFM strip done on
o Unremitting contractions may result admission
to fetal hypoxemia
1
Get heartbeat, acceleration and group B streptococcal infections
deceleration in relation is recommended
to uterine contractions o Temperature: important in
Purpose: to find out whether chorioamnionitis
the fetus in utero can o BP is taken more frequently
during active phase (usually
undergo the stress of labor timed after contraction) a
If you have decelerations rise in BP is usually seen
while baby is in utero, you during contraction
expect that it will be worse Abdominal exam
during labor o Fundic height: important for
Allows you to detect high estimating fetal weight and
risk/compromised fetuses lightening (if FH is 35-36: this
and allows for early may be a big baby)
intervention o Leopold maneuver: LM3 allows
determination of presentation
Studies show greater
o Uterine contractions (normal must
benefit in using LAT for high be alternating contract and relax)
risk patients only o Uterine tenderness
If no fetal heart rate abnormalities are o Tenderness over scar may
detected, continuous electronic monitoring is indicate dehiscence or possible
replaced by intermittent assessment for the uterine rupture (abruption
remainder of labor. placenta)
o Check for fetal heart tones
Home Births
most studies suggest increased risks with Pelvic exam
home deliveries o Cervical dilatation, effacement,
station, presentation, position,
Initial Evaluation (lahat naman ito sinabi plus the consistency of cervix
opinion of the all knowing ever researching Cervical Assessment
Williams MALIBAN NA LANG KAPAG usually is expressed in terms of the length of the
NAKASULAT na hindi) cervical canal compared with that of an
1. Get px history (Gen data) uneffaced cervix.
Chief complaint When the cervix becomes as thin as the
o Vaginal bleeding adjacent lower uterine segment, it is completely,
o Watery vaginal discharge or 100-percent, effaced.
o Hypogastric pain Cervical dilatation is determined by estimating
Ask how many hours, the average diameter of the cervical opening by
intervals, if accompanied sweeping the examining finger from the margin
by watery/bloody of the cervical opening on one side to that on the
discharge opposite side.
Note the no. of hours she The diameter traversed is estimated in
has been experiencing them centimeters.
because the longer the The position of the cervix is determined by the
bag of water has ruptured, relationship of the cervical os to the fetal head
the higher the risk for and is categorized as posterior, mid-position, or
infection = anterior
chorioamnionitis Consistency of cervix is determined to be soft,
increased maternal and firm, or intermediate between these two.
infant mortality and The level or station of the presenting fetal
morbidity part in the birth canal is described in relationship
Menstrual hx (important for to the ischial spines, which are halfway between
determining AOG) the pelvic inlet and the pelvic outlet.
Obstetrical hx When the lowermost portion of the presenting
o Ask for the ff: manner of
delivery in past pregnancies, fetal part is at the level of the spines, it is
complications, duration of designated as being at zero (0) station.
labor, hx of difficult labor, Classification of station that divides the pelvis
weight of babies (may indicate above and below the spines into fifths. Each fifth
a problem in descent or an represents 1 cm above or below the spines.
arrest in cervical dilatation) Thus, as the presenting fetal part descends from
2. Physical Exam
the inlet toward the ischial spines
Get vital signs
o Temperature, pulse, and blood Station +5 cm corresponds to the fetal head
pressure are evaluated at least being visible at the introitus
If the leading part of the fetal head is at 0 station
every 4 hours.
or below, most often the fetal head has
o If membranes have been
ruptured for many hours before engaged thus, the biparietal plane has passed
labor onset or if there is a through the pelvic inlet. If the head is unusually
borderline temperature molded or if there is an extensive caput
formation or both, engagement might not have
elevation, the temperature is
checked hourly. taken place although the head appears to be at
0 station.
o With prolonged membrane
rupture, defined as greater than
18 hours, antimicrobial
administration for prevention of
2
Clinical pelvimetry for primigravidas Intrapartum Fetal Monitoring
- Check for intact bag of water and ruptured Recommend that during first-stage labor, in
Membranes the absence of any abnormalities
Significant for three reasons. Fetal heart rate should be checked
1. First, if the presenting part is not fixed in the immediately after a contraction at least
pelvis, the possibility of umbilical cord prolapse every 30 minutes and then every 15 minutes
and compression is greatly increased during the second stage.
2. Second, labor is likely to begin soon if the If continuous electronic monitoring is used,
pregnancy is at or near term. the tracing is evaluated at least every 30
3. Third, if delivery is delayed after membrane minutes during the first stage and at least
rupture, intrauterine infection is more likely as the every 15 minutes during second-stage labor.
time interval increases For women with pregnancies at risk, fetal
Upon sterile speculum examination, ruptured heart auscultation is performed at least
membranes are diagnosed if amnionic fluid every 15 minutes during first-stage labor and
pools in the posterior fornix or clear fluid flows every 5 minutes during the second stage
from the cervical canal. *none is completely
reliable Uterine Contractions
pH determination of vaginal fluid. The pH of With the palm of the hand resting lightly on
vaginal secretions normally ranges from 4.5 to the uterus, the time of contraction onset is
5.5, whereas that of amnionic fluid is usually determined
7.0 to 7.5. Intensity is gauged from the degree of
Nitrazine - Test papers are impregnated with firmness the uterus achieves.
the dye, and the color of the reaction between At the acme of effective contractions, the
these paper strips and vaginal fluids is finger or thumb cannot readily indent the
interpreted by comparison with a standard color er d ring a firm con rac ion. The ime
chart. A pH above 6.5 is consistent with at which the contraction disappears is noted
ruptured membranes. False-positive test results next. This sequence is repeated to evaluate
may occur with coexistent blood, semen, or the frequency, duration, and intensity of
bacterial vaginosis, whereas false-negative uterine contractions
tests may result with scant fluid.
Arborization or ferning of vaginal fluid, which * See Labor Admission Test above
suggests amnionic rather than cervical fluid.
Amnionic fluid crystallizes to form a fernlike Subsequent Cervical Examinations
pattern due to its relative concentrations of During the first stage of labor, the need for
sodium chloride, proteins, and carbohydrates subsequent vaginal examinations to monitor
(Fig. 4-2, p. 49). cervical change and presenting part position
Detection of alpha-fetoprotein in the vaginal will vary considerably
vault When the membranes rupture, an
Rarely required, identification may also follow examination to exclude cord prolapse should
injection of indigo carmine into the amnionic be performed expeditiously if the fetal head
sac via abdominal amniocentesis. was not definitely engaged at the previous
examination
*There are 2 indications for a The fetal heart rate should also be checked
speculum exam: Watery vaginal discharge and immediately and during the next uterine
bleeding contraction to help detect occult umbilical
cord compression
Laboratory Studies (hindi nabanggit) Periodic pelvic examinations are typically
When admitted in labor, most often the performed at 2- to 3-hour intervals to
hematocrit or hemoglobin concentration evaluate labor progress.
should be rechecked
o Hematocrit Oral Intake
o Blood type and antibody screen, Food should be withheld during active labor
if needed and delivery (NPO 8 hours)
o Syphilis and human Gastric emptying time is remarkably
immunodeficiency virus (HIV) prolonged once labor is established and
serology. analgesics are administered (note:
o Urine specimen for protein Progesterone effect of delayed gastric
determination in hypertensive emptying)
women only As a consequence, ingested food and most
Women who have had no prenatal care medications remain in the stomach and are
should be considered to be at risk for not absorbed. Instead, they may be vomited
syphilis, hepatitis B, and HIV and aspirated
Sips of clear liquids, occasional ice chips,
Management of the First Stage of Labor and lip moisturizers are permitted
- Rational plan for monitoring labor can then be In the new protocol, it has been studied that
established based on the needs of the fetus and the intake and presence of IV line does not
mother contribute to the increased effectiveness of
- marked individual variations in labor lengths, labor. BUT, these are still done due to the
precise statements as to its anticipated duration are physician beneficial belief
unwise

3
Intravenous Fluids Amniotomy
Although it has become customary in many If the membranes are intact, there is a great
hospitals to establish an intravenous temptation, even during normal labor, to
infusion system routinely early in labor, there perform amniotomy.
is seldom any real need for this in the The presumed benefits are:
normal pregnant woman, at least until o more rapid labor,
analgesia is administered. o earlier detection of meconium-
An intravenous infusion system is stained amnionic fluid, and the
advantageous during the immediate o opportunity to apply an electrode to
puerperium to administer oxytocin the fetus or insert a pressure
prophylactically and at times therapeutically catheter into the uterine cavity for
when uterine atony persists. monitoring.
With longer labors, the administration of Importantly, the fetal head must be well
glucose, sodium, and water to the otherwise applied to the cervix and not be dislodged
fasting woman at the rate of 60 to 120 mL/hr from the pelvis during the procedure to avert
prevents dehydration and acidosis umbilical cord prolapse.
IV fluids are given when px has nothing per The advantages and disadvantages of
orem for 6-8 hrs to prevent dehydration amniotomy were NOT DISCUSSED so you
(provide glucose and water) may skip this part if you want:
Ph ician di cre ion a o ha ing an IV A common indication for artificial rupture of
line open for emergencies which
the membranes
require immediate
surgical
infusion
amniotomy
of medication
However caution must also be exercised includes the need for direct monitoring of the
because full NPO may cause fetal heart rate or uterine contractions or
hypoglycaemia, shock, seizures both.
During amniotomy, to minimize cord
Maternal Position prolapse risk, dislodgement of the fetal head
The normal laboring woman need not be is avoided
confined to bed early in labor Fundal or suprapubic pressure or both may
In bed, the laboring woman should be be helpful.
allowed to assume the position she finds Some clinicians prefer to rupture
most comfortable this will be lateral membranes during a contraction
recumbency most of the time If the vertex is not well applied to the lower
She must not be restricted to lying supine uterine segment, a gradual egress of
because of resultant aortocaval amnionic fluid can sometimes be
compression and its potential to lower accomplished by several membrane
uterine perfusion punctures with a 26-gauge needle held with
o aortocaval system reduce blood a ring forceps and with direct visualization
flow to fetus fetal compromise using a vaginal speculum.
o May also reduce intensity of In many of these, however, membranes tear
contractions interferes with and fluid is lost rapidly. Because of the risk
progress of labor of cord prolapse or rarely abruption, the fetal
Upright position: speed contractions in early heart rate is assessed before and
labor immediately after amniotomy.
Exceptions to allowing pregnant woman to Elective Amniotomy
take desired/comfortable positions: o Membrane rupture with the intention
a) Membranes have ruptured in the of accelerating labor is often
presence of a NON- engaged fetal head performed.
b) Sedated patients o In the investigations presented,
- In these cases, ask patient to assume left amniotomy at approximately 5-cm
lateral decubitus as it increases blood flow dilation accelerated spontaneous
to fetus labor by 1 to 1½ hours.
o Neither the need for oxytocin
Analgesia stimulation nor the overall cesarean
In general, pain relief should depend on the delivery rate was increased.
needs and desires of the woman. o Although mild and moderate cord
Give only during active phase compression patterns were
o Non-medicated measures increased following amniotomy,
Natural childbirth cesarean delivery rates for fetal
Support person/doula distress were not increased.
provide coaching during delivery o Most importantly, there were no
Soaking tubs (temp kept at 98- adverse perinatal effects.
100oF to prevent fever in mother Amniotomy Induction
and baby) o Artificial rupture of the
Analgesics you can give: membranes sometimes called
o Benzodiazepine surgical induction can be used to
o Nalbuphine Hcl induce labor, and it always implies a
o Meperedine Hcl (give in smallest commitment to delivery.
possible dose to prevent adverse o The main disadvantage of
effects in fetus) amniotomy used alone for labor
Anesthesia: pudendal, epidural, spinal, induction is the unpredictable and
general (*epidural is still the best) occasionally long interval until labor
onset
4
o Found that amniotomy alone or Fetal Monitoring
combined with oxytocin was o Main parameters: FHR and FHR
superior to oxytocin alone variability
o Oxytocin induction to either early o Usually based on recordings while
o px is hooked to
amniotomy at 1 to 2 cm or late
electronic fetal
amniotomy at 5 cm. monitor
o Early amniotomy was associated o Normal baseline HR: 110-160bpm
with a significant 4-hour reduction in o (varies by 6-25
labor duration. o Accelerates appropriately for gest age
o With early amniotomy, however, o Does not decelerate during
there was an increased incidence of contractions
chorioamnionitis. o Fetoscope/stethoscope for
Amniotomy Augmentation auscultation OR
o If manual ausculatation, follow one px to one
o It is common practice to perform attendant ratio
amniotomy when labor is abnormally o Normal pregnancies: get FHR after each
slow contraction every 30 mins during 1st stage
o Amniotomy with oxytocin of labor and every 15 mins during 2nd stage
augmentation for arrested active- o High risk pregnancies: get FHR every 15
phase labor shortened the time to mins during 1st stage and 3-5 mins during
delivery by 44 minutes compared 2nd stage
o Always listen for heart tones AFTER
with that of oxytocin alone. contractions because contractions may
o Although amniotomy did not alter cause a slowing of HR
the delivery route, one drawback o EFM: standard of care for high risk
was that it significantly increased pregnancies and is now used for ALL
the incidence of chorioamnionitis. pregnancies
o Use of amniotomy to enhance o Abnormal EFM findings are confirmed via
progress in active labor, but fetal pulse oximetry (help det if C/S is
needed)
cautions that this may increase the Monitoring Progress of Labor (not discussed but
risks of infection and maternal fever uhmm..ang ikli lang naman wag ka na mag-skip..You
**OPPABELLS STOP SKIPPING AND READ hi are not a hiphopper so do not hop.)
part again.
Check for cervical dilatation - most
Urinary Bladder Function accurate measure of labor progress
Distention of the bladder should be avoided Partograph (sigmoid curve) used to
assess progress of labor and identify when
because it can hinder descent of the fetal
intervention is needed
presenting part and lead to subsequent o Graphical record of cervical
bladder hypotonia and infection dilatation in cm against duration
During each abdominal examination, the of labor in hrs
suprapubic region should be inspected and o Highly effective in reducing
palpated to detect distention. complications from prolonged
If the bladder is readily seen or palpated labor (postpartum hemorrhage,
above the symphysis, the woman should be sepsis, uterine rupture)
encouraged to void. o Plotting begins in the active
phase when cervix is 4cm
If the bladder is distended and voiding is not
dilated
possible, catheterization is indicated. 3 components of partograph:
Most women resumed normal voiding before 1. Fetal condition
discharge from the hospital. 2. Progress of labor
Labor epidural analgesia 3. Maternal condition

o Risk factors for retention were Content of the partograph:
primiparity, oxytocin-induced or - o Px info
augmented labor, perineal o Fetal HR (every half hour)
lacerations, operative vaginal o Amniotic fluid record color
delivery, catheterization during every 
vaginal exam

I: membranes intact

labor, and labor duration > 10 hours. C: membranes
ruptured; clear fluid
Enema and Vulvar & Perineal Preparation (with M: meconium-stained

the new protocol this is no longer recommended but B: blood-stained
again..this varies wi h he ph ician di cre ion) o Moulding

Prevent contamination of newborn with feces 1: sutures apposed 

Cleanse GI tract 2: sutures overlap but
Minimizes infection in episiotomy wound reducible 

Fleet enema 3: sutures overlap but not
o Do early because if you do it at 7 reducible
o cm dilatation, the baby might o Cervical dilatation (begin plot at
come out before she empties the 4cm)
GI tract o Alert line: line starts at 4cm to
Woman in lithotomy position w/ cleansing the point 
of expected full
of vulva and perineal scrubbing downward dilatation at a rate of 1cm/hr) 

and away from introitus You go beyond this IF
C/I: ruptured membranes, sedated dilatation is less than
1cm/hr
o Action line: parallel and 4 hrs to
the right of alert line
You go beyond this if
5
there is delay in progress
(>4hrs)
o Descent assessed by
abdominal palpation
Refers to part of head
palpable above symphysis
pubis

At zero, sinciput is at the
level of symphysis pubis
o Hours time since onset of
labor
o Time Actual time
o Contractions chart every ahlf
an hour
Palpate number of
contractions in 10
minutes and their
duration
o Oxytocin reconrd the
amount of oxy given per
voulume IC fluids
o Drugs given
o Pulse (every 30 minutes)
o Temp. (every 2 hours)
o Protein, acetion and voume
(record each time urine is
passed)
Check for uterine contractions every 30
minutes
o IF high risk, do it every 15
minutes
IE or vaginal exam is done as needed
o Done every 4 hours in latent
phase and after rupture of
membranes
o IF with ruptured membranes,
limit IE (possible infection)

6
INTRAPARTUM ASSESSMENT the frequency of red cells
Sources: Lecture slides, Dra. Coloma lec re, passing under the monitor is
th
William 24 Ed, previous notes from based on heart action so if
upperclassmen the heart pumps, more
blood cells are passing
Conduct of Labor and Delivery through and when the heart
Monitoring relaxes, lesser blood
─ Maternal vessels pass through
Vital signs Blood vessel reflects the
Uterine contractions action of the heart
General condition ─ The maternal pulse must be counted as the
─ Fetal FHR is counted to make sure that it is the
FHR FHR is counted and not the maternal heart
FHT rate
─ Progress of labor Tachycardia of the mother may
dilatation and descent (done by sometimes be misinterpreted as
partograph) FHR. (Example is in cases of
h a idiform mole here mo her
Monitoring the fetus heart rate is increased)
─ Clinical (signs and symptoms) ─ The fetal heart must be auscultated
IMMEDIATELY AFTER CONTRACTION
Auscultation of fetal heart tones
because if there will be changes in the heart
Character of the amniotic fluid
rate, they will be most likely detectable
─ Electronic FHR monitoring (EFM)/ immediately after the uterine contraction
Cardiotocograph ( CTG)
** Changes in the fetal heart rate that are most likely
** LOOK OUT FOR FETAL DISTRESS to to be ominous almost always are detectable
recognize when it will happen and then apply immediately after a uterine contraction
appropriate action so that we can have a healthy Recommendations:
fetus at the end of the delivery In the absence of any abnormalities
If the delivery went on and we have ─ First stage every 30 min
a baby that is not well, it will reflect
─ Second stage every 15 minutes
on the way labor and delivery is
conducted
High-risk pregnancies there are more
Distress is no longer the complications therefore FHR is auscultated more
acceptable term, instead frequently
compromi e i ed when we are
─ First stage every 15 minutes
describing the reason for doing
─ Second stage every 5 minutes
actions such as cesarean section.
** LOOK OUT FOR FETAL COMPROMISE
Suspect compromise if:
AMNIOTIC FLUID ─ FHT repeatedly below 110 bpm, even
though there is recovery to 110-160 bpm
─ Also used as a gauge
** Further labor if allowed should be should be
monitored electronically
If the fetus becomes hypoxic, it will incite
the pituitary to release arginine
Electronic FHR monitoring
vasopressin. Arginine vassopresin is a
ADMISSION TEST (baseline trace)
substance that will increase peristalsis
and with the movement of the bowels, it ─ Hooking the mother to a electronic monitor
is possible that meconium wil be passed and we obtain a baseline trace
so that the amniotic fluid, which is If normal, we can shift to intermittent
naturally colorless or white, becomes auscultation then check again the
stained green We can say that there is trace every 2 hrs (no continuous
fetal compromise based on this trace)
mechanism If abnormal trace or high-risk case,
Meconium staining of amniotic fluid is then it has to be continuous trace
always considered as fetal compromise, If at any point that we get an
unless proven otherwise abnormal trace, from that
point on it will be continuous
trace/ continuous electronic
Fetal hypoxia Pituitary release of Increased intestinal Passage of meconium
arginine vasopressin peristalsis
Auscultation
─ Stethoscope or Doppler devices
Doppler devices small gadgets
that use the principle of transmitting
sounds towards the blood vessels
and then the impulses are bounced
back to it
Although directing the
sound of the Doppler device
towards the blood vessel,
1
Chemo- and baro-receptors
Chemoreceptors senses in
changes in oxygen and
carbon dioxide in the blood
Baroreceptors sensitive to
blood pressure changes
BASELINE RATE
─ Approximate mean rate rounded to
increments of 5 bpm during a 10-minute
segment, minimum interpretable duration of
2 min
The average fetal heart rate is
considered the result of tonic
balance between accelerator and
decelerator influences on the
pacemaker cells
Sympathetic system accelerator
influence
Parasympathetic system
decelerator factor mediated via
vagal slowing of heart rate
─ Normal: 110-150 bpm
─ Bradycardia : < 110 bpm, for at least 3 min.
(more than 2 mins)
moderate 80-100
severe less than 80
A rate between 100 and 119
bpm, in the absence of
other changes, usually is
not considered to represent
fetal compromise may be
attributed to head
compression from occiput
posterior or transverse
positions particularly during
nd
the 2 stage of labor
Bradycardia within the
range pf 80-120bpm with
good variability may be
considered reassuring
<180 bpm nonreassuring
and generally considered
nonreassuring
Congenital fetal heart block
and serious fetal
compromise may cause
bradycardia
─ Tachycardia: > 160 bpm
mild 161-180 bpm
severe > 180 bpm
Maternal fever from
chorioamnionitis most
common explanation of fetal
tachycardia
Other causes: fetal
compromise, arrhythmias
and maternal administration
of parasympathetic
Reading EFM trace
(atropine) or
─ Baseline heart rate
sympathomimetic
─ Baseline variability (terbutaline) drugs
─ Accelerations Wandering baseline baseline rate is
─ Decelerations unsteady and wanders between 120-
Of all the 4, variability has 160; a rare finding associated with
the greatest import neurologically abnormal fetus and may
occur in preterminal event
FHR regulation
─ Under this various systems:
Central nervous system
Autonomic nervous system
- Sympathetic
- Parasympathetic
2
BASELINE VARIABILITY
─ Oscillations at the baseline or jiggliness
─ Ano her erm i a oo hedne (parang
bagong lagare, hindi yung pudpud na
lagare)
─ Is an important index of cardiovascular
function and appears to be regulated by the
autonomic nervous system
A sympathetic and parasympathetic
p h and p ll media ed ia he SA
node produces moment-to-moment
or beat-to-beat oscillation of the
baseline heart rate

Cardiotocograph Variability may be further divided into


- Cardio heart short term and long term
- Toco uterine contractions Short-term variability
reflects instantaneous
How to read uterine contractions (Toco part): change in FHR from one
─ anything around 50 mild beat or R wave to the
next
─ 75 moderate
measures the time interval
─ If at the top point strong contractions
between cardiac systoles
200 units adequate contractions
Long-term variability
used to describe the
oscillatory changes during 1
minute and result in the
waviness of the baseline
normal frequency: 3-5
cycles per minute

- Shows moderate contractions


- Frequency: contractions every3-
4mins

For the FHR (Cardio part):


- Read portions of the trace
where there are no contractions Grades of baseline variability
─ Absent variability only a straight line
Example: ─ Minimal (poor) +/< 5 bpm
─ Moderate = NORMAL 6-25 bpm
─ Marked > 25 bpm

How do we know if it is 5-25?


─ 1 row 10bpm

Example:

- For the most part, the rate is


along 140 to 150 so FHR= 140-
150 normal - Shows moderate variability
Usually, it should be expressed only good/ normal variability
as 1 number. So FHR= 145bpm
But 140, 145 or 150bpm all are If it shows a sawtooth appearance
acceptable good variability
3
If straight bad Up t o 30 weeks baseline
variability is similar during both fetal
rest and activity
>30 weeks fetal inactivity was
associated with diminished baseline
variability and conversely, variability
was increased during fetal activity
Decreased variability
Can be an ominous sign indicating
seriously compromised fetus
Fetal acidemia loss of
variability in combination
with decelerations
Common cause is administration of
analgesic drugs during labor
CNS depressants can
cause transient diminished
beat-to-beat variability
Fetal Hypoxia decreased
variability in the absence of
decelerations
A persistently flat FHR
baseline/ absent variability
within normal baseline rate
range and without
decelerations may reflect
previous insult to the fetus
that has resulted to
neurological damage
** It is generally believed that reduced
baseline heart rate variability is considered the
single most reliable sign of fetal compromise

Sinusoidal heart rate


May be observed in fetal intracranial
hemorrhage, with severe fetal
asphyxia and with severe fetal
anemia from Rh alloimmunization,
fetomaternal hemorrhage, twin-twin
transfusion syndrome or vasa previa
with bleeding
may be attributed to administration
of meperidine, morphine,
alpharodine and butorphanol
shows a sine frequency of 6 cycles
per minute when due to narcotics
has also been described with
chorioamnionitis, fetal distress and
umbilical cord occlusion
not generally associated with fetal
compromise

Increased Variability
Seen during fetal beathing
May be affected by fetal body
movements

4
column is equal to 10
seconds.
─ The heart rate is
considered to have
accelerations if from
baseline, it will
increase to 15bpm
and sustained for 15
seconds (increase in
1 ½ column and 1 ½
row)
─ If it is not sustained
for 15secs only
considered as a
variability, not an
acceleration
From Williams:
- >32 weeks acceleration has a
peak of 15bpm with a duration
of 15 seconds or more but
<2min
- <32 weeks peak of 10bpm for
15 seconds to 2 minutes is
considered normal
- Prolonged accelerations 2
minutes or more but less than
10 minutes

─ DECELERATION
ADVANCING GESTATION - may be correlated with different
─ Parasympathetic dominance (vagus nerve) conditions
As the baby grows older, the - pagbaba from the baseline
parasympathetic become dominate Early Decelerations
Results to: in time with the contractions
- Decreasing rate (pag taas ng contraction,
- Increasing variability bababa yung heart rate, pag
nd
Ex. First and 2 trimester baba ng contraction, babalik
babies HR= 180 OR 160, but yung heart rate sa baseline)
as it grows older like at 30 gradual decrease and return
weeks, HR decreases to baseline associated with
The normal gradual contraction
slowing of the fetal heart rate drop with
heart rate is thought to contractions
correspond to the associated with head
maturation of the compression
parasympathetic (vagal) Head compression may
heart control occur during the pelvic
nd
stage of labor/ 2 stage of
PERIODIC PATTERNS labor (as the baby goes
The baby reacts with the through the passage)
environment, therefore there are Fitting the head through the
accelerations and decelerations. bony passage head
(but mostly accelerations) compression reflected in
─ ACCELERATION the heart rate
Visually abrupt increase above FHR It can be milder compared
baseline defined as onset of to late and variable
acceleration to peak in less than 30 decelerations since it can be
nd
seconds physiologic during the 2
Intact neurohormonal cardiovascular stage of labor (however,
control mechanisms may be abnormal if not in
nd
Favorable sign of fetal well-being the 2 stage of labor or if
baby is still above the
pelvis)

How do we know if there is


acceleration? (According to
Dra. Coloma)
─ There are 6 columns
between 2 dark lines.
So if 6 columns are
equal to 1 min, then 1

5
Variable Decelerations
Late Decelerations Anytime, anywhere; abrupt
pagbaba ng contraction, (kahit saan niya gusto
tsaka pa lang bababa yung pumwesto)
heart rate Defined as an abrupt
smooth, gradual, decrease in FHR beginning
symmetrical decrease in with the onset of the
FHR beginning at or after contraction and reaching
the contraction peak and nadir in less than 30
returning to baseline only seconds
after the contraction has The decrease must last
ended between >15 seconds
Gradual decrease and 2 minutes and must
defined as 30 seconds be >15bpm in amplitude
or more from the onset Associated with cord
of deceleration to the compression
nadir If the cord is compressed
associated with placental (cord is compressed by the
insufficiency head or if hinahawakan ng
Placenta ages there is bata yung cord kasi bored
degeneration with age na bored na siya sa loob
In some cases of kasi 9 months na siyaa
pregnancy, placental loob) FHR changes
degeneration occurs early Cord compression depends
placenta is insufficient on how badly it is
function-wise reflected in compressed sometimes it
the heart rate can be relieved by changing
The only solution for this is the position of the mother
to deliver the baby

6
<10mins from onset to
return to baseline
common causes: cervical
examination, uterine
hyperactivity, cord
entanglement and maternal
supine hypotension
may be due to epidural,
spinal, or paracervical
analgesia

FETAL DISTRESS
too broad and vague to be applied with any
precision to clinical situations
Uncertainty regarding the diagnosis
based on interpretation of FHR
patterns has given rise to
descriptions such as reassuring or
nonreassuring.
Reassuring suggests
a restoration of
Usually, decelerations are associated with confidence by a
some deviations from the normal, which can particular pattern
be excessive to produce compromise Nonreassuring
Whereas accelerations means that baby is inability to remove
reactive and can indicate well being doubt
Late and early decelerations are usually These patterns during
symmetrical appearance (just like Mt. Fuji or labor are dynamic
Mt. Mayon), while variable decelerations are they can rapidly change
abrupt from reassuring to
nonreassuring and vice
Electronic monitoring can be used before versa
and during labor ** These assessments are subjective clinical
No contraction and not in labor non-stress judgments that are inevitably subject to imperfection
test using electronic monitor and must be recognized as such
If with contractions and patient is in labor
Intrapartum INTERPRETATION
FIGO classification
Shouldering ─ Normal
─ There an o er hoo ─ Suspicious
─ Shows that there is compensation ─ Abnormal
If there is no overshoot fetal NICHD
compromise ─ Reassuring = FIGO normal
─ Non-reassuring = FIGO suspicious
─ Ominous = FIGO abnormal

NORMAL TRACE or REASSURING PATTERN


(memorize this!)
─ Baseline rate 110-160 bpm
─ Baseline variability 6-25 bpm good
variability
─ Accelerations present
─ Decelerations absent
Prolonged decelerations
defined as an isolated SUSPICIOUS or NON-REASSURING
deceleration >15bpm lasting ─ One of the 4 features falls into non-
2mins or longer but reassuring category and the remainder are
reassuring
7
If one of the reassuring pattern - For example, loss of variability
is abnormal considered may be hypoxia or may be baby
suspicious or non-reassuring is just sleeping
─ If rate or variability is abnormal - If asleep can be awakened
─ Outside of the normal using vibroacoustic stimulation
(VAS)
ABNORMAL or OMINOUS
─ 2 or more features fall in non-reassuring OR General approach Patients with category
1 or more fall in abnormal II tracings should be evaluated for factors
that may reduce fetal oxygenation, taking
Abnormal (memorize this!) into account associated clinical
─ Baseline rate circumstances
<100, >180 for more than 10 min* Resuscitative measures can be initiated with
Sinusoidal pattern frequent reassessment to determine
There is no variability in whether to perform an operative intervention
sinusoidal and the urgency of the intervention
─ Baseline variability Continued surveillance and frequent
reassessment are indicated until the pattern
<5 for 90min
resolves of progresses to category III
Atypical Variable
─ >60 sec duration, >60 beat drop
─ Late component late in relation to UC or
slow return to baseline
─ Associated with poor variability
─ No shouldering

** Take note of these categories since these will


be used in the exams

CATEGORY I TRACINGS
─ Baseline rate: 110 to 160 beats per minutes
─ Moderate baseline fetal heart rate variability
(amplitude 6 to 25bpm)
─ No late or variable decelerations
─ Early decelerations may be present or
absent
─ Accelerations may be present or absent

** Category I electronic fetal monitoring (EFM)


racing are con idered normal beca e die
have demonstrated that these findings are
associated with the absence of fetal metabolic
academia at the time of observation
** No intervention is indicated
Fetus hypoxic or asleep?
CATEGORY II TRACINGS: DEFINITION AND ─ Vibroacoustic stimulation (VAS) special
MANAGEMENT gadget that emits sounds that can evoke
─ Category II fetal heart rate (FHR) tracings response or stimulate
include all FHR patterns that are not ─ Digital scalp stimulation scratch, pinch or
classified as category I (normal) or category rub the scalp of the baby
III (abnormal) ─ Allis scalp stimulation allis is a clamp with
small teethlike edge which can elicit pain or
CATEGORY III TRACINGS: DEFINITION AND response from the baby if baby is asleep
MANAGEMENT ─ Fetal scalp blood sampling gadget used to
A category III tracing is defined by either of the ake blood from bab calp o de ermine
following criteria: blood gasses
─ Absent baseline fetal heart rate (FHR)
variability and (any of the following): Management approach to non-reassuring FHR
─ Recurrent late decelerations and pattern
─ Recurrent variable decelerations INTRAUTERINE RESUSCITATION
─ Bradycardia 1. Administer oxygen to mother given if baby
─ A sinusoidal pattern is presumed to be hypoxic
2. Reposition patient not specified what
** Category 3 tracings are considered abnormal position but most likely left lateral (reposition
and require prompt evaluation, according to the uterus out of the aorta or vena cava to
ACOG. improve the circulation)
3. Discontinue uterine stimulants oxytocin is
In category II, it is in between so we need to used to improve the contractions but if we
verify. see a nonreassuring pattern, immediately
How do we verify? stop oxytocin drop
4. Hydrate patient IVF
8
These 4 are the basic
management approach

5. Vaginal examination
6. Alert anesthesia, nursing and neonatal care
staff
7. Possible abdominal delivery done if there
is no improvement in the tracing even
though the basic management approaches
were done

Examples:

3. Baseline FHR: 130-140


Variability: poor
Acceleration: (-)
Deceleration: (+) Late deceleration
CATEGORY III

1. Intrapartum? Yes --? Kasi may


contractions
Baseline FHR: 130-140
Variability: moderate
Acceleration: (+)
Deceleration: (-)
CATEGORY I

4. Baseline FHR: 130-140


Variability: none
Acceleration: (-)
Deceleration: (-)
CATEGORY III kasi sinusoidal pattern

2. Baseline FHR: 170-180 tachycardia


Variability: moderate
Acceleration: (-)
Deceleration: (-)
CATEGORY II

9
7. Baseline FHR: 150-160
Variability: moderate
Acceleration: (-)
Deceleration: (+) Variable decelerations
CATEGORY II kasi may variable
decelerations with shouldering/
overshoots

5. Baseline FHR: 140


Variability: none
Acceleration: (-)
Deceleration: (+) late deceleration
CATEGORY III

6. Baseline FHR: 130-140


Variability: morderate
Acceleration: (-)
Deceleration: (+) Early deceleration
CATEGORY I

10
OB Anesthesia
nd
Dr. Irma A. Lee, MD, FPOGS, MHPed Somatic Pain (pain during 2 stage of labor)

BLACK PPT BLUE SuperSubsec ORANGE - LECTURE From distention (due to the stretching) of
pelvic floor, vagina, perineum
Goals of Childbirth Preparation Via somatic nerve fibers transmitted from
pudendal nerve to S2-S4
Patient education concerning pregnancy,
labor and delivery
Relaxation training Analgesia during Labor
Instruction in breathing techniques
Husband (support person) participation A) Regional Analgesia/Anesthesia
Early prenatal bonding
Ideal Anesthetic for Labor & Delivery Epidural Analgesia/Anesthesia
o Most effective method of intrapartum
Safe for mother and baby pain relief
Parturient is conscious, cooperative, o Doe n need o ai for a 4-5cm
comfortable and in control cervical dilatation to be implemented
Will promote EINC (Essential Intrapartum o Catheter can be inserted as early as
Newborn Care) in the latent phase
o Early Parental Bonding core of Placed on the epidural
EINC space
There are no perfec ane he ic or analge ia for Do not puncture the dura
Labor and Delivery, only near ideal and that is
epid ral analge ia. (Dra. Lee, 2014) o (+) regular contractions = give
anesthetics/analgesics
Agent of choice o takes a longer time to take effect as
o based on what is safe for the Mother compared with spinal blockade
and the Baby o Patient still has (+) motor function
o almost all anesthetics especially IV o Uses local anesthetics
and gas INH Bupivacaine, lidocaine,
crosses the placenta ropivacaine, 2-
induces a sedative effect to chloroprocaine, epinephrine
the mother that could lead
to fetal respiratory
depression Epidural Analgesia for Labor and Vaginal Delivery
narcotic analgesics
(opioids) Stage of Pain Pathway Local
To be able to implement the EINC Labor Anesthetics
o < 0.25%
comfortable bupivacaine or
First Stage T10-L1
o ✖ parturient in labor when the baby < 0.20%
Lidocaine
is sleeping
Obstetric Pain Pathways S2-S4
Second Stage Same as above
T10-L1
Visceral Pain

From uterine contractions and cervical Concentration of Local Anesthetics Agent for
dilatation Epidural
o Main problem that an applicant
comes for labor - For Maintenance of Epidural Anesthesia
Via visceral afferent fibers entering spinal Intermittent Bolus Injection
cord at T11-T12 and fibers entering T10 and
L1 Administer initially 7cc of 0.125%
bupivacaine or 0.1% lidocaine after a 3cc
test dose
st
Pain Pathway during the First Stage Same amount is injected once 1 dose
wears off at 5-10cc
Pain of uterine contractions Gradually increase but not to exceed 0.25
for bupivacaine or 0.20% lidocaine to avoid
Uterine plexus motor block
Supplemental doses of local anesthetics
Pelvic ganglia and plexus given for the duration of labor
Results in blockade of sacral segments,
Hypogastric nerve intense motor blockade or both

Superficial hypogastric plexus


Infusion pump (patient-controlled epidural
Lumbar and lower sympathetic chain and white rami anesthesia)
of T11-T12
1
- preferred to be used than a single shot of Complications can still manifest even after 6mos
anesthetics postpartum
o Single shot used for short term
procedures, one time use
- Wait for the progress in labor
o (+) Full cervical dilatation, in need
for episiotomy, assisted vaginal Alternative Regional Anesthetics Techniques
delivery with the use of forceps or
vacuum, or end up with CS delivery Spinal Analgesia / Anesthesia
- Used even as postoperative analgesia o Low Spinal Block or Saddle Block
- More popular for maintaining epidural o For vaginal delivery requiring
anesthesia perineal anesthesia
- Benefits o Total motor inhibition = (-)
o Easy to maintain level of analgesia movement
o More stable maternal VS o Other indications:
o Decrease risk of systemic delivery of preterm fetuses
anesthesia toxicity low forceps delivery
cerclage procedure to
manage cervical
Bupivacaine insufficiency/incompentency
due to recurrent pregnancy
Oftenly used local anesthetic for epidural completion curettage
analgesia o All local anesthetics can be used
Drug of choice o Lidocaine short duration
o Provides essential sensory blockade o Tetracaine & Bupivacaine
with minimal motor blockade Intermediate to long
8-10mL of 0.25% to 0.5% for 2hrs duration
Peak effect achieved in 20 mins
Provide excellent sensory blockade with
nd
minimal motor blockade Saddle Block (Subarachnoid) for 2 Stage of Labor

DO NOT MEMORIZE THE VOLUME OR o Loss of sensory input from the perineum and
THE # OF CC inner thigh
o Hindi pa raw sila gagawa ng mga o Blocks S2-S4 segments
nd
future Anes sa 2 year OB1 o Intended for outlet forceps and vacuum
delivery
o (+) hyperbaric solution = mixture of
anesthetic agent with dextrose
o ask the patient to lie down after
administration
o
Epidural Analgesia

Complicati Contraindications Other Alternative Regional Anesthetic Techniques


ons Indications
Hypotens Patient refusal or Vaginal Paracervical block
ion inability to delivery of
st
spinal or cooperate twins Used during 1 stage of labor
regional (due to Vaginal Introduced intravaginally at about 3 or 9
Inadequa meningitis) delivery of o clock
nd
te infection at the site preterm infants If full cervical dilatation (2 stage
analgesia of needle puncture Pre-eclampsia
High or (psoriasis/leprosy/ Patient with
total eczema) medical No sensory or motor blockade
spinal coagulopathy (ITP) complications Blocks transmission of impulse thru the
respirator uncorrected (mitral paracer ical ganglion (Frankenha en
y arrest maternal stenosis/regurg ganglion)
Urinary hypovolemia itation, SC Unpopular due to Fetal Bradycardia
retention (hemorrhage 2° to injuries, Maternal complications: hematoma,
Headach placenta previa) intracranial systemic local anesthetic toxicity, vasovagal
e inadequate neurovascular syncope)
Postdural training or disease, Agent of choice: 2-chloroprocaine
puncture experience in the asthma) Blocks uterine pain pathway at the pelvic
seizures technique inferior hypogastric ganglia and plexus in the
Meningiti (anesthesiologist) uterosacral ligament 1-1.5cm on the lateral
s vaginal fornices
Back Use of 5-10cc 1% lidocaine or 0.25%
pain bupivacaine

2
nd
Pudendal Nerve Block must know procedure Hindi ka o p ede makapa a ng 2 year if hindi
kayo
Pudendal nerve provides:
maka-iden if a q i ni o.
Sensory innervation for the lower vagina,
vulva and perineum Meperidine HCl (Demerol)
o Mostly used during fetal o Most widely used opioid for labor
delivery/episiotomy analgesia
Motor innervation to perineal muscles and o Given at 25-50mg IV or 50-100mg
external anal sphincter IM q2-4hrs
Transperineal administration not advisable o Can be given even during the latent
o Instead do transvaginal phase
administration o Onset of analgesia 5min after IV
o Pudendal nerve anatomical and 45min after IM
landmark Anticipate that upon
Need to aspirate after delivery, baby will not cry
due to depressive side
Analgesia for spontaneous vaginal delivery effect
and outlet forceps delivery o Often used with phenothiazines
Complications (systemic local anesthetic (Promethazine) to
toxicity, hematoma) vomiting
75mg Meperidine + 25mg

Pudendal Block 50mg Meperidine + 25mg

nd
Safe and effective for the 2 stage of labor, o Treatment of choice for hypertonic
episiotomy, and episiorrhaphy uterine contractions (at latent phase
Uses 10cc 1-2% lidocaine or 0.25% of labor)
bupivacaine o To prevent neonatal respiratory
depression
delivery should be within the
st
Perineal Infiltration 1 hr or more than 4hrs
after IV administration
Most common local anesthetic technique for o Decrease FHR variability 25 min
vaginal delivery after IV or 40min after IM
administration but recovers within
Anesthetics for episiotomy and repair
60mins
infiltrate perineum on the side (along the
area) where you are going to cut
Nalbuphine (Nubain) synthetic form
o Demonstrate ceiling effect for
B) Systemic Analgesia respiratory depression at 30mg
dose
o 10-20mg q4-6hrs
Used when conditions contraindicated the
o onset within 2-3 mins after IV and
use of regional (hemorrhage,
within 15min after IM
coagulopathies)
o Advantages - less maternal nausea
Epidural anesthesia is not available
and vomiting
o Risks for episdural anesthesia when
o Disadvantage - more maternal
there are no anesthesiologist
sedation and dizziness
around (Meningitis,

systemic! Naloxone (Narcan)


o Opioid antagonist to reverse
neonatal respiratory depression
Opioids o Given at 0.1mg/kg of a 1mg/mL IV
o Lipid soluble with low MW easily or IM at thighs of the baby
crosses the placenta
o Causes neonatal respiratory
depression Inhalational Analgesics
o Result in -to-beat variability of Nitrous Oxide
FHR (intrauterine) o Gas Anesthetics
o Do not give to parturients who o Given intermittently as 50%nitrous
oxide in 50% oxygen (Nitronox; 1:1)
by mask or mouthpiece
o For forceps delivery and generalized
NEED TO KNOW FOR THE QUIZ: endotracheal anesthesia
o Part of balanced general anesthesia
o Baseline heartbeat
o Presence or absence of acceleration
o Type of variability present Halogenated Agents
o Deceleration late, variable or early o Volatile anesthetics

3
o Causes dose related uterine SM Spinal (Subarachnoid Block) for CS
relaxation
halothane, enflurane, 15mg 0.5% heavy bupivacaine 3cc or 10mg
isoflurane tetracaine 2cc + epinephrine 1:200000
o For internal podalic version of the (0.0005%)
nd
2 twin, breech decomposition and L3-L4 or L2-L3 using G26 spinal needle
replacement of acute uterine
inversion put 2 IV line, one for
blood one is for halothane or Complications (same as epidural)
isoflurane
o With cardio-depressant and Hypotension due to vasodilatation from
hypotensive effects sympathetic blockade and veno-caval
o Hepatotoxic compression
To al pinal blockade d e o do e of
analgesic
Problems Regarding Use of Inhalational Anesthetics Spinal headache due to CSF leakage
Use smaller spinal gauge needle
Need for specialized vaporizers Convulsions
Concern regarding pollution of labor and Bladder dysfunction
delivery room
Incomplete analgesia
Potential for maternal amnesia Contraindications
Potential for loss of protective airway
reflexes and pulmonary aspiration of gastric Hypotension (ruptured ectopic pregnancy)
contents Coagulopathies
Ob er e hen a he pa ien la Neurologic disorders
meal or oral intake Infection on sites of skin puncture

Intravenous Drugs during Anesthesia Management of Spinal Block Complications


Ketamine (Ketalar)
Sedative, good analgesia prior to delivery Hypotension
Avoid in HPN patients
Uterine displacement to the left
IMPORTANT SIDE EFFECT (remain patient Hydration with 0.5 to 1L of NSS
asleep, do not wake her up) Ephedrine 5-10mg/IV
Total spinal block
Propofol (Diprivan)
Sedation for short surgical procedures Treat associated hypotension
Tracheal intubation
Ventilator support
Thiopental (Pentothal)
Short-acting barbiturate
Used with a muscle relaxant 2. Continuous lumbar epidural block
(succinylcholine) prior to tracheal intubation block is from T8-S5 dermatomes
Induces sleep, poor analgesic opiates are added to avoid motor block
Causes neonatal respiratory depression
nd
Used during 2 stage of labor, short minor 3. Combines Spinal Epidural Techniques
gynecologic procedures (for excisional Provide effective analgesia for labor and
biopsy) cesarean delivery

4. Balanced General Anesthesia


Anesthesia for Cesarean Section Uses nitrous oxide, thiopental and
succinylcholine
1. Spinal Block o Thiopental put patient to sleep
o for elective CS o Succinylcholine SM relaxation
o level of sensory block up to T4 for endotracheal tube insertion
dermatome (below the xyphoid
Used fo
process)
within 5mins
o first stage labor block up
o
to T10 (Level of the
Causes maternal and fetal CNS
umbilicus)
depression
o larger dose of anesthetic agent +
de ro e = h perbaric ol n Major hazard is aspiration pneumonitis
o Tetracaine: 8-10mg
o Bupivacaine: 12mg
o Lidocaine: 50-75mg Prophylaxis for Aspiration during General
Anesthesia

o Fasting for 8hrs

4
o Histamine H2 antagonists to reduce
gastric activity (Cimetidine)
o Sellick maneuver skillful tracheal
intubation with pressure on cricoid
cartilage to occlude esophagus
o NGT
o Awake extubation

5. Local Anesthetic Block for CS


Emergency CS in the absence of
anesthesiologist
To augment patchy regional block given
in an emergency
Requires a lot of LA agents

Non-Pharmacologic Analgesic Techniques

Minimal Training Specialized Training


Emotional support Lamaze/Biofeedback
Touch & massage TENS
Therapeutic use of Acupuncture
heat & cold Hypnosis
Hydrotherapy
Vertical position

5
Partograph

2 Phases of Labor:
1. Latent Phase
- Not much of the contraction that would effect good cervical dilatation
Hypertonic contractions strong contractions yet not effective in dilating the cervix
Allow the patient to rest energy is needed later on
Sedation may be given
- Period of waiting
- Primigravid may take as long as 14- 20 hours
- Multigravid 8 hours
Going beyond this time would mean that the patient may not be going into labor yet
In cases of rupture of membranes needs immediate intervention because it may lead to
infection in the uterine cavity causing amnionitis causing further the non-contractile function of the
uterus
2. Active Phase
- Enough powers/ contractions that would make the cervix dilate in order to have effective delivery of fetus
- 4cm dilatation represents that patient has already entered active phase of labor
- Primigravid cervical dilatation of 1cm/ hour
- Multigravid cervical dilatation of 1.5cm per hour
No progress of cervical dilation in 2 hours arrest in cervical dilatation
Protracted disorder not conforming to the 1cm/hour dilatation; not necessarily warrant cesarean/
abdominal delivery, reassess first the patient in terms of the passenger, passages and the power
(may be given oxytocin, stimulants)
At the time the patient has entered 8cm dilatation, there must be descent, if descent does not
happen, there may be inadequate pelvic diameter
Partograph
- Does not tell us when to do cesarean section
- It cautions us that if there is a problem is the passenger, passages and the power in order to have a more
effective vaginal deliver

Acceleration phase
- Expect cervical dilatation to be happening
1
Phase of Maximum Slope
- More rapid cervical dilatation
The phase of maximum slope tells us that we anticipate a rapid cervical rate of cervical dilatation
compared to the deceleration phase
Deceleration phase
- Descent of the presenting part
- Pelvic division of labor
- Corresponds more to the descent than the cervical dilatation
- There will be a slowing in the cervical dilatation
TRUE LABOR FALSE LABOR
- Regular contractions - Irregular contractions
- Increased intensity of contractions - No change in intensity of contractions
- Increased/ long duration of contractions - Short duration of contractions
- Decreased interval between contractions - No change in interval between
3 regular contractions in a 10 minute period contractions
- Pain/ discomfort starts from the fundus going to - Pain/discomfort is on the lower abdomen
the hypogastric area then to the lumbosacral - Relieved by sedation
area/ lower back
- Not relieved by sedation

Examples:
Case 1: JS, 25 year old primigravid 38-39 weeks
Hypogastric pains radiating to the lumbosacral area
VS: BP= 100/60, PR= 80/min, RR= 20/min
Abdomen: FH= 32cm, FHT= 140bpm; LM 1- breech, LM 2- FBL, LM3- Cephalic
Uterine contractions: 5-6min, 40-50 sec moderate
IE: Cervix 3cm, 80% effaced, LOT, (+) BOW, Station -2

3 hours after, UC every 3-4min, 50secs moderate


IE: Cervix 4cm, 80% effaced, LOT, (+) BOW, Station -2

2 hours after, complained of watery vaginal discharge


UC: 3-4 min, 50-60sec, moderate
Speculum exam: pooling of clear amniotic fluid
IE: Cervix 5cm, 90% effaced, LOA, (-)BOW, station -2

2 hours after, IE: Cervix 8cm dilated, fully effaced, LOA, (-)BOW, station 0
One and a half hour later, she complained of having urge to defecate
IE: Cervix fully dilated, fully effaced, OA, (-)BOW, station +2

Case 2: GS, 20 years old, primi 39-40 weeks


Painful uterine contractions since 4 hours ago with bloody mucoid discharge
PPE: BP= 110/70, FH: 34CM, LM1- breech, LM2- FBR, LM3- cephalic, FHT- 140bpm
IE: Cervix 3 cm dilated, 70% effaced, (+) BOW, cephalic, station 0
After 4 hours, Cervic 5cm, 80%effaced, station 0, ROA
After 4 hours, cervix fully dilated, OA, station +1
2
Case 3: AM, 28 years old, G3P2 (2002) 38-39weeks
Watery vaginal discharge with painful uterine contractions since 3 hours ago
PPE: BP= 120/70, FH= 33cm, LM1- breech, LM2-FBL, LM3- cephalic, FHT= 155bpm
Speculum: (+) pooling of clear amniotic fluid
IE: Cervix 2cm dilated, 70% effaced, (-)BOW, cephalic, station -2
After 5 hours, Cervix 6cm, 80% effaced, station -1, LOA
After 2 hours, cervix 6cm, OA, Station -1
3 hours later, cervix 9cm, fully effaced, station 0
1 hour later, cervix fully dilated, fully effaced, station +1

You might also like